You are on page 1of 71

Conflicts of law :

Art. 15 NCC. Laws relating to family rights and duties, or to the status, condition and legal capacity of
persons are binding upon citizens of the Philippines, even though living abroad. (9a)

ARTICLE IV

Citizenship

SECTION 1. The following are citizens of the Philippines:

(1) Those who are citizens of the Philippines at the time of the adoption of this Constitution;

(2) Those whose fathers or mothers are citizens of the Philippines;

(3) Those born before January 17, 1973, of Filipino mothers, who elect Philippine citizenship upon
reaching the age of majority; and

(4) Those who are naturalized in accordance with law.

SECTION 2. Natural-born citizens are those who are citizens of the Philippines from birth without
having to perform any act to acquire or perfect their Philippine citizenship. Those who elect Philippine
citizenship in accordance with paragraph (3), Section 1 hereof shall be deemed natural-born citizens.

SECTION 3. Philippine citizenship may be lost or reacquired in the manner provided by law.

SECTION 4. Citizens of the Philippines who marry aliens shall retain their citizenship, unless by their
act or omission they are deemed, under the law, to have renounced it.

SECTION 5. Dual allegiance of citizens is inimical to the national interest and shall be dealt with by
law.

[G.R. No. 161434. March 3, 2004]

MARIA JEANETTE C. TECSON and FELIX B. DESIDERIO, JR., petitioners, vs. The COMMISSION
ON ELECTIONS, RONALD ALLAN KELLY POE (a.k.a. FERNANDO POE, JR.) and
VICTORINO X. FORNIER, respondents.

[G.R. No. 161634. March 3, 2004]

ZOILO ANTONIO VELEZ, petitioner, vs. RONALD ALLAN KELLEY POE, a.k.a. FERNANDO POE,
JR., respondent.

[G. R. No. 161824. March 3, 2004]


VICTORINO X. FORNIER, petitioner, vs. HON. COMMISSION ON ELECTIONS and RONALD
ALLAN KELLEY POE, ALSO KNOWN AS FERNANDO POE JR., respondents.

DECISION
VITUG, J.:

Citizenship is a treasured right conferred on those whom the state believes are deserving
of the privilege. It is a precious heritage, as well as an inestimable acquisition, [1] that cannot be
taken lightly by anyone - either by those who enjoy it or by those who dispute it.
Before the Court are three consolidated cases, all of which raise a single question of profound
importance to the nation. The issue of citizenship is brought up to challenge the qualifications of a
presidential candidate to hold the highest office of the land. Our people are waiting for the judgment of
the Court with bated breath. Is Fernando Poe, Jr., the hero of silver screen, and now one of the main
contenders for the presidency, a natural-born Filipino or is he not?
The moment of introspection takes us face to face with Spanish and American colonial roots and
reminds us of the rich heritage of civil law and common law traditions, the fusion resulting in a hybrid
of laws and jurisprudence that could be no less than distinctly Filipino.

Antecedent Case Settings

On 31 December 2003, respondent Ronald Allan Kelly Poe, also known as Fernando Poe, Jr.
(hereinafter "FPJ"), filed his certificate of candidacy for the position of President of the Republic of
the Philippines under the Koalisyon ng Nagkakaisang Pilipino (KNP) Party, in the forthcoming national
elections. In his certificate of candidacy, FPJ, representing himself to be a natural-born citizen of
the Philippines, stated his name to be "Fernando Jr.," or "Ronald Allan" Poe, his date of birth to be 20
August 1939 and his place of birth to be Manila.
Victorino X. Fornier, petitioner in G.R. No. 161824, entitled "Victorino X. Fornier, Petitioner, versus
Hon. Commission on Elections and Ronald Allan Kelley Poe, also known as Fernando Poe, Jr.,
Respondents," initiated, on 09 January 2004, a petition docketed SPA No. 04-003 before the
Commission on Elections ("COMELEC") to disqualify FPJ and to deny due course or to cancel his
certificate of candidacy upon the thesis that FPJ made a material misrepresentation in his certificate of
candidacy by claiming to be a natural-born Filipino citizen when in truth, according to Fornier, his
parents were foreigners; his mother, Bessie Kelley Poe, was an American, and his father, Allan Poe,
was a Spanish national, being the son of Lorenzo Pou, a Spanish subject. Granting, petitioner
asseverated, that Allan F. Poe was a Filipino citizen, he could not have transmitted his Filipino
citizenship to FPJ, the latter being an illegitimate child of an alien mother. Petitioner based the
allegation of the illegitimate birth of respondent on two assertions - first, Allan F. Poe contracted a prior
marriage to a certain Paulita Gomez before his marriage to Bessie Kelley and, second, even if no such
prior marriage had existed, Allan F. Poe, married Bessie Kelly only a year after the birth of respondent.
In the hearing before the Third Division of the COMELEC on 19 January 2004, petitioner, in
support of his claim, presented several documentary exhibits - 1) a copy of the certificate of birth of
FPJ, 2) a certified photocopy of an affidavit executed in Spanish by Paulita Poe y Gomez attesting to
her having filed a case for bigamy and concubinage against the father of respondent, Allan F. Poe,
after discovering his bigamous relationship with Bessie Kelley, 3) an English translation of the affidavit
aforesaid, 4) a certified photocopy of the certificate of birth of Allan F. Poe, 5) a certification issued by
the Director of the Records Management and Archives Office, attesting to the fact that there was no
record in the National Archives that a Lorenzo Poe or Lorenzo Pou resided or entered the Philippines
before 1907, and 6) a certification from the Officer-In-Charge of the Archives Division of the National
Archives to the effect that no available information could be found in the files of the National Archives
regarding the birth of Allan F. Poe.
On his part, respondent, presented twenty-two documentary pieces of evidence, the more
significant ones being - a) a certification issued by Estrella M. Domingo of the Archives Division of the
National Archives that there appeared to be no available information regarding the birth of Allan F. Poe
in the registry of births for San Carlos, Pangasinan, b) a certification issued by the Officer-In-Charge of
the Archives Division of the National Archives that no available information about the marriage of Allan
F. Poe and Paulita Gomez could be found, c) a certificate of birth of Ronald Allan Poe, d) Original
Certificate of Title No. P-2247 of the Registry of Deeds for the Province of Pangasinan, in the name of
Lorenzo Pou, e) copies of Tax Declaration No. 20844, No. 20643, No. 23477 and No. 23478 in the
name of Lorenzo Pou, f) a copy of the certificate of death of Lorenzo Pou, g) a copy of the purported
marriage contract between Fernando Pou and Bessie Kelley, and h) a certification issued by the City
Civil Registrar of San Carlos City, Pangasinan, stating that the records of birth in the said office during
the period of from 1900 until May 1946 were totally destroyed during World War II.
On 23 January 2004, the COMELEC dismissed SPA No. 04-003 for lack of merit. Three days
later, or on 26 January 2004, Fornier filed his motion for reconsideration. The motion was denied
on 06 February 2004 by the COMELEC en banc. On 10 February 2004, petitioner assailed the
decision of the COMELEC before this Court conformably with Rule 64, in relation to Rule 65, of the
Revised Rules of Civil Procedure. The petition, docketed G. R. No. 161824, likewise prayed for a
temporary restraining order, a writ of preliminary injunction or any other resolution that would stay the
finality and/or execution of the COMELEC resolutions.
The other petitions, later consolidated with G. R. No. 161824, would include G. R. No. 161434,
entitled "Maria Jeanette C. Tecson, and Felix B. Desiderio, Jr., vs. The Commission on Elections,
Ronald Allan Kelley Poe (a.k.a. Fernando Poe, Jr.), and Victorino X. Fornier," and the other, docketed
G. R. No. 161634, entitled "Zoilo Antonio G. Velez, vs. Ronald Allan Kelley Poe, a.k.a. Fernando Poe,
Jr.," both challenging the jurisdiction of the COMELEC and asserting that, under Article VII, Section 4,
paragraph 7, of the 1987 Constitution, only the Supreme Court had original and exclusive jurisdiction
to resolve the basic issue on the case.

Jurisdiction of the Court

In G. R. No. 161824
In seeking the disqualification of the candidacy of FPJ and to have the COMELEC deny due
course to or cancel FPJs certificate of candidacy for alleged misrepresentation of a material fact (i.e.,
that FPJ was a natural-born citizen) before the COMELEC, petitioner Fornier invoked Section 78 of the
Omnibus Election Code

Section 78. Petition to deny due course to or cancel a certificate of candidacy. --- A verified petition
seeking to deny due course or to cancel a certificate of candidacy may be filed by any person
exclusively on the ground that any material representation contained therein as required under Section
74 hereof is false

in consonance with the general powers of COMELEC expressed in Section 52 of the Omnibus
Election Code -

Section 52. Powers and functions of the Commission on Elections. In addition to the powers and
functions conferred upon it by the Constitution, the Commission shall have exclusive charge of the
enforcement and administration of all laws relative to the conduct of elections for the purpose of
ensuring free, orderly and honest elections -

and in relation to Article 69 of the Omnibus Election Code which would authorize "any interested party"
to file a verified petition to deny or cancel the certificate of candidacy of any nuisance candidate.
Decisions of the COMELEC on disqualification cases may be reviewed by the Supreme Court per
Rule 64[2] in an action for certiorari under Rule 65[3] of the Revised Rules of Civil Procedure. Section 7,
Article IX, of the 1987 Constitution also reads

"Each Commission shall decide by a majority vote of all its Members any case or matter brought
before it within sixty days from the date of its submission for decision or resolution. A case or matter is
deemed submitted for decision or resolution upon the filing of the last pleading, brief, or memorandum,
required by the rules of the Commission or by the Commission itself. Unless otherwise provided by
this Constitution or by law, any decision, order, or ruling of each Commission may be brought to the
Supreme Court on certiorari by the aggrieved party within thirty days from receipt of a copy thereof."

Additionally, Section 1, Article VIII, of the same Constitution provides that judicial power is vested
in one Supreme Court and in such lower courts as may be established by law which power includes
the duty of the courts of justice to settle actual controversies involving rights which are legally
demandable and enforceable, and to determine whether or not there has been a grave abuse of
discretion amounting to lack or excess of jurisdiction on the part of any branch or instrumentality of the
Government.
It is sufficiently clear that the petition brought up in G. R. No. 161824 was aptly elevated to, and
could well be taken cognizance of by, this Court. A contrary view could be a gross denial to our people
of their fundamental right to be fully informed, and to make a proper choice, on who could or should be
elected to occupy the highest government post in the land.
In G. R. No. 161434 and G. R. No. 161634
Petitioners Tecson, et al., in G. R. No. 161434, and Velez, in G. R. No. 161634, invoke the
provisions of Article VII, Section 4, paragraph 7, of the 1987 Constitution in assailing the jurisdiction of
the COMELEC when it took cognizance of SPA No. 04-003 and in urging the Supreme Court to
instead take on the petitions they directly instituted before it. The Constitutional provision cited reads:

"The Supreme Court, sitting en banc, shall be the sole judge of all contests relating to the election,
returns, and qualifications of the President or Vice-President, and may promulgate its rules for the
purpose."

The provision is an innovation of the 1987 Constitution. The omission in the 1935 and the 1973
Constitution to designate any tribunal to be the sole judge of presidential and vice-presidential
contests, has constrained this Court to declare, in Lopez vs. Roxas,[4] as not (being) justiciable
controversies or disputes involving contests on the elections, returns and qualifications of the
President or Vice-President. The constitutional lapse prompted Congress, on 21 June 1957, to enact
Republic Act No. 1793, "An Act Constituting an Independent Presidential Electoral Tribunal to Try,
Hear and Decide Protests Contesting the Election of the President-Elect and the Vice-President-Elect
of the Philippines and Providing for the Manner of Hearing the Same." Republic Act 1793 designated
the Chief Justice and the Associate Justices of the Supreme Court to be the members of the
tribunal. Although the subsequent adoption of the parliamentary form of government under the 1973
Constitution might have implicitly affected Republic Act No. 1793, the statutory set-up, nonetheless,
would now be deemed revived under the present Section 4, paragraph 7, of the 1987 Constitution.
Ordinary usage would characterize a "contest" in reference to a post-election scenario. Election
contests consist of either an election protest or a quo warranto which, although two distinct remedies,
would have one objective in view, i.e., to dislodge the winning candidate from office. A perusal of the
phraseology in Rule 12, Rule 13, and Rule 14 of the "Rules of the Presidential Electoral Tribunal,"
promulgated by the Supreme Court en banc on 18 April 1992, would support this premise -

Rule 12. Jurisdiction. - The Tribunal shall be the sole judge of all contests relating to the election,
returns, and qualifications of the President or Vice-President of the Philippines.

Rule 13. How Initiated. - An election contest is initiated by the filing of an election protest or a petition
for quo warranto against the President or Vice-President. An election protest shall not include a
petition for quo warranto. A petition for quo warranto shall not include an election protest.

Rule 14. Election Protest. - Only the registered candidate for President or for Vice-President of the
Philippines who received the second or third highest number of votes may contest the election of the
President or the Vice-President, as the case may be, by filing a verified petition with the Clerk of the
Presidential Electoral Tribunal within thirty (30) days after the proclamation of the winner.

The rules categorically speak of the jurisdiction of the tribunal over contests relating to the
election, returns and qualifications of the "President" or "Vice-President", of the Philippines, and not of
"candidates" for President or Vice-President. A quo warranto proceeding is generally defined as being
an action against a person who usurps, intrudes into, or unlawfully holds or exercises a public office.
[5]
In such context, the election contest can only contemplate a post-election scenario. In Rule 14, only
a registered candidate who would have received either the second or third highest number of votes
could file an election protest. This rule again presupposes a post-election scenario.
It is fair to conclude that the jurisdiction of the Supreme Court, defined by Section 4, paragraph 7,
of the 1987 Constitution, would not include cases directly brought before it, questioning the
qualifications of a candidate for the presidency or vice-presidency before the elections are held.
Accordingly, G. R. No. 161434, entitled "Maria Jeanette C. Tecson, et al., vs. Commission on
Elections et al.," and G. R. No. 161634, entitled "Zoilo Antonio Velez vs. Ronald Allan Kelley
Poe a.k.a. Fernando Poe, Jr." would have to be dismissed for want of jurisdiction.
The Citizenship Issue
Now, to the basic issue; it should be helpful to first give a brief historical background on the
concept of citizenship.
Perhaps, the earliest understanding of citizenship was that given by Aristotle, who, sometime in
384 to 322 B.C., described the "citizen" to refer to a man who shared in the administration of justice
and in the holding of an office. [6] Aristotle saw its significance if only to determine the constituency of
the "State," which he described as being composed of such persons who would be adequate in
number to achieve a self-sufficient existence. [7] The concept grew to include one who would both
govern and be governed, for which qualifications like autonomy, judgment and loyalty could be
expected.Citizenship was seen to deal with rights and entitlements, on the one hand, and with
concomitant obligations, on the other.[8] In its ideal setting, a citizen was active in public life and
fundamentally willing to submit his private interests to the general interest of society.
The concept of citizenship had undergone changes over the centuries. In the 18th century, the
concept was limited, by and large, to civil citizenship, which established the rights necessary for
individual freedom, such as rights to property, personal liberty and justice. [9] Its meaning expanded
during the 19th century to include political citizenship, which encompassed the right to participate in
the exercise of political power.[10] The 20th century saw the next stage of the development of social
citizenship, which laid emphasis on the right of the citizen to economic well-being and social security.
[11]
The idea of citizenship has gained expression in the modern welfare state as it so developed
in Western Europe. An ongoing and final stage of development, in keeping with the rapidly shrinking
global village, might well be the internationalization of citizenship.[12]

The Local Setting - from Spanish


Times to the Present

There was no such term as "Philippine citizens" during the Spanish regime but "subjects
of Spain" or "Spanish subjects." [13] In church records, the natives were called 'indios', denoting a low
regard for the inhabitants of the archipelago. Spanish laws on citizenship became highly codified
during the 19th century but their sheer number made it difficult to point to one comprehensive law. Not
all of these citizenship laws of Spain however, were made to apply to the Philippine Islands except for
those explicitly extended by Royal Decrees.[14]
Spanish laws on citizenship were traced back to the Novisima Recopilacion, promulgated in
Spain on 16 July 1805 but as to whether the law was extended to the Philippines remained to be the
subject of differing views among experts; [15] however, three royal decrees were undisputably made
applicable to Spaniards in the Philippines - the Order de la Regencia of 14 August 1841,[16] the Royal
Decree of 23 August 1868 specifically defining the political status of children born in the Philippine
Islands,[17] and finally, the Ley Extranjera de Ultramar of 04 July 1870, which was expressly made
applicable to the Philippines by the Royal Decree of 13 July 1870. [18]
The Spanish Constitution of 1876 was never extended to the Philippine Islands because of the
express mandate of its Article 89, according to which the provisions of the Ultramar among which this
country was included, would be governed by special laws. [19]
It was only the Civil Code of Spain, made effective in this jurisdiction on 18 December 1889,
which came out with the first categorical enumeration of who were Spanish citizens. -

(a) Persons born in Spanish territory,

(b) Children of a Spanish father or mother, even if they were born outside of Spain,

(c) Foreigners who have obtained naturalization papers,

(d) Those who, without such papers, may have become domiciled inhabitants of any town of
the Monarchy.[20]

The year 1898 was another turning point in Philippine history. Already in the state of decline as a
superpower, Spain was forced to so cede her sole colony in the East to an upcoming world power, the
United States. An accepted principle of international law dictated that a change in sovereignty, while
resulting in an abrogation of all political laws then in force, would have no effect on civil laws, which
would remain virtually intact.
The Treaty of Paris was entered into on 10 December 1898 between Spain and the United
States.[21] Under Article IX of the treaty, the civil rights and political status of the native inhabitants of
the territories ceded to the United States would be determined by its Congress -

"Spanish subjects, natives of the Peninsula, residing in the territory over which Spain by the present
treaty relinquishes or cedes her sovereignty may remain in such territory or may remove therefrom,
retaining in either event all their rights of property, including the right to sell or dispose of such property
or of its proceeds; and they shall also have the right to carry on their industry, commerce, and
professions, being subject in respect thereof to such laws as are applicable to foreigners. In case they
remain in the territory they may preserve their allegiance to the Crown of Spain by making, before a
court of record, within a year from the date of the exchange of ratifications of this treaty, a declaration
of their decision to preserve such allegiance; in default of which declaration they shall be held to have
renounced it and to have adopted the nationality of the territory in which they reside.

Thus

"The civil rights and political status of the native inhabitants of the territories hereby ceded to the
United States shall be determined by the Congress." [22]

Upon the ratification of the treaty, and pending legislation by the United States Congress on the
subject, the native inhabitants of the Philippines ceased to be Spanish subjects. Although they did not
become American citizens, they, however, also ceased to be "aliens" under American laws and were
thus issued passports describing them to be citizens of the Philippines entitled to the protection of the
United States.
The term "citizens of the Philippine Islands" appeared for the first time in the Philippine Bill of
1902, also commonly referred to as the Philippine Organic Act of 1902, the first comprehensive
legislation of the Congress of the United States on the Philippines -

".... that all inhabitants of the Philippine Islands continuing to reside therein, who were Spanish
subjects on the 11th day of April, 1891, and then resided in said Islands, and their children born
subsequent thereto, shall be deemed and held to be citizens of the Philippine Islands and as such
entitled to the protection of the United States, except such as shall have elected to preserve their
allegiance to the Crown of Spain in accordance with the provisions of the treaty of peace between the
United States and Spain, signed at Paris, December tenth eighteen hundred and ninety eight." [23]

Under the organic act, a citizen of the Philippines was one who was an inhabitant of the Philippines,
and a Spanish subject on the 11th day of April 1899. The term inhabitant was taken to include 1) a
native-born inhabitant, 2) an inhabitant who was a native of Peninsular Spain, and 3) an inhabitant
who obtained Spanish papers on or before 11 April 1899. [24]
Controversy arose on to the status of children born in the Philippines from 11 April 1899 to 01 July
1902, during which period no citizenship law was extant in the Philippines. Weight was given to the
view, articulated in jurisprudential writing at the time, that the common law principle of jus soli,
otherwise also known as the principle of territoriality, operative in the United States and England,
governed those born in the Philippine Archipelago within that period. [25] More about this later.
In 23 March 1912, the Congress of the United States made the following amendment to the
Philippine Bill of 1902 -

"Provided, That the Philippine Legislature is hereby authorized to provide by law for the acquisition of
Philippine citizenship by those natives of the Philippine Islands who do not come within the foregoing
provisions, the natives of other insular possession of the United States, and such other persons
residing in the Philippine Islands who would become citizens of the United States, under the laws of
the United States, if residing therein."[26]

With the adoption of the Philippine Bill of 1902, the concept of "Philippine citizens" had for the first
time crystallized. The word "Filipino" was used by William H. Taft, the first Civil Governor General in
the Philippines when he initially made mention of it in his slogan, "The Philippines for the Filipinos." In
1916, the Philippine Autonomy Act, also known as the Jones Law restated virtually the provisions of
the Philippine Bill of 1902, as so amended by the Act of Congress in 1912 -

That all inhabitants of the Philippine Islands who were Spanish subjects on the eleventh day of
April, eighteen hundred and ninety-nine, and then resided in said Islands, and their children
born subsequently thereto, shall be deemed and held to be citizens of the Philippine Islands,
except such as shall have elected to preserve their allegiance to the Crown of Spain in accordance
with the provisions of the treaty of peace between the United States and Spain, signed at Paris
December tenth, eighteen hundred and ninety-eight and except such others as have since become
citizens of some other country; Provided, That the Philippine Legislature, herein provided for, is hereby
authorized to provide for the acquisition of Philippine citizenship by those natives of the Philippine
Islands who do not come within the foregoing provisions, the natives of the insular possessions of the
United States, and such other persons residing in the Philippine Islands who are citizens of the United
States, or who could become citizens of the United States under the laws of the United States, if
residing therein."

Under the Jones Law, a native-born inhabitant of the Philippines was deemed to be a citizen of
the Philippines as of 11 April 1899 if he was 1) a subject of Spain on 11 April 1899, 2) residing in the
Philippines on said date, and, 3) since that date, not a citizen of some other country.
While there was, at one brief time, divergent views on whether or not jus soli was a mode of
acquiring citizenship, the 1935 Constitution brought to an end to any such link with common law, by
adopting, once and for all, jus sanguinis or blood relationship as being the basis of Filipino citizenship -

Section 1, Article III, 1935 Constitution. The following are citizens of the Philippines -

(1) Those who are citizens of the Philippine Islands at the time of the adoption of this Constitution

(2) Those born in the Philippines Islands of foreign parents who, before the adoption of this
Constitution, had been elected to public office in the Philippine Islands.

(3) Those whose fathers are citizens of the Philippines.

(4) Those whose mothers are citizens of the Philippines and upon reaching the age of majority, elect
Philippine citizenship.

(5) Those who are naturalized in accordance with law.

Subsection (4), Article III, of the 1935 Constitution, taken together with existing civil law provisions
at the time, which provided that women would automatically lose their Filipino citizenship and acquire
that of their foreign husbands, resulted in discriminatory situations that effectively incapacitated the
women from transmitting their Filipino citizenship to their legitimate children and required illegitimate
children of Filipino mothers to still elect Filipino citizenship upon reaching the age of majority. Seeking
to correct this anomaly, as well as fully cognizant of the newly found status of Filipino women as
equals to men, the framers of the 1973 Constitution crafted the provisions of the new Constitution on
citizenship to reflect such concerns -

Section 1, Article III, 1973 Constitution - The following are citizens of the Philippines:

(1) Those who are citizens of the Philippines at the time of the adoption of this Constitution.

(2) Those whose fathers or mothers are citizens of the Philippines.

(3) Those who elect Philippine citizenship pursuant to the provisions of the Constitution of nineteen
hundred and thirty-five.

(4) Those who are naturalized in accordance with law.

For good measure, Section 2 of the same article also further provided that

"A female citizen of the Philippines who marries an alien retains her Philippine citizenship, unless by
her act or omission she is deemed, under the law to have renounced her citizenship."

The 1987 Constitution generally adopted the provisions of the 1973 Constitution, except for
subsection (3) thereof that aimed to correct the irregular situation generated by the
questionable proviso in the 1935 Constitution.
Section I, Article IV, 1987 Constitution now provides:

The following are citizens of the Philippines:

(1) Those who are citizens of the Philippines at the time of the adoption of this Constitution.

(2) Those whose fathers or mothers are citizens of the Philippines.

(3) Those born before January 17, 1973 of Filipino mothers, who elect Philippine citizenship
upon reaching the age of majority; and

(4) Those who are naturalized in accordance with law.

The Case Of FPJ

Section 2, Article VII, of the 1987 Constitution expresses:

"No person may be elected President unless he is a natural-born citizen of the Philippines, a
registered voter, able to read and write, at least forty years of age on the day of the election, and a
resident of the Philippines for at least ten years immediately preceding such election."

The term "natural-born citizens," is defined to include "those who are citizens of the Philippines
from birth without having to perform any act to acquire or perfect their Philippine citizenship." [27]
The date, month and year of birth of FPJ appeared to be 20 August 1939 during the regime of the
1935 Constitution. Through its history, four modes of acquiring citizenship - naturalization, jus soli, res
judicata and jus sanguinis[28] had been in vogue. Only two, i.e., jus soli and jus sanguinis, could qualify
a person to being a natural-born citizen of the Philippines. Jus soli, per Roa vs. Collector of
Customs[29] (1912), did not last long. With the adoption of the 1935 Constitution and the reversal
of Roa in Tan Chong vs. Secretary of Labor[30] (1947), jus sanguinis or blood relationship would now
become the primary basis of citizenship by birth.
Documentary evidence adduced by petitioner would tend to indicate that the earliest established
direct ascendant of FPJ was his paternal grandfather Lorenzo Pou, married to Marta Reyes, the father
of Allan F. Poe. While the record of birth of Lorenzo Pou had not been presented in evidence, his
death certificate, however, identified him to be a Filipino, a resident of San Carlos, Pangasinan, and 84
years old at the time of his death on 11 September 1954. The certificate of birth of the father of FPJ,
Allan F. Poe, showed that he was born on 17 May 1915 to an Espaol father, Lorenzo Pou, and a
mestiza Espaol mother, Marta Reyes. Introduced by petitioner was an uncertified copy of a supposed
certificate of the alleged marriage of Allan F. Poe and Paulita Gomez on 05 July 1936. The marriage
certificate of Allan F. Poe and Bessie Kelley reflected the date of their marriage to be on 16 September
1940. In the same certificate, Allan F. Poe was stated to be twenty-five years old, unmarried, and a
Filipino citizen, and Bessie Kelley to be twenty-two years old, unmarried, and an American citizen. The
birth certificate of FPJ, would disclose that he was born on 20 August 1939 to Allan F. Poe, a Filipino,
twenty-four years old, married to Bessie Kelly, an American citizen, twenty-one years old and married.
Considering the reservations made by the parties on the veracity of some of the entries on the
birth certificate of respondent and the marriage certificate of his parents, the only conclusions that
could be drawn with some degree of certainty from the documents would be that -

1. The parents of FPJ were Allan F. Poe and Bessie Kelley;

2. FPJ was born to them on 20 August 1939;

3. Allan F. Poe and Bessie Kelley were married to each other on 16 September, 1940;

4. The father of Allan F. Poe was Lorenzo Poe; and

5. At the time of his death on 11 September 1954, Lorenzo Poe was 84 years old.

Would the above facts be sufficient or insufficient to establish the fact that FPJ is a natural-born
Filipino citizen? The marriage certificate of Allan F. Poe and Bessie Kelley, the birth certificate of FPJ,
and the death certificate of Lorenzo Pou are documents of public record in the custody of a public
officer. The documents have been submitted in evidence by both contending parties during the
proceedings before the COMELEC.
The birth certificate of FPJ was marked Exhibit "A" for petitioner and Exhibit "3" for
respondent. The marriage certificate of Allan F. Poe to Bessie Kelley was submitted as Exhibit "21" for
respondent. The death certificate of Lorenzo Pou was submitted by respondent as his Exhibit
"5." While the last two documents were submitted in evidence for respondent, the admissibility thereof,
particularly in reference to the facts which they purported to show, i.e., the marriage certificate in
relation to the date of marriage of Allan F. Poe to Bessie Kelley and the death certificate relative to the
death of Lorenzo Pou on 11 September 1954 in San Carlos, Pangasinan, were all admitted by
petitioner, who had utilized those material statements in his argument. All three documents were
certified true copies of the originals.

Section 3, Rule 130, Rules of Court states that -

Original document must be produced; exceptions. - When the subject of inquiry is the contents of a
document, no evidence shall be admissible other than the original document itself, except in the
following cases:

xxxxxxxxx

(d) When the original is a public record in the custody of a public office or is recorded in a public office.
Being public documents, the death certificate of Lorenzo Pou, the marriage certificate of Allan F. Poe
and Bessie Kelly, and the birth certificate of FPJ,constitute prima facie proof of their contents. Section
44, Rule 130, of the Rules of Court provides:

Entries in official records. Entries in official records made in the performance of his duty by a public
officer of the Philippines, or by a person in the performance of a duty specially enjoined by law,
are prima facie evidence of the facts therein stated.

The trustworthiness of public documents and the value given to the entries made therein could be
grounded on 1) the sense of official duty in the preparation of the statement made, 2) the penalty
which is usually affixed to a breach of that duty, 3) the routine and disinterested origin of most such
statements, and 4) the publicity of record which makes more likely the prior exposure of such errors as
might have occurred.[31]
The death certificate of Lorenzo Pou would indicate that he died on 11 September 1954, at the
age of 84 years, in San Carlos, Pangasinan. It could thus be assumed that Lorenzo Pou was born
sometime in the year 1870 when the Philippines was still a colony of Spain. Petitioner would argue
that Lorenzo Pou was not in the Philippines during the crucial period of from 1898 to 1902 considering
that there was no existing record about such fact in the Records Management and Archives
Office. Petitioner, however, likewise failed to show that Lorenzo Pou was at any other place during the
same period. In his death certificate, the residence of Lorenzo Pou was stated to be San Carlos,
Pangasinan. In the absence of any evidence to the contrary, it should be sound to conclude, or at least
to presume, that the place of residence of a person at the time of his death was also his residence
before death. It would be extremely doubtful if the Records Management and Archives Office would
have had complete records of all residents of the Philippines from 1898 to 1902.

Proof of Paternity and Filiation


Under Civil Law.

Petitioner submits, in any case, that in establishing filiation (relationship or civil status of the child
to the father [or mother]) or paternity (relationship or civil status of the father to the child) of an
illegitimate child, FPJ evidently being an illegitimate son according to petitioner, the mandatory rules
under civil law must be used.
Under the Civil Code of Spain, which was in force in the Philippines from 08 December 1889 up
until the day prior to 30 August 1950 when the Civil Code of the Philippines took effect,
acknowledgment was required to establish filiation or paternity. Acknowledgment was either judicial
(compulsory) or voluntary. Judicial or compulsory acknowledgment was possible only if done during
the lifetime of the putative parent; voluntary acknowledgment could only be had in a record of birth, a
will, or a public document.[32] Complementary to the new code was Act No. 3753 or the Civil Registry
Law expressing in Section 5 thereof, that -

In case of an illegitimate child, the birth certificate shall be signed and sworn to jointly by the parents
of the infant or only by the mother if the father refuses. In the latter case, it shall not be permissible to
state or reveal in the document the name of the father who refuses to acknowledge the child, or to give
therein any information by which such father could be identified.

In order that the birth certificate could then be utilized to prove voluntary acknowledgment of filiation or
paternity, the certificate was required to be signed or sworn to by the father. The failure of such
requirement rendered the same useless as being an authoritative document of recognition.
[33]
In Mendoza vs. Mella,[34] the Court ruled -

"Since Rodolfo was born in 1935, after the registry law was enacted, the question here really is
whether or not his birth certificate (Exhibit 1), which is merely a certified copy of the registry record,
may be relied upon as sufficient proof of his having been voluntarily recognized. No such reliance, in
our judgment, may be placed upon it. While it contains the names of both parents, there is no showing
that they signed the original, let alone swore to its contents as required in Section 5 of Act No.
3753. For all that might have happened, it was not even they or either of them who furnished the data
to be entered in the civil register. Petitioners say that in any event the birth certificate is in the nature of
a public document wherein voluntary recognition of a natural child may also be made, according to the
same Article 131. True enough, but in such a case, there must be a clear statement in the document
that the parent recognizes the child as his or her own."

In the birth certificate of respondent FPJ, presented by both parties, nowhere in the document
was the signature of Allan F. Poe found. There being no will apparently executed, or at least shown to
have been executed, by decedent Allan F. Poe, the only other proof of voluntary recognition remained
to be "some other public document." In Pareja vs. Pareja,[35] this Court defined what could constitute
such a document as proof of voluntary acknowledgment:

"Under the Spanish Civil Code there are two classes of public documents, those executed by private
individuals which must be authenticated by notaries, and those issued by competent public
officials by reason of their office. The public document pointed out in Article 131 as one of the means
by which recognition may be made belongs to the first class."

Let us leave it at that for the moment.


The 1950 Civil Code categorized the acknowledgment or recognition of illegitimate children into
voluntary, legal or compulsory. Voluntary recognition was required to be expressedly made in a record
of birth, a will, a statement before a court of record or in any authentic writing. Legal acknowledgment
took place in favor of full blood brothers and sisters of an illegitimate child who was recognized or
judicially declared as natural.Compulsory acknowledgment could be demanded generally in cases
when the child had in his favor any evidence to prove filiation. Unlike an action to claim legitimacy
which would last during the lifetime of the child, and might pass exceptionally to the heirs of the child,
an action to claim acknowledgment, however, could only be brought during the lifetime of the
presumed parent.
Amicus Curiae Ruben F. Balane defined, during the oral argument, "authentic writing," so as to be
an authentic writing for purposes of voluntary recognition, simply as being a genuine or indubitable
writing of the father. The term would include a public instrument (one duly acknowledged before a
notary public or other competent official) or a private writing admitted by the father to be his.
The Family Code has further liberalized the rules; Article 172, Article 173, and Article 175 provide:

Art. 172. The filiation of legitimate children is established by any of the following:

(1) The record of birth appearing in the civil register or a final judgment; or

(2) An admission of legitimate filiation in a public document or a private handwritten instrument and
signed by the parent concerned.

In the absence of the foregoing evidence, the legitimate filiation shall be proved by:

(1) The open and continuous possession of the status of a legitimate child; or

(2) Any other means allowed by the Rules of Court and special laws.

Art. 173. The action to claim legitimacy may be brought by the child during his or her lifetime and shall
be transmitted to the heirs should the child die during minority or in a state of insanity. In these cases,
the heirs shall have a period of five years within which to institute the action.

The action already commenced by the child shall survive notwithstanding the death of either or both of
the parties.

x x x x x x x x x.

Art. 175. Illegitimate children may establish their illegitimate filiation in the same way and on the same,
evidence as legitimate children.
The action must be brought within the same period specified in Article 173, except when the action is
based on the second paragraph of Article 172, in which case the action may be brought during the
lifetime of the alleged parent.

The provisions of the Family Code are retroactively applied; Article 256 of the code reads:

"Art. 256. This Code shall have retroactive effect insofar as it does not prejudice or impair vested or
acquired rights in accordance with the Civil Code or other laws.

Thus, in Vda. de Sy-Quia vs. Court of Appeals,[36] the Court has ruled:

"We hold that whether Jose was a voluntarily recognized natural child should be decided under Article
278 of the Civil Code of the Philippines. Article 2260 of that Code provides that 'the voluntary
recognition of a natural child shall take place according to this Code, even if the child was born before
the effectivity of this body of laws' or before August 30, 1950. Hence, Article 278 may be given
retroactive effect."

It should be apparent that the growing trend to liberalize the acknowledgment or recognition of
illegitimate children is an attempt to break away from the traditional idea of keeping well apart
legitimate and non-legitimate relationships within the family in favor of the greater interest and welfare
of the child. The provisions are intended to merely govern the private and personal affairs of the
family. There is little, if any, to indicate that the legitimate or illegitimate civil status of the individual
would also affect his political rights or, in general, his relationship to the State. While, indeed,
provisions on "citizenship" could be found in the Civil Code, such provisions must be taken in the
context of private relations, the domain of civil law; particularly -

"Civil Law is that branch of law which has for its double purpose the organization of the family and the
regulation of property. It has thus [been] defined as the mass of precepts which determine and
regulate the relations of assistance, authority and obedience among members of a family, and those
which exist among members of a society for the protection of private interests." [37]

In Yaez de Barnuevo vs. Fuster,[38] the Court has held:

"In accordance with Article 9 of the Civil Code of Spain, x x x the laws relating to family rights and
duties, or to the status, condition and legal capacity of persons, govern Spaniards although they reside
in a foreign country; that, in consequence, 'all questions of a civil nature, such as those dealing with
the validity or nullity of the matrimonial bond, the domicile of the husband and wife, their support, as
between them, the separation of their properties, the rules governing property, marital authority,
division of conjugal property, the classification of their property, legal causes for divorce, the extent of
the latter, the authority to decree it, and, in general, the civil effects of marriage and divorce upon the
persons and properties of the spouses, are questions that are governed exclusively by the national law
of the husband and wife."

The relevance of "citizenship" or "nationality" to Civil Law is best exemplified in Article 15 of the
Civil Code, stating that -

"Laws relating to family rights and duties, or to the status, condition and legal capacity of persons are
binding upon citizens of the Philippines, even though living abroad" -

that explains the need to incorporate in the code a reiteration of the Constitutional provisions on
citizenship. Similarly, citizenship is significant in civil relationships found in different parts of the Civil
Code,[39] such as on successional rights and family relations. [40] In adoption, for instance, an adopted
child would be considered the child of his adoptive parents and accorded the same rights as their
legitimate child but such legal fiction extended only to define his rights under civil law [41] and not his
political status.
Civil law provisions point to an obvious bias against illegitimacy. This discriminatory attitude may
be traced to the Spanish family and property laws, which, while defining proprietary and successional
rights of members of the family, provided distinctions in the rights of legitimate and illegitimate
children. In the monarchial set-up of old Spain, the distribution and inheritance of titles and wealth
were strictly according to bloodlines and the concern to keep these bloodlines uncontaminated by
foreign blood was paramount.
These distinctions between legitimacy and illegitimacy were codified in the Spanish Civil Code,
and the invidious discrimination survived when the Spanish Civil Code became the primary source of
our own Civil Code. Such distinction, however, remains and should remain only in the sphere of civil
law and not unduly impede or impinge on the domain of political law.
The proof of filiation or paternity for purposes of determining his citizenship status should thus be
deemed independent from and not inextricably tied up with that prescribed for civil law purposes. The
Civil Code or Family Code provisions on proof of filiation or paternity, although good law, do not have
preclusive effects on matters alien to personal and family relations. The ordinary rules on evidence
could well and should govern. For instance, the matter about pedigree is not necessarily precluded
from being applicable by the Civil Code or Family Code provisions.
Section 39, Rule 130, of the Rules of Court provides -

Act or Declaration about pedigree. The act or declaration of a person deceased, or unable to testify, in
respect to the pedigree of another person related to him by birth or marriage, may be received in
evidence where it occurred before the controversy, and the relationship between the two persons is
shown by evidence other than such act or declaration. The word `pedigree includes relationship, family
genealogy, birth, marriage, death, the dates when and the places where these facts occurred, and the
names of the relatives. It embraces also facts of family history intimately connected with pedigree.

For the above rule to apply, it would be necessary that (a) the declarant is already dead or unable
to testify, (b) the pedigree of a person must be at issue, (c) the declarant must be a relative of the
person whose pedigree is in question, (d) declaration must be made before the controversy has
occurred, and (e) the relationship between the declarant and the person whose pedigree is in question
must be shown by evidence other than such act or declaration.
Thus, the duly notarized declaration made by Ruby Kelley Mangahas, sister of Bessie Kelley Poe
submitted as Exhibit 20 before the COMELEC, might be accepted to prove the acts of Allan F. Poe,
recognizing his own paternal relationship with FPJ, i.e, living together with Bessie Kelley and his
children (including respondent FPJ) in one house, and as one family -

"I, Ruby Kelley Mangahas, of legal age and sound mind, presently residing in Stockton, California,
U.S.A., after being sworn in accordance with law do hereby declare that:

1. I am the sister of the late Bessie Kelley Poe.

2. Bessie Kelley Poe was the wife of Fernando Poe, Sr.

3. Fernando and Bessie Poe had a son by the name of Ronald Allan Poe, more popularly
known in the Philippines as `Fernando Poe, Jr., or `FPJ.

4. Ronald Allan Poe `FPJ was born on August 20, 1939 at St. Luke's Hospital, Magdalena
Street, Manila.

xxxxxxxxx

7. Fernando Poe Sr., and my sister Bessie, met and became engaged while they were
students at the University of the Philippines in 1936. I was also introduced to
Fernando Poe, Sr., by my sister that same year.

8. Fernando Poe, Sr., and my sister Bessie had their first child in 1938.
9. Fernando Poe, Sr., my sister Bessie and their first three children, Elizabeth, Ronald, Allan
and Fernando II, and myself lived together with our mother at our family's house on
Dakota St. (now Jorge Bocobo St.), Malate until the liberation of Manila in 1945,
except for some months between 1943-1944.

10. Fernando Poe, Sr., and my sister, Bessie, were blessed with four (4) more children after
Ronald Allan Poe.

xxxxxxxxx

18. I am executing this Declaration to attest to the fact that my nephew, Ronald Allan Poe is
a natural born Filipino, and that he is the legitimate child of Fernando Poe, Sr.

Done in City of Stockton, California, U.S.A., this 12th day of January 2004.

Ruby Kelley Mangahas

Declarant

DNA Testing

In case proof of filiation or paternity would be unlikely to satisfactorily establish or would be


difficult to obtain, DNA testing, which examines genetic codes obtained from body cells of the
illegitimate child and any physical residue of the long dead parent could be resorted to. A positive
match would clear up filiation or paternity. In Tijing vs. Court of Appeals,[42] this Court has
acknowledged the strong weight of DNA testing -

"Parentage will still be resolved using conventional methods unless we adopt the modern and
scientific ways available. Fortunately, we have now the facility and expertise in using DNA test for
identification and parentage testing. The University of the Philippines Natural Science Research
Institute (UP-NSRI) DNA Analysis Laboratory has now the capability to conduct DNA typing using short
tandem repeat (STR) analysis. The analysis is based on the fact that the DNA of a child/person has
two (2) copies, one copy from the mother and the other from the father. The DNA from the mother, the
alleged father and the child are analyzed to establish parentage. Of course, being a novel scientific
technique, the use of DNA test as evidence is still open to challenge. Eventually, as the appropriate
case comes, courts should not hesitate to rule on the admissibility of DNA evidence. For it was said,
that courts should apply the results of science when competently obtained in aid of situations
presented, since to reject said result is to deny progress."

Petitioners Argument For


Jurisprudential Conclusiveness

Petitioner would have it that even if Allan F. Poe were a Filipino citizen, he could not have
transmitted his citizenship to respondent FPJ, the latter being an illegitimate child. According to
petitioner, prior to his marriage to Bessie Kelley, Allan F. Poe, on July 5, 1936, contracted marriage
with a certain Paulita Gomez, making his subsequent marriage to Bessie Kelley bigamous and
respondent FPJ an illegitimate child. The veracity of the supposed certificate of marriage between
Allan F. Poe and Paulita Gomez could be most doubtful at best. But the documentary evidence
introduced by no less than respondent himself, consisting of a birth certificate of respondent and a
marriage certificate of his parents showed that FPJ was born on 20 August 1939 to a Filipino father
and an American mother who were married to each other a year later, or on 16 September 1940. Birth
to unmarried parents would make FPJ an illegitimate child. Petitioner contended that as an illegitimate
child, FPJ so followed the citizenship of his mother, Bessie Kelley, an American citizen, basing his
stand on the ruling of this Court in Morano vs. Vivo,[43] citing Chiongbian vs. de Leon[44] and Serra vs.
Republic.[45]
On the above score, the disquisition made by amicus curiae Joaquin G. Bernas, SJ, is most
convincing; he states -

"We must analyze these cases and ask what the lis mota was in each of them. If the pronouncement
of the Court on jus sanguinis was on the lis mota, the pronouncement would be a decision constituting
doctrine under the rule of stare decisis. But if the pronouncement was irrelevant to the lis mota, the
pronouncement would not be a decision but a mere obiter dictum which did not establish doctrine. I
therefore invite the Court to look closely into these cases.

First, Morano vs. Vivo. The case was not about an illegitimate child of a Filipino father. It was about a
stepson of a Filipino, a stepson who was the child of a Chinese mother and a Chinese father. The
issue was whether the stepson followed the naturalization of the stepfather. Nothing about jus
sanguinis there. The stepson did not have the blood of the naturalized stepfather.

Second, Chiongbian vs. de Leon. This case was not about the illegitimate son of a Filipino father. It
was about a legitimate son of a father who had become Filipino by election to public office before the
1935 Constitution pursuant to Article IV, Section 1(2) of the 1935 Constitution. No one was illegitimate
here.

Third, Serra vs. Republic. The case was not about the illegitimate son of a Filipino father. Serra was an
illegitimate child of a Chinese father and a Filipino mother.The issue was whether one who was
already a Filipino because of his mother who still needed to be naturalized. There is nothing there
about invidious jus sanguinis.

Finally, Paa vs. Chan.[46] This is a more complicated case. The case was about the citizenship of
Quintin Chan who was the son of Leoncio Chan. Quintin Chan claimed that his father, Leoncio, was
the illegitimate son of a Chinese father and a Filipino mother. Quintin therefore argued that he got his
citizenship from Leoncio, his father. But the Supreme Court said that there was no valid proof that
Leoncio was in fact the son of a Filipina mother. The Court therefore concluded that Leoncio was not
Filipino. If Leoncio was not Filipino, neither was his son Quintin. Quintin therefore was not only not a
natural-born Filipino but was not even a Filipino.

The Court should have stopped there. But instead it followed with an obiter dictum. The Court
said obiter that even if Leoncio, Quintin's father, were Filipino, Quintin would not be Filipino because
Quintin was illegitimate. This statement about Quintin, based on a contrary to fact assumption, was
absolutely unnecessary for the case. x x x It was obiter dictum, pure and simple, simply repeating the
obiter dictum in Morano vs. Vivo.

xxxxxxxxx

"Aside from the fact that such a pronouncement would have no textual foundation in the Constitution, it
would also violate the equal protection clause of the Constitution not once but twice. First, it would
make an illegitimate distinction between a legitimate child and an illegitimate child, and second, it
would make an illegitimate distinction between the illegitimate child of a Filipino father and the
illegitimate child of a Filipino mother.

The doctrine on constitutionally allowable distinctions was established long ago by People vs. Cayat.
[47]
I would grant that the distinction between legitimate children and illegitimate children rests on real
differences. x x x But real differences alone do not justify invidious distinction. Real differences may
justify distinction for one purpose but not for another purpose.

x x x What is the relevance of legitimacy or illegitimacy to elective public service? What possible state
interest can there be for disqualifying an illegitimate child from becoming a public officer. It was not the
fault of the child that his parents had illicit liaison. Why deprive the child of the fullness of political rights
for no fault of his own? To disqualify an illegitimate child from holding an important public office is to
punish him for the indiscretion of his parents. There is neither justice nor rationality in that. And if there
is neither justice nor rationality in the distinction, then the distinction transgresses the equal protection
clause and must be reprobated.

The other amici curiae, Mr. Justice Vicente Mendoza (a former member of this Court), Professor
Ruben Balane and Dean Martin Magallona, at bottom, have expressed similar views. The thesis of
petitioner, unfortunately hinging solely on pure obiter dicta, should indeed fail.
Where jurisprudence regarded an illegitimate child as taking after the citizenship of its mother, it
did so for the benefit the child. It was to ensure a Filipino nationality for the illegitimate child of an alien
father in line with the assumption that the mother had custody, would exercise parental authority and
had the duty to support her illegitimate child. It was to help the child, not to prejudice or discriminate
against him.
The fact of the matter perhaps the most significant consideration is that the 1935 Constitution, the
fundamental law prevailing on the day, month and year of birth of respondent FPJ, can never be more
explicit than it is. Providing neither conditions nor distinctions, the Constitution states that among the
citizens of the Philippines are those whose fathers are citizens of the Philippines. There utterly is no
cogent justification to prescribe conditions or distinctions where there clearly are none provided.

In Sum

(1) The Court, in the exercise of its power of judicial review, possesses jurisdiction over the
petition in G. R. No. 161824, filed under Rule 64, in relation to Rule 65, of the Revised Rules of Civil
Procedure. G.R. No. 161824 assails the resolution of the COMELEC for alleged grave abuse of
discretion in dismissing, for lack of merit, the petition in SPA No. 04-003 which has prayed for the
disqualification of respondent FPJ from running for the position of President in the 10 th May 2004
national elections on the contention that FPJ has committed material representation in his certificate of
candidacy by representing himself to be a natural-born citizen of the Philippines.
(2) The Court must dismiss, for lack of jurisdiction and prematurity, the petitions in G. R. No.
161434 and No. 161634 both having been directly elevated to this Court in the latters capacity as the
only tribunal to resolve a presidential and vice-presidential election contest under the
Constitution.Evidently, the primary jurisdiction of the Court can directly be invoked only after, not
before, the elections are held.
(3) In ascertaining, in G.R. No. 161824, whether grave abuse of discretion has been committed
by the COMELEC, it is necessary to take on the matter of whether or not respondent FPJ is a natural-
born citizen, which, in turn, depended on whether or not the father of respondent, Allan F. Poe, would
have himself been a Filipino citizen and, in the affirmative, whether or not the alleged illegitimacy of
respondent prevents him from taking after the Filipino citizenship of his putative father. Any conclusion
on the Filipino citizenship of Lorenzo Pou could only be drawn from the presumption that having died
in 1954 at 84 years old, Lorenzo would have been born sometime in the year 1870, when the
Philippines was under Spanish rule, and that San Carlos, Pangasinan, his place of residence upon his
death in 1954, in the absence of any other evidence, could have well been his place of residence
before death, such that Lorenzo Pou would have benefited from the en masse Filipinization that the
Philippine Bill had effected in 1902.That citizenship (of Lorenzo Pou), if acquired, would thereby
extend to his son, Allan F. Poe, father of respondent FPJ. The 1935 Constitution, during which regime
respondent FPJ has seen first light, confers citizenship to all persons whose fathers are Filipino
citizens regardless of whether such children are legitimate or illegitimate.
(4) But while the totality of the evidence may not establish conclusively that respondent FPJ is a
natural-born citizen of the Philippines, the evidence on hand still would preponderate in his favor
enough to hold that he cannot be held guilty of having made a material misrepresentation in his
certificate of candidacy in violation of Section 78, in relation to Section 74, of the Omnibus Election
Code. Petitioner has utterly failed to substantiate his case before the Court, notwithstanding the ample
opportunity given to the parties to present their position and evidence, and to prove whether or not
there has been material misrepresentation, which, as so ruled in Romualdez-Marcos vs. COMELEC,
[48]
must not only be material, but also deliberate and willful.
WHEREFORE, the Court RESOLVES to DISMISS
1. G. R. No. 161434, entitled "Maria Jeanette C. Tecson and Felix B. Desiderio, Jr.,
Petitioners, versus Commission on Elections, Ronald Allan Kelley Poe (a.k.a. "Fernando Poe, Jr.,) and
Victorino X. Fornier, Respondents," and G. R. No. 161634, entitled "Zoilo Antonio Velez,
Petitioner, versus Ronald Allan Kelley Poe, a.k.a. Fernando Poe, Jr., Respondent," for want of
jurisdiction.
2. G. R. No. 161824, entitled Victorino X. Fornier, Petitioner, versus Hon. Commission on
Elections and Ronald Allan Kelley Poe, also known as Fernando Poe, Jr., for failure to show grave
abuse of discretion on the part of respondent Commission on Elections in dismissing the petition in
SPA No. 04-003.
No Costs.
SO ORDERED.
//////////////////////////////////////////////////////////////////////////////////

Case Digest: Tecson vs. Comelec

G.R. No. 161434 March 3, 2004


MARIA JEANETTE C. TECSON and FELIX B. DESIDERIO, JR. vs.COMELEC, FPJ and
VICTORINO X. FORNIER,

G.R. No. 161634 March 3, 2004


ZOILO ANTONIO VELEZ vs.FPJ

G. R. No. 161824 March 3, 2004


VICTORINO X. FORNIER, vs. HON. COMMISSION ON ELECTIONS and FPJ

Facts:
Petitioners sought for respondent Poes disqualification in the presidential elections for having
allegedly misrepresented material facts in his (Poes) certificate of candidacy by claiming that he is a
natural Filipino citizen despite his parents both being foreigners. Comelec dismissed the petition,
holding that Poe was a Filipino Citizen. Petitioners assail the jurisdiction of the Comelec, contending
that only the Supreme Court may resolve the basic issue on the case under Article VII, Section 4,
paragraph 7, of the 1987 Constitution.

Issue:
Whether or not it is the Supreme Court which had jurisdiction.
Whether or not Comelec committed grave abuse of discretion in holding that Poe was a Filipino
citizen.

Ruling:
1.) The Supreme Court had no jurisdiction on questions regarding qualification of a candidate for the
presidency or vice-presidency before the elections are held.

"Rules of the Presidential Electoral Tribunal" in connection with Section 4, paragraph 7, of the 1987
Constitution, refers to contests relating to the election, returns and qualifications of the "President" or
"Vice-President", of the Philippines which the Supreme Court may take cognizance, and not of
"candidates" for President or Vice-President before the elections.

2.) Comelec committed no grave abuse of discretion in holding Poe as a Filipino Citizen.

The 1935 Constitution on Citizenship, the prevailing fundamental law on respondents birth, provided
that among the citizens of the Philippines are "those whose fathers are citizens of the Philippines."

Tracing respondents paternal lineage, his grandfather Lorenzo, as evidenced by the latters death
certificate was identified as a Filipino Citizen. His citizenship was also drawn from the presumption that
having died in 1954 at the age of 84, Lorenzo would have been born in 1870. In the absence of any
other evidence, Lorenzos place of residence upon his death in 1954 was presumed to be the place of
residence prior his death, such that Lorenzo Pou would have benefited from the "en masse
Filipinization" that the Philippine Bill had effected in 1902. Being so, Lorenzos citizenship would have
extended to his son, Allan---respondents father.

Respondent, having been acknowledged as Allans son to Bessie, though an American citizen, was a
Filipino citizen by virtue of paternal filiation as evidenced by the respondents birth certificate. The
1935 Constitution on citizenship did not make a distinction on the legitimacy or illegitimacy of the child,
thus, the allegation of bigamous marriage and the allegation that respondent was born only before the
assailed marriage had no bearing on respondents citizenship in view of the established paternal
filiation evidenced by the public documents presented.

But while the totality of the evidence may not establish conclusively that respondent FPJ is a natural-
born citizen of the Philippines, the evidence on hand still would preponderate in his favor enough to
hold that he cannot be held guilty of having made a material misrepresentation in his certificate of
candidacy in violation of Section 78, in relation to Section 74 of the Omnibus Election Code.

//////////////////////////////////////////////////////////////////////////////////////////////////////////////////

March 8, 2016

G.R. No. 221697

MARY GRACE NATIVIDAD S. POE-LLAMANZARES, Petitioners,


vs.
COMELEC AND ESTRELLA C. ELAMPARO Respondents.

x-----------------------x

G.R. No. 221698-700

MARY GRACE NATIVIDAD S. POE-LLAMANZARES, Petitioners,


vs.
COMELEC, FRANCISCO S. TATAD, ANTONIO P. CONTRERAS AND AMADO D.
VALDEZ Respondents.

DECISION

PEREZ, J.:

Before the Court are two consolidated petitions under Rule 64 in relation to Rule 65 of the Rules of
Court with extremely urgent application for an ex parte issuance of temporary restraining order/status
quo ante order and/or writ of preliminary injunction assailing the following: (1) 1 December 2015
Resolution of the Commission on Elections (COMELEC) Second Division; (2) 23 December 2015
Resolution of the COMELEC En Banc, in SPA No. 15-001 (DC); (3) 11 December 2015 Resolution of
the COMELEC First Division; and ( 4) 23 December 2015 Resolution of the COMELEC En Banc, in
SPA No. 15-002 (DC), SPA No. 15-007 (DC) and SPA No. 15-139 (DC) for having been issued without
jurisdiction or with grave abuse of discretion amounting to lack or excess of jurisdiction.

The Facts

Mary Grace Natividad S. Poe-Llamanzares (petitioner) was found abandoned as a newborn infant in
the Parish Church of Jaro, Iloilo by a certain Edgardo Militar (Edgardo) on 3 September 1968. Parental
care and custody over petitioner was passed on by Edgardo to his relatives, Emiliano Militar (Emiliano)
and his wife. Three days after, 6 September 1968, Emiliano reported and registered petitioner as a
foundling with the Office of the Civil Registrar of Iloilo City (OCR-Iloilo). In her Foundling Certificate
and Certificate of Live Birth, the petitioner was given the name "Mary Grace Natividad Contreras
Militar." 1
When petitioner was five (5) years old, celebrity spouses Ronald Allan Kelley Poe (a.k.a. Fenando
Poe, Jr.) and Jesusa Sonora Poe (a.k.a. Susan Roces) filed a petition for her adoption with the
Municipal Trial Court (MTC) of San Juan City. On 13 May 1974, the trial court granted their petition
and ordered that petitioner's name be changed from "Mary Grace Natividad Contreras Militar" to "Mary
Grace Natividad Sonora Poe." Although necessary notations were made by OCR-Iloilo on petitioner's
foundling certificate reflecting the court decreed adoption, 2 the petitioner's adoptive mother discovered
only sometime in the second half of 2005 that the lawyer who handled petitioner's adoption failed to
secure from the OCR-Iloilo a new Certificate of Live Birth indicating petitioner's new name and the
name of her adoptive parents. 3 Without delay, petitioner's mother executed an affidavit attesting to the
lawyer's omission which she submitted to the OCR-Iloilo. On 4 May 2006, OCR-Iloilo issued a new
Certificate of Live Birth in the name of Mary Grace Natividad Sonora Poe. 4

Having reached the age of eighteen (18) years in 1986, petitioner registered as a voter with the local
COMELEC Office in San Juan City. On 13 December 1986, she received her COMELEC Voter's
Identification Card for Precinct No. 196 in Greenhills, San Juan, Metro Manila. 5

On 4 April 1988, petitioner applied for and was issued Philippine Passport No. F927287 6 by the
Department of Foreign Affairs (DFA). Subsequently, on 5 April 1993 and 19 May 1998, she renewed
her Philippine passport and respectively secured Philippine Passport Nos. L881511 and DD156616. 7

Initially, the petitioner enrolled and pursued a degree in Development Studies at the University of the
Philippines8but she opted to continue her studies abroad and left for the United States of America
(U.S.) in 1988. Petitioner graduated in 1991 from Boston College in Chestnuts Hill, Massachusetts
where she earned her Bachelor of Arts degree in Political Studies. 9

On 27 July 1991, petitioner married Teodoro Misael Daniel V. Llamanzares (Llamanzares), a citizen of
both the Philippines and the U.S., at Sanctuario de San Jose Parish in San Juan City. 10 Desirous of
being with her husband who was then based in the U.S., the couple flew back to the U.S. two days
after the wedding ceremony or on 29 July 1991. 11

While in the U.S., the petitioner gave birth to her eldest child Brian Daniel (Brian) on 16 April
1992.12 Her two daughters Hanna MacKenzie (Hanna) and Jesusa Anika (Anika) were both born in the
Philippines on 10 July 1998 and 5 June 2004, respectively. 13

14
On 18 October 2001, petitioner became a naturalized American citizen. She obtained U.S. Passport
No. 017037793 on 19 December 2001. 15

On 8 April 2004, the petitioner came back to the Philippines together with Hanna to support her
father's candidacy for President in the May 2004 elections. It was during this time that she gave birth
to her youngest daughter Anika. She returned to the U.S. with her two daughters on 8 July 2004. 16

After a few months, specifically on 13 December 2004, petitioner rushed back to the Philippines upon
learning of her father's deteriorating medical condition. 17 Her father slipped into a coma and eventually
expired. The petitioner stayed in the country until 3 February 2005 to take care of her father's funeral
arrangements as well as to assist in the settlement of his estate. 18

According to the petitioner, the untimely demise of her father was a severe blow to her entire family. In
her earnest desire to be with her grieving mother, the petitioner and her husband decided to move and
reside permanently in the Philippines sometime in the first quarter of 2005. 19 The couple began
preparing for their resettlement including notification of their children's schools that they will be
transferring to Philippine schools for the next semester; 20 coordination with property movers for the
relocation of their household goods, furniture and cars from the U.S. to the Philippines; 21 and inquiry
with Philippine authorities as to the proper procedure to be followed in bringing their pet dog into the
country.22 As early as 2004, the petitioner already quit her job in the U.S. 23

Finally, petitioner came home to the Philippines on 24 May 200524 and without delay, secured a Tax
Identification Number from the Bureau of Internal Revenue. Her three (3) children immediately
followed25 while her husband was forced to stay in the U.S. to complete pending projects as well as to
arrange the sale of their family home there.26

The petitioner and her children briefly stayed at her mother's place until she and her husband
purchased a condominium unit with a parking slot at One Wilson Place Condominium in San Juan City
in the second half of 2005.27 The corresponding Condominium Certificates of Title covering the unit
and parking slot were issued by the Register of Deeds of San Juan City to petitioner and her husband
on 20 February 2006.28 Meanwhile, her children of school age began attending Philippine private
schools.

On 14 February 2006, the petitioner made a quick trip to the U.S. to supervise the disposal of some of
the family's remaining household belongings.29 She travelled back to the Philippines on 11 March
2006.30

In late March 2006, petitioner's husband officially informed the U.S. Postal Service of the family's
change and abandonment of their address in the U.S. 31 The family home was eventually sold on 27
April 2006.32 Petitioner's husband resigned from his job in the U.S. in April 2006, arrived in the country
on 4 May 2006 and started working for a major Philippine company in July 2006. 33

In early 2006, petitioner and her husband acquired a 509-square meter lot in Corinthian Hills, Quezon
City where they built their family home34 and to this day, is where the couple and their children have
been residing.35 A Transfer Certificate of Title covering said property was issued in the couple's name
by the Register of Deeds of Quezon City on 1June 2006.

On 7 July 2006, petitioner took her Oath of Allegiance to the Republic of the Philippines pursuant to
Republic Act (R.A.) No. 9225 or the Citizenship Retention and Re-acquisition Act of 2003. 36 Under the
same Act, she filed with the Bureau of Immigration (BI) a sworn petition to reacquire Philippine
citizenship together with petitions for derivative citizenship on behalf of her three minor children on 10
July 2006.37 As can be gathered from its 18 July 2006 Order, the BI acted favorably on petitioner's
petitions and declared that she is deemed to have reacquired her Philippine citizenship while her
children are considered as citizens of the Philippines.38 Consequently, the BI issued Identification
Certificates (ICs) in petitioner's name and in the names of her three (3) children. 39

Again, petitioner registered as a voter of Barangay Santa Lucia, San Juan City on 31 August
2006.40 She also secured from the DFA a new Philippine Passport bearing the No. XX4731999. 41 This
passport was renewed on 18 March 2014 and she was issued Philippine Passport No. EC0588861 by
the DFA.42

On 6 October 2010, President Benigno S. Aquino III appointed petitioner as Chairperson of the Movie
and Television Review and Classification Board (MTRCB).43 Before assuming her post, petitioner
executed an "Affidavit of Renunciation of Allegiance to the United States of America and Renunciation
of American Citizenship" before a notary public in Pasig City on 20 October 2010, 44 in satisfaction of
the legal requisites stated in Section 5 of R.A. No. 9225. 45 The following day, 21 October 2010
petitioner submitted the said affidavit to the BI46 and took her oath of office as Chairperson of the
MTRCB.47 From then on, petitioner stopped using her American passport. 48

On 12 July 2011, the petitioner executed before the Vice Consul of the U.S. Embassy in Manila an
"Oath/Affirmation of Renunciation of Nationality of the United States." 49 On that day, she accomplished
a sworn questionnaire before the U.S. Vice Consul wherein she stated that she had taken her oath as
MTRCB Chairperson on 21 October 2010 with the intent, among others, of relinquishing her American
citizenship.50 In the same questionnaire, the petitioner stated that she had resided outside of the U.S.,
specifically in the Philippines, from 3 September 1968 to 29 July 1991 and from May 2005 to present. 51

On 9 December 2011, the U.S. Vice Consul issued to petitioner a "Certificate of Loss of Nationality of
the United States" effective 21 October 2010.52

On 2 October 2012, the petitioner filed with the COMELEC her Certificate of Candidacy (COC) for
Senator for the 2013 Elections wherein she answered "6 years and 6 months" to the question "Period
of residence in the Philippines before May 13, 2013." 53 Petitioner obtained the highest number of votes
and was proclaimed Senator on 16 May 2013. 54

55
On 19 December 2013, petitioner obtained Philippine Diplomatic Passport No. DE0004530.

On 15 October 2015, petitioner filed her COC for the Presidency for the May 2016 Elections. 56 In her
COC, the petitioner declared that she is a natural-born citizen and that her residence in the Philippines
up to the day before 9 May 2016 would be ten (10) years and eleven (11) months counted from 24
May 2005.57 The petitioner attached to her COC an "Affidavit Affirming Renunciation of U.S.A.
Citizenship" subscribed and sworn to before a notary public in Quezon City on 14 October 2015. 58

Petitioner's filing of her COC for President in the upcoming elections triggered the filing of several
COMELEC cases against her which were the subject of these consolidated cases.

Origin of Petition for Certiorari in G.R. No. 221697

A day after petitioner filed her COC for President, Estrella Elamparo (Elamparo) filed a petition to deny
due course or cancel said COC which was docketed as SPA No. 15-001 (DC) and raffled to the
COMELEC Second Division.59She is convinced that the COMELEC has jurisdiction over her
petition.60 Essentially, Elamparo's contention is that petitioner committed material misrepresentation
when she stated in her COC that she is a natural-born Filipino citizen and that she is a resident of the
Philippines for at least ten (10) years and eleven (11) months up to the day before the 9 May 2016
Elections.61

On the issue of citizenship, Elamparo argued that petitioner cannot be considered as a natural-born
Filipino on account of the fact that she was a foundling. 62 Elamparo claimed that international law does
not confer natural-born status and Filipino citizenship on foundlings. 63 Following this line of reasoning,
petitioner is not qualified to apply for reacquisition of Filipino citizenship under R.A. No. 9225 for she is
not a natural-born Filipino citizen to begin with. 64 Even assuming arguendo that petitioner was a
natural-born Filipino, she is deemed to have lost that status when she became a naturalized American
citizen.65 According to Elamparo, natural-born citizenship must be continuous from birth. 66

On the matter of petitioner's residency, Elamparo pointed out that petitioner was bound by the sworn
declaration she made in her 2012 COC for Senator wherein she indicated that she had resided in the
country for only six ( 6) years and six ( 6) months as of May 2013 Elections. Elamparo likewise insisted
that assuming arguendo that petitioner is qualified to regain her natural-born status under R.A. No.
9225, she still fell short of the ten-year residency requirement of the Constitution as her residence
could only be counted at the earliest from July 2006, when she reacquired Philippine citizenship under
the said Act. Also on the assumption that petitioner is qualified to reacquire lost Philippine Citizenship,
Elamparo is of the belief that she failed to reestablish her domicile in the Philippines. 67

Petitioner seasonably filed her Answer wherein she countered that:

(1) the COMELEC did not have jurisdiction over Elamparo's petition as it was actually a
petition for quo warranto which could only be filed if Grace Poe wins in the Presidential
elections, and that the Department of Justice (DOJ) has primary jurisdiction to revoke the BI's
July 18, 2006 Order;

(2) the petition failed to state a cause of action because it did not contain allegations which, if
hypothetically admitted, would make false the statement in her COC that she is a natural-born
Filipino citizen nor was there any allegation that there was a willful or deliberate intent to
misrepresent on her part;

(3) she did not make any material misrepresentation in the COC regarding her citizenship and
residency qualifications for:

a. the 1934 Constitutional Convention deliberations show that foundlings were


considered citizens;
b. foundlings are presumed under international law to have been born of citizens of
the place where they are found;

c. she reacquired her natural-born Philippine citizenship under the provisions of R.A.
No. 9225;

d. she executed a sworn renunciation of her American citizenship prior to the filing of
her COC for President in the May 9, 2016 Elections and that the same is in full force
and effect and has not been withdrawn or recanted;

e. the burden was on Elamparo in proving that she did not possess natural-born
status;

f. residence is a matter of evidence and that she reestablished her domicile in the
Philippines as early as May 24, 2005;

g. she could reestablish residence even before she reacquired natural-born


citizenship under R.A. No. 9225;

h. statement regarding the period of residence in her 2012 COC for Senator was an
honest mistake, not binding and should give way to evidence on her true date of
reacquisition of domicile;

i. Elamparo's petition is merely an action to usurp the sovereign right of the Filipino
people to decide a purely political question, that is, should she serve as the country's
next leader.68

After the parties submitted their respective Memoranda, the petition was deemed submitted for
resolution.

On 1 December 2015, the COMELEC Second Division promulgated a Resolution finding that
petitioner's COC, filed for the purpose of running for the President of the Republic of the Philippines in
the 9 May 2016 National and Local Elections, contained material representations which are false.
The fallo of the aforesaid Resolution reads:

WHEREFORE, in view of all the foregoing considerations, the instant Petition to Deny Due Course to
or Cancel Certificate of Candidacy is hereby GRANTED. Accordingly, the Certificate of Candidacy for
President of the Republic of the Philippines in the May 9, 2016 National and Local Elections filed by
respondent Mary Grace Natividad Sonora Poe Llamanzares is hereby CANCELLED.69

Motion for Reconsideration of the 1 December 2015 Resolution was filed by petitioner which the
COMELEC En Banc resolved in its 23 December 2015 Resolution by denying the same. 70

Origin of Petition for Certiorari in G.R. Nos. 221698-700

This case stemmed from three (3) separate petitions filed by Francisco S. Tatad (Tatad), Antonio P.
Contreras (Contreras) and Amado D. Valdez (Valdez) against petitioner before the COMELEC which
were consolidated and raffled to its First Division.

In his petition to disqualify petitioner under Rule 25 of the COMELEC Rules of Procedure, 71 docketed
as SPA No. 15-002 (DC), Tatad alleged that petitioner lacks the requisite residency and citizenship to
qualify her for the Presidency.72

Tatad theorized that since the Philippines adheres to the principle of jus sanguinis, persons of
unknown parentage, particularly foundlings, cannot be considered natural-born Filipino citizens since
blood relationship is determinative of natural-born status. 73 Tatad invoked the rule of statutory
construction that what is not included is excluded. He averred that the fact that foundlings were not
expressly included in the categories of citizens in the 193 5 Constitution is indicative of the framers'
intent to exclude them.74 Therefore, the burden lies on petitioner to prove that she is a natural-born
citizen.75

Neither can petitioner seek refuge under international conventions or treaties to support her claim that
foundlings have a nationality.76 According to Tatad, international conventions and treaties are not self-
executory and that local legislations are necessary in order to give effect to treaty obligations assumed
by the Philippines.77 He also stressed that there is no standard state practice that automatically
confers natural-born status to foundlings. 78

Similar to Elamparo's argument, Tatad claimed that petitioner cannot avail of the option to reacquire
Philippine citizenship under R.A. No. 9225 because it only applies to former natural-born citizens and
petitioner was not as she was a foundling.79

Referring to petitioner's COC for Senator, Tatad concluded that she did not comply with the ten (10)
year residency requirement.80 Tatad opined that petitioner acquired her domicile in Quezon City only
from the time she renounced her American citizenship which was sometime in 2010 or
2011.81 Additionally, Tatad questioned petitioner's lack of intention to abandon her U.S. domicile as
evinced by the fact that her husband stayed thereat and her frequent trips to the U.S. 82

In support of his petition to deny due course or cancel the COC of petitioner, docketed as SPA No. 15-
139 (DC), Valdez alleged that her repatriation under R.A. No. 9225 did not bestow upon her the status
of a natural-born citizen.83 He advanced the view that former natural-born citizens who are repatriated
under the said Act reacquires only their Philippine citizenship and will not revert to their original status
as natural-born citizens.84

He further argued that petitioner's own admission in her COC for Senator that she had only been a
resident of the Philippines for at least six (6) years and six (6) months prior to the 13 May 2013
Elections operates against her. Valdez rejected petitioner's claim that she could have validly
reestablished her domicile in the Philippines prior to her reacquisition of Philippine citizenship. In
effect, his position was that petitioner did not meet the ten (10) year residency requirement for
President.

Unlike the previous COMELEC cases filed against petitioner, Contreras' petition, 85 docketed as SPA
No. 15-007 (DC), limited the attack to the residency issue. He claimed that petitioner's 2015 COC for
President should be cancelled on the ground that she did not possess the ten-year period of residency
required for said candidacy and that she made false entry in her COC when she stated that she is a
legal resident of the Philippines for ten (10) years and eleven (11) months by 9 May 2016. 86 Contreras
contended that the reckoning period for computing petitioner's residency in the Philippines should be
from 18 July 2006, the date when her petition to reacquire Philippine citizenship was approved by the
BI.87 He asserted that petitioner's physical presence in the country before 18 July 2006 could not be
valid evidence of reacquisition of her Philippine domicile since she was then living here as an
American citizen and as such, she was governed by the Philippine immigration laws. 88

In her defense, petitioner raised the following arguments:

First, Tatad's petition should be dismissed outright for failure to state a cause of action. His petition did
not invoke grounds proper for a disqualification case as enumerated under Sections 12 and 68 of the
Omnibus Election Code.89 Instead, Tatad completely relied on the alleged lack of residency and
natural-born status of petitioner which are not among the recognized grounds for the disqualification of
a candidate to an elective office.90

Second, the petitions filed against her are basically petitions for quo warranto as they focus on
establishing her ineligibility for the Presidency.91 A petition for quo warranto falls within the exclusive
jurisdiction of the Presidential Electoral Tribunal (PET) and not the COMELEC. 92
Third, the burden to prove that she is not a natural-born Filipino citizen is on the
respondents.93 Otherwise stated, she has a presumption in her favor that she is a natural-born citizen
of this country.

Fourth, customary international law dictates that foundlings are entitled to a nationality and are
presumed to be citizens of the country where they are found.94 Consequently, the petitioner is
considered as a natural-born citizen of the Philippines. 95

Fifth, she claimed that as a natural-born citizen, she has every right to be repatriated under R.A. No.
9225 or the right to reacquire her natural-born status. 96 Moreover, the official acts of the Philippine
Government enjoy the presumption of regularity, to wit: the issuance of the 18 July 2006 Order of the
BI declaring her as natural-born citizen, her appointment as MTRCB Chair and the issuance of the
decree of adoption of San Juan RTC.97 She believed that all these acts reinforced her position that she
is a natural-born citizen of the Philippines.98

Sixth, she maintained that as early as the first quarter of 2005, she started reestablishing her domicile
of choice in the Philippines as demonstrated by her children's resettlement and schooling in the
country, purchase of a condominium unit in San Juan City and the construction of their family home in
Corinthian Hills.99

Seventh, she insisted that she could legally reestablish her domicile of choice in the Philippines even
before she renounced her American citizenship as long as the three determinants for a change of
domicile are complied with.100 She reasoned out that there was no requirement that renunciation of
foreign citizenship is a prerequisite for the acquisition of a new domicile of choice. 101

Eighth, she reiterated that the period appearing in the residency portion of her COC for Senator was a
mistake made in good faith.102

In a Resolution103 promulgated on 11 December 2015, the COMELEC First Division ruled that
petitioner is not a natural-born citizen, that she failed to complete the ten (10) year residency
requirement, and that she committed material misrepresentation in her COC when she declared
therein that she has been a resident of the Philippines for a period of ten (10) years and eleven (11)
months as of the day of the elections on 9 May 2016. The COMELEC First Division concluded that she
is not qualified for the elective position of President of the Republic of the Philippines. The dispositive
portion of said Resolution reads:

WHEREFORE, premises considered, the Commission RESOLVED, as it hereby RESOLVES,


to GRANT the Petitions and cancel the Certificate of Candidacy of MARY GRACE NATIVIDAD
SONORA POE-LLAMANZARES for the elective position of President of the Republic of the
Philippines in connection with the 9 May 2016 Synchronized Local and National Elections.

Petitioner filed a motion for reconsideration seeking a reversal of the COMELEC First Division's
Resolution. On 23 December 2015, the COMELEC En Banc issued a Resolution denying petitioner's
motion for reconsideration.

Alarmed by the adverse rulings of the COMELEC, petitioner instituted the present petitions
for certiorari with urgent prayer for the issuance of an ex parte temporary restraining order/status quo
ante order and/or writ of preliminary injunction. On 28 December 2015, temporary restraining orders
were issued by the Court enjoining the COMELEC and its representatives from implementing the
assailed COMELEC Resolutions until further orders from the Court. The Court also ordered the
consolidation of the two petitions filed by petitioner in its Resolution of 12 January 2016. Thereafter,
oral arguments were held in these cases.

The Court GRANTS the petition of Mary Grace Natividad S. Poe-Llamanzares and to ANNUL and SET
ASIDE the:
1. Resolution dated 1 December 2015 rendered through its Second Division, in SPA No. 15-
001 (DC), entitled Estrella C. Elamparo, petitioner, vs. Mary Grace Natividad Sonora Poe-
Llamanzares.

2. Resolution dated 11 December 2015, rendered through its First Division, in the consolidated
cases SPA No. 15-002 (DC) entitled Francisco S. Tatad, petitioner, vs. Mary Grace Natividad
Sonora Poe-Llamanzares, respondent; SPA No. 15-007 (DC) entitled Antonio P. Contreras,
petitioner, vs. Mary Grace Natividad Sonora Poe-Llamanzares, respondent; and SPA No. 15-
139 (DC) entitled Amado D. Valdez, petitioner, v. Mary Grace Natividad Sonora Poe-
Llamanzares, respondent.

3. Resolution dated 23 December 2015 of the Commission En Banc, upholding the 1


December 2015 Resolution of the Second Division.

4. Resolution dated 23 December 2015 of the Commission En Banc, upholding the 11


December 2015 Resolution of the First Division.

The procedure and the conclusions from which the questioned Resolutions emanated are tainted with
grave abuse of discretion amounting to lack of jurisdiction. The petitioner is a QUALIFIED
CANDIDATE for President in the 9 May 2016 National Elections.

The issue before the COMELEC is whether or not the COC of petitioner should be denied due course
or cancelled "on the exclusive ground" that she made in the certificate a false material representation.
The exclusivity of the ground should hedge in the discretion of the COMELEC and restrain it from
going into the issue of the qualifications of the candidate for the position, if, as in this case, such issue
is yet undecided or undetermined by the proper authority. The COMELEC cannot itself, in the same
cancellation case, decide the qualification or lack thereof of the candidate.

We rely, first of all, on the Constitution of our Republic, particularly its provisions in Article IX, C,
Section 2:

Section 2. The Commission on Elections shall exercise the following powers and functions:

(1) Enforce and administer all laws and regulations relative to the conduct of an
election, plebiscite, initiative, referendum, and recall.

(2) Exercise exclusive original jurisdiction over all contests relating to the elections,
returns, and qualifications of all elective regional, provincial, and city officials, and
appellate jurisdiction over all contests involving elective municipal officials decided by
trial courts of general jurisdiction, or involving elective barangay officials decided by
trial courts of limited jurisdiction.

Decisions, final orders, or rulings of the Commission on election contests involving


elective municipal and barangay offices shall be final, executory, and not appealable.

(3) Decide, except those involving the right to vote, all questions affecting elections,
including determination of the number and location of polling places, appointment of
election officials and inspectors, and registration of voters.

(4) Deputize, with the concurrence of the President, law enforcement agencies and
instrumentalities of the Government, including the Armed Forces of the Philippines, for
the exclusive purpose of ensuring free, orderly, honest, peaceful, and credible
elections.

(5) Register, after sufficient publication, political parties, organizations, or coalitions


which, in addition to other requirements, must present their platform or program of
government; and accredit citizens' arms of the Commission on Elections. Religious
denominations and sects shall not be registered. Those which seek to achieve their
goals through violence or unlawful means, or refuse to uphold and adhere to this
Constitution, or which are supported by any foreign government shall likewise be
refused registration.

Financial contributions from foreign governments and their agencies to political


parties, organizations, coalitions, or candidates related to elections constitute
interference in national affairs, and, when accepted, shall be an additional ground for
the cancellation of their registration with the Commission, in addition to other penalties
that may be prescribed by law.

(6) File, upon a verified complaint, or on its own initiative, petitions in court for
inclusion or exclusion of voters; investigate and, where appropriate, prosecute cases
of violations of election laws, including acts or omissions constituting election frauds,
offenses, and malpractices.

(7) Recommend to the Congress effective measures to minimize election spending,


including limitation of places where propaganda materials shall be posted, and to
prevent and penalize all forms of election frauds, offenses, malpractices, and
nuisance candidacies.

(8) Recommend to the President the removal of any officer or employee it has
deputized, or the imposition of any other disciplinary action, for violation or disregard
of, or disobedience to its directive, order, or decision.

(9) Submit to the President and the Congress a comprehensive report on the conduct
of each election, plebiscite, initiative, referendum, or recall.

Not any one of the enumerated powers approximate the exactitude of the provisions of Article VI,
Section 17 of the same basic law stating that:

The Senate and the House of Representatives shall each have an Electoral Tribunal which
shall be the sole judge of all contests relating to the election, returns, and qualifications of their
respective Members. Each Electoral Tribunal shall be composed of nine Members, three of
whom shall be Justices of the Supreme Court to be designated by the Chief Justice, and the
remaining six shall be Members of the Senate or the House of Representatives, as the case
may be, who shall be chosen on the basis of proportional representation from the political
parties and the parties or organizations registered under the party-list system represented
therein. The senior Justice in the Electoral Tribunal shall be its Chairman.

or of the last paragraph of Article VII, Section 4 which provides that:

The Supreme Court, sitting en banc, shall be the sole judge of all contests relating to the
election, returns, and qualifications of the President or Vice-President, and may promulgate its
rules for the purpose.

The tribunals which have jurisdiction over the question of the qualifications of the President, the Vice-
President, Senators and the Members of the House of Representatives was made clear by the
Constitution. There is no such provision for candidates for these positions.

Can the COMELEC be such judge?

The opinion of Justice Vicente V. Mendoza in Romualdez-Marcos v. Commission on


Elections,104 which was affirmatively cited in the En Banc decision in Fermin v. COMELEC105 is our
guide. The citation in Fermin reads:

Apparently realizing the lack of an authorized proceeding for declaring the ineligibility of candidates,
the COMELEC amended its rules on February 15, 1993 so as to provide in Rule 25 1, the following:
Grounds for disqualification. - Any candidate who does not possess all the
qualifications of a candidate as provided for by the Constitution or by existing law or
who commits any act declared by law to be grounds for disqualification may be
disqualified from continuing as a candidate.

The lack of provision for declaring the ineligibility of candidates, however, cannot be supplied by a
mere rule. Such an act is equivalent to the creation of a cause of action which is a substantive matter
which the COMELEC, in the exercise of its rule-making power under Art. IX, A, 6 of the Constitution,
cannot do it. It is noteworthy that the Constitution withholds from the COMELEC even the power to
decide cases involving the right to vote, which essentially involves an inquiry into qualifications based
on age, residence and citizenship of voters. [Art. IX, C, 2(3)]

The assimilation in Rule 25 of the COMELEC rules of grounds for ineligibility into grounds for
disqualification is contrary to the evident intention of the law. For not only in their grounds but also in
their consequences are proceedings for "disqualification" different from those for a declaration of
"ineligibility." "Disqualification" proceedings, as already stated, are based on grounds specified in 12
and 68 of the Omnibus Election Code and in 40 of the Local Government Code and are for the
purpose of barring an individual from becoming a candidate or from continuing as a candidate for
public office. In a word, their purpose is to eliminate a candidate from the race either from the start or
during its progress. "Ineligibility," on the other hand, refers to the lack of the qualifications prescribed in
the Constitution or the statutes for holding public office and the purpose of the proceedings for
declaration of ineligibility is to remove the incumbent from office.

Consequently, that an individual possesses the qualifications for a public office does not imply that he
is not disqualified from becoming a candidate or continuing as a candidate for a public office and vice
versa. We have this sort of dichotomy in our Naturalization Law. (C.A. No. 473) That an alien has the
qualifications prescribed in 2 of the Law does not imply that he does not suffer from any of [the]
disqualifications provided in 4.

Before we get derailed by the distinction as to grounds and the consequences of the respective
proceedings, the importance of the opinion is in its statement that "the lack of provision for declaring
the ineligibility of candidates, however, cannot be supplied by a mere rule". Justice Mendoza lectured
in Romualdez-Marcos that:

Three reasons may be cited to explain the absence of an authorized proceeding for
determining before election the qualifications of a candidate.

First is the fact that unless a candidate wins and is proclaimed elected, there is no necessity for
determining his eligibility for the office. In contrast, whether an individual should be disqualified as a
candidate for acts constituting election offenses (e.g., vote buying, over spending, commission of
prohibited acts) is a prejudicial question which should be determined lest he wins because of the very
acts for which his disqualification is being sought. That is why it is provided that if the grounds for
disqualification are established, a candidate will not be voted for; if he has been voted for, the votes in
his favor will not be counted; and if for some reason he has been voted for and he has won, either he
will not be proclaimed or his proclamation will be set aside.

Second is the fact that the determination of a candidates' eligibility, e.g., his citizenship or, as in this
case, his domicile, may take a long time to make, extending beyond the beginning of the term of the
office. This is amply demonstrated in the companion case (G.R. No. 120265, Agapito A. Aquino v.
COMELEC) where the determination of Aquino's residence was still pending in the COMELEC even
after the elections of May 8, 1995. This is contrary to the summary character proceedings relating to
certificates of candidacy. That is why the law makes the receipt of certificates of candidacy a
ministerial duty of the COMELEC and its officers. The law is satisfied if candidates state in their
certificates of candidacy that they are eligible for the position which they seek to fill, leaving the
determination of their qualifications to be made after the election and only in the event they are
elected. Only in cases involving charges of false representations made in certificates of candidacy is
the COMELEC given jurisdiction.
Third is the policy underlying the prohibition against pre-proclamation cases in elections for President,
Vice President, Senators and members of the House of Representatives. (R.A. No. 7166, 15) The
purpose is to preserve the prerogatives of the House of Representatives Electoral Tribunal and the
other Tribunals as "sole judges" under the Constitution of the election, returns and qualifications of
members of Congress of the President and Vice President, as the case may be. 106

To be sure, the authoritativeness of the Romualdez pronouncements as reiterated in Fermin, led to the
amendment through COMELEC Resolution No. 9523, on 25 September 2012 of its Rule 25. This, the
15 February1993 version of Rule 25, which states that:

Grounds for disqualification. -Any candidate who does not possess all the qualifications of a candidate
as provided for by the Constitution or by existing law or who commits any act declared by law to be
grounds for disqualification may be disqualified from continuing as a candidate. 107

was in the 2012 rendition, drastically changed to:

Grounds. - Any candidate who, in action or protest in which he is a party, is declared by final decision
of a competent court, guilty of, or found by the Commission to be suffering from any disqualification
provided by law or the Constitution.

A Petition to Disqualify a Candidate invoking grounds for a Petition to Deny to or Cancel a Certificate
of Candidacy or Petition to Declare a Candidate as a Nuisance Candidate, or a combination thereof,
shall be summarily dismissed.

Clearly, the amendment done in 2012 is an acceptance of the reality of absence of an authorized
proceeding for determining before election the qualifications of candidate. Such that, as presently
required, to disqualify a candidate there must be a declaration by a final judgment of a competent
court that the candidate sought to be disqualified "is guilty of or found by the Commission to be
suffering from any disqualification provided by law or the Constitution."

Insofar as the qualification of a candidate is concerned, Rule 25 and Rule 23 are flipsides of one to the
other. Both do not allow, are not authorizations, are not vestment of jurisdiction, for the COMELEC to
determine the qualification of a candidate. The facts of qualification must beforehand be established in
a prior proceeding before an authority properly vested with jurisdiction. The prior determination of
qualification may be by statute, by executive order or by a judgment of a competent court or tribunal.

If a candidate cannot be disqualified without a prior finding that he or she is suffering from a
disqualification "provided by law or the Constitution," neither can the certificate of candidacy be
cancelled or denied due course on grounds of false representations regarding his or her qualifications,
without a prior authoritative finding that he or she is not qualified, such prior authority being the
necessary measure by which the falsity of the representation can be found. The only exception that
can be conceded are self-evident facts of unquestioned or unquestionable veracity and judicial
confessions. Such are, anyway, bases equivalent to prior decisions against which the falsity of
representation can be determined.

The need for a predicate finding or final pronouncement in a proceeding under Rule 23 that deals with,
as in this case, alleged false representations regarding the candidate's citizenship and residence,
forced the COMELEC to rule essentially that since foundlings108 are not mentioned in the enumeration
of citizens under the 1935 Constitution,109 they then cannot be citizens. As the COMELEC stated in
oral arguments, when petitioner admitted that she is a foundling, she said it all. This borders on
bigotry. Oddly, in an effort at tolerance, the COMELEC, after saying that it cannot rule that herein
petitioner possesses blood relationship with a Filipino citizen when "it is certain that such relationship
is indemonstrable," proceeded to say that "she now has the burden to present evidence to prove her
natural filiation with a Filipino parent."

The fact is that petitioner's blood relationship with a Filipino citizen is DEMONSTRABLE.
At the outset, it must be noted that presumptions regarding paternity is neither unknown nor
unaccepted in Philippine Law. The Family Code of the Philippines has a whole chapter on Paternity
and Filiation.110 That said, there is more than sufficient evider1ce that petitioner has Filipino parents
and is therefore a natural-born Filipino. Parenthetically, the burden of proof was on private
respondents to show that petitioner is not a Filipino citizen. The private respondents should have
shown that both of petitioner's parents were aliens. Her admission that she is a foundling did not shift
the burden to her because such status did not exclude the possibility that her parents were Filipinos,
especially as in this case where there is a high probability, if not certainty, that her parents are
Filipinos.

The factual issue is not who the parents of petitioner are, as their identities are unknown, but whether
such parents are Filipinos. Under Section 4, Rule 128:

Sect. 4. Relevancy, collateral matters - Evidence must have such a relation to the fact in issue as to
induce belief in its existence or no-existence. Evidence on collateral matters shall not be allowed,
except when it tends in any reasonable degree to establish the probability of improbability of the fact in
issue.

The Solicitor General offered official statistics from the Philippine Statistics Authority (PSA) 111 that from
1965 to 1975, the total number of foreigners born in the Philippines was 15,986 while the total number
of Filipinos born in the country was 10,558,278. The statistical probability that any child born in the
Philippines in that decade is natural-born Filipino was 99.83%. For her part, petitioner presented
census statistics for Iloilo Province for 1960 and 1970, also from the PSA. In 1960, there were 962,532
Filipinos and 4,734 foreigners in the province; 99.62% of the population were Filipinos. In 1970, the
figures were 1,162,669 Filipinos and 5,304 foreigners, or 99.55%. Also presented were figures for the
child producing ages (15-49). In 1960, there were 230,528 female Filipinos as against 730 female
foreigners or 99.68%. In the same year, there were 210,349 Filipino males and 886 male aliens,
or 99.58%. In 1970, there were 270,299 Filipino females versus 1, 190 female aliens, or 99.56%. That
same year, there were 245,740 Filipino males as against only 1,165 male aliens
or 99.53%. COMELEC did not dispute these figures. Notably, Commissioner Arthur Lim admitted,
during the oral arguments, that at the time petitioner was found in 1968, the majority of the population
in Iloilo was Filipino.112

Other circumstantial evidence of the nationality of petitioner's parents are the fact that she was
abandoned as an infant in a Roman Catholic Church in Iloilo City.1wphi1 She also has typical Filipino
features: height, flat nasal bridge, straight black hair, almond shaped eyes and an oval face.

There is a disputable presumption that things have happened according to the ordinary course of
nature and the ordinary habits of life.113 All of the foregoing evidence, that a person with typical Filipino
features is abandoned in Catholic Church in a municipality where the population of the Philippines is
overwhelmingly Filipinos such that there would be more than a 99% chance that a child born in the
province would be a Filipino, would indicate more than ample probability if not statistical certainty, that
petitioner's parents are Filipinos. That probability and the evidence on which it is based are admissible
under Rule 128, Section 4 of the Revised Rules on Evidence.

To assume otherwise is to accept the absurd, if not the virtually impossible, as the norm. In the words
of the Solicitor General:

Second. It is contrary to common sense because foreigners do not come to the Philippines so they
can get pregnant and leave their newborn babies behind. We do not face a situation where the
probability is such that every foundling would have a 50% chance of being a Filipino and a 50%
chance of being a foreigner. We need to frame our questions properly. What are the chances that the
parents of anyone born in the Philippines would be foreigners? Almost zero. What are the chances
that the parents of anyone born in the Philippines would be Filipinos? 99.9%.

According to the Philippine Statistics Authority, from 2010 to 2014, on a yearly average, there were
1,766,046 children born in the Philippines to Filipino parents, as opposed to 1,301 children in the
Philippines of foreign parents. Thus, for that sample period, the ratio of non-Filipino children to natural
born Filipino children is 1:1357. This means that the statistical probability that any child born in the
Philippines would be a natural born Filipino is 99.93%.

From 1965 to 1975, the total number of foreigners born in the Philippines is 15,986 while the total
number of Filipinos born in the Philippines is 15,558,278. For this period, the ratio of non-Filipino
children is 1:661. This means that the statistical probability that any child born in the Philippines on
that decade would be a natural born Filipino is 99.83%.

We can invite statisticians and social anthropologists to crunch the numbers for us, but I am confident
that the statistical probability that a child born in the Philippines would be a natural born Filipino will not
be affected by whether or not the parents are known. If at all, the likelihood that a foundling would
have a Filipino parent might even be higher than 99.9%. Filipinos abandon their children out of poverty
or perhaps, shame. We do not imagine foreigners abandoning their children here in the Philippines
thinking those infants would have better economic opportunities or believing that this country is a
tropical paradise suitable for raising abandoned children. I certainly doubt whether a foreign couple
has ever considered their child excess baggage that is best left behind.

To deny full Filipino citizenship to all foundlings and render them stateless just because there may be a
theoretical chance that one among the thousands of these foundlings might be the child of not just
one, but two, foreigners is downright discriminatory, irrational, and unjust. It just doesn't make any
sense. Given the statistical certainty - 99.9% - that any child born in the Philippines would be a natural
born citizen, a decision denying foundlings such status is effectively a denial of their birthright. There is
no reason why this Honorable Court should use an improbable hypothetical to sacrifice the
fundamental political rights of an entire class of human beings. Your Honor, constitutional interpretation
and the use of common sense are not separate disciplines.

As a matter of law, foundlings are as a class, natural-born citizens. While the 1935 Constitution's
enumeration is silent as to foundlings, there is no restrictive language which would definitely exclude
foundlings either. Because of silence and ambiguity in the enumeration with respect to foundlings,
there is a need to examine the intent of the framers. In Nitafan v. Commissioner of Internal
Revenue,114 this Court held that:

The ascertainment of that intent is but in keeping with the fundamental principle of
constitutional construction that the intent of the framers of the organic law and of the people
adopting it should be given effect. The primary task in constitutional construction is to
ascertain and thereafter assure the realization of the purpose of the framers and of the people
in the adoption of the Constitution. It may also be safely assumed that the people in ratifying
the Constitution were guided mainly by the explanation offered by the framers. 115

As pointed out by petitioner as well as the Solicitor General, the deliberations of the 1934
Constitutional Convention show that the framers intended foundlings to be covered by the
enumeration. The following exchange is recorded:

Sr. Rafols: For an amendment. I propose that after subsection 2, the following is inserted: "The natural
children of a foreign father and a Filipino mother not recognized by the father.

xxxx

President:
[We] would like to request a clarification from the proponent of the amendment. The gentleman refers
to natural children or to any kind of illegitimate children?

Sr. Rafols:
To all kinds of illegitimate children. It also includes natural children of unknown parentage, natural or
illegitimate children of unknown parents.

Sr. Montinola:
For clarification. The gentleman said "of unknown parents." Current codes consider them Filipino, that
is, I refer to the Spanish Code wherein all children of unknown parentage born in Spanish territory are
considered Spaniards, because the presumption is that a child of unknown parentage is the son of a
Spaniard. This may be applied in the Philippines in that a child of unknown parentage born in the
Philippines is deemed to be Filipino, and there is no need ...

Sr. Rafols:
There is a need, because we are relating the conditions that are [required] to be Filipino.

Sr. Montinola:
But that is the interpretation of the law, therefore, there is no [more] need for amendment.

Sr. Rafols:
The amendment should read thus:
"Natural or illegitimate of a foreign father and a Filipino mother recognized by one, or the children of
unknown parentage."

Sr. Briones:
The amendment [should] mean children born in the Philippines of unknown parentage.

Sr. Rafols:
The son of a Filipina to a Foreigner, although this [person] does not recognize the child, is not
unknown.

President:
Does the gentleman accept the amendment or not?

Sr. Rafols:
I do not accept the amendment because the amendment would exclude the children of a Filipina with
a foreigner who does not recognize the child. Their parentage is not unknown and I think those of
overseas Filipino mother and father [whom the latter] does not recognize, should also be considered
as Filipinos.

President:
The question in order is the amendment to the amendment from the Gentleman from Cebu, Mr.
Briones.

Sr. Busion:
Mr. President, don't you think it would be better to leave this matter in the hands of the Legislature?

Sr. Roxas:
Mr. President, my humble opinion is that these cases are few and far in between, that the constitution
need [not] refer to them. By international law the principle that children or people born in a country of
unknown parents are citizens in this nation is recognized, and it is not necessary to include a provision
on the subject exhaustively.116

Though the Rafols amendment was not carried out, it was not because there was any objection to the
notion that persons of "unknown parentage" are not citizens but only because their number was not
enough to merit specific mention. Such was the account, 117 cited by petitioner, of delegate and
constitution law author Jose Aruego who said:

During the debates on this provision, Delegate Rafols presented an amendment to include as
Filipino citizens the illegitimate children with a foreign father of a mother who was a citizen of
the Philippines, and also foundlings; but this amendment was defeated primarily because the
Convention believed that the cases, being too few to warrant the inclusion of a provision in the
Constitution to apply to them, should be governed by statutory legislation. Moreover, it was
believed that the rules of international law were already clear to the effect that illegitimate
children followed the citizenship of the mother, and that foundlings followed the nationality of
the place where they were found, thereby making unnecessary the inclusion in the
Constitution of the proposed amendment.

This explanation was likewise the position of the Solicitor General during the 16 February 2016 Oral
Arguments:

We all know that the Rafols proposal was rejected. But note that what was declined was the proposal
for a textual and explicit recognition of foundlings as Filipinos. And so, the way to explain the
constitutional silence is by saying that it was the view of Montinola and Roxas which prevailed that
there is no more need to expressly declare foundlings as Filipinos.

Obviously, it doesn't matter whether Montinola's or Roxas' views were legally correct. Framers of a
constitution can constitutionalize rules based on assumptions that are imperfect or even wrong. They
can even overturn existing rules. This is basic. What matters here is that Montinola and Roxas were
able to convince their colleagues in the convention that there is no more need to expressly declare
foundlings as Filipinos because they are already impliedly so recognized.

In other words, the constitutional silence is fully explained in terms of linguistic efficiency and the
avoidance of redundancy. The policy is clear: it is to recognize foundlings, as a class, as Filipinos
under Art. IV, Section 1 (3) of the 1935 Constitution. This inclusive policy is carried over into the 1973
and 1987 Constitution. It is appropriate to invoke a famous scholar as he was paraphrased by Chief
Justice Fernando: the constitution is not silently silent, it is silently vocal. 118

The Solicitor General makes the further point that the framers "worked to create a just and humane
society," that "they were reasonable patriots and that it would be unfair to impute upon them a
discriminatory intent against foundlings." He exhorts that, given the grave implications of the argument
that foundlings are not natural-born Filipinos, the Court must search the records of the 1935, 1973 and
1987 Constitutions "for an express intention to deny foundlings the status of Filipinos. The burden is
on those who wish to use the constitution to discriminate against foundlings to show that the
constitution really intended to take this path to the dark side and inflict this across the board
marginalization."

We find no such intent or language permitting discrimination against foundlings. On the contrary, all
three Constitutions guarantee the basic right to equal protection of the laws. All exhort the State to
render social justice. Of special consideration are several provisions in the present charter: Article II,
Section 11 which provides that the "State values the dignity of every human person and guarantees
full respect for human rights," Article XIII, Section 1 which mandates Congress to "give highest priority
to the enactment of measures that protect and enhance the right of all the people to human dignity,
reduce social, economic, and political inequalities x x x" and Article XV, Section 3 which requires the
State to defend the "right of children to assistance, including proper care and nutrition, and special
protection from all forms of neglect, abuse, cruelty, exploitation, and other conditions prejudicial to their
development." Certainly, these provisions contradict an intent to discriminate against foundlings on
account of their unfortunate status.

Domestic laws on adoption also support the principle that foundlings are Filipinos. These laws do not
provide that adoption confers citizenship upon the adoptee. Rather, the adoptee must be a Filipino in
the first place to be adopted. The most basic of such laws is Article 15 of the Civil Code which provides
that "[l]aws relating to family rights, duties, status, conditions, legal capacity of persons are binding on
citizens of the Philippines even though living abroad." Adoption deals with status, and a Philippine
adoption court will have jurisdiction only if the adoptee is a Filipino. In Ellis and Ellis v. Republic,119 a
child left by an unidentified mother was sought to be adopted by aliens. This Court said:

In this connection, it should be noted that this is a proceedings in rem, which no court may entertain
unless it has jurisdiction, not only over the subject matter of the case and over the parties, but also
over the res, which is the personal status of Baby Rose as well as that of petitioners herein. Our Civil
Code (Art. 15) adheres to the theory that jurisdiction over the status of a natural person is determined
by the latter's nationality. Pursuant to this theory, we have jurisdiction over the status of Baby Rose,
she being a citizen of the Philippines, but not over the status of the petitioners, who are
foreigners.120 (Underlining supplied)
Recent legislation is more direct. R.A. No. 8043 entitled "An Act Establishing the Rules to Govern the
Inter-Country Adoption of Filipino Children and For Other Purposes" (otherwise known as the "Inter-
Country Adoption Act of 1995"), R.A. No. 8552, entitled "An Act Establishing the Rules and Policies on
the Adoption of Filipino Children and For Other Purposes" (otherwise known as the Domestic Adoption
Act of 1998) and this Court's A.M. No. 02-6-02-SC or the "Rule on Adoption," all expressly refer to
"Filipino children" and include foundlings as among Filipino children who may be adopted.

It has been argued that the process to determine that the child is a foundling leading to the issuance of
a foundling certificate under these laws and the issuance of said certificate are acts to acquire or
perfect Philippine citizenship which make the foundling a naturalized Filipino at best. This is
erroneous. Under Article IV, Section 2 "Natural-born citizens are those who are citizens of the
Philippines from birth without having to perform any act to acquire or perfect their Philippine
citizenship." In the first place, "having to perform an act" means that the act must be personally done
by the citizen. In this instance, the determination of foundling status is done not by the child but by the
authorities.121 Secondly, the object of the process is the determination of the whereabouts of the
parents, not the citizenship of the child. Lastly, the process is certainly not analogous to naturalization
proceedings to acquire Philippine citizenship, or the election of such citizenship by one born of an
alien father and a Filipino mother under the 1935 Constitution, which is an act to perfect it.

In this instance, such issue is moot because there is no dispute that petitioner is a foundling, as
evidenced by a Foundling Certificate issued in her favor.122 The Decree of Adoption issued on 13 May
1974, which approved petitioner's adoption by Jesusa Sonora Poe and Ronald Allan Kelley Poe,
expressly refers to Emiliano and his wife, Rosario Militar, as her "foundling parents," hence effectively
affirming petitioner's status as a foundling.123

Foundlings are likewise citizens under international law. Under the 1987 Constitution, an international
law can become part of the sphere of domestic law either by transformation or incorporation. The
transformation method requires that an international law be transformed into a domestic law through a
constitutional mechanism such as local legislation.124 On the other hand, generally accepted principles
of international law, by virtue of the incorporation clause of the Constitution, form part of the laws of the
land even if they do not derive from treaty obligations. Generally accepted principles of international
law include international custom as evidence of a general practice accepted as law, and general
principles of law recognized by civilized nations.125 International customary rules are accepted as
binding as a result from the combination of two elements: the established, widespread, and consistent
practice on the part of States; and a psychological element known as the opinionjuris sive
necessitates (opinion as to law or necessity). Implicit in the latter element is a belief that the practice in
question is rendered obligatory by the existence of a rule of law requiring it. 126 "General principles of
law recognized by civilized nations" are principles "established by a process of reasoning" or judicial
logic, based on principles which are "basic to legal systems generally," 127 such as "general principles
of equity, i.e., the general principles of fairness and justice," and the "general principle against
discrimination" which is embodied in the "Universal Declaration of Human Rights, the International
Covenant on Economic, Social and Cultural Rights, the International Convention on the Elimination of
All Forms of Racial Discrimination, the Convention Against Discrimination in Education, the
Convention (No. 111) Concerning Discrimination in Respect of Employment and Occupation." 128 These
are the same core principles which underlie the Philippine Constitution itself, as embodied in the due
process and equal protection clauses of the Bill of Rights. 129

Universal Declaration of Human Rights ("UDHR") has been interpreted by this Court as part of the
generally accepted principles of international law and binding on the State. 130 Article 15 thereof states:

1. Everyone has the right to a nationality.

2. No one shall be arbitrarily deprived of his nationality nor denied the right to change his
nationality.

The Philippines has also ratified the UN Convention on the Rights of the Child (UNCRC). Article 7 of
the UNCRC imposes the following obligations on our country:

Article 7
1. The child shall be registered immediately after birth and shall have the right from birth to a name,
the right to acquire a nationality and as far as possible, the right to know and be cared for by his or her
parents.

2. States Parties shall ensure the implementation of these rights in accordance with their national law
and their obligations under the relevant international instruments in this field, in particular where the
child would otherwise be stateless.

In 1986, the country also ratified the 1966 International Covenant on Civil and Political Rights
(ICCPR). Article 24 thereof provide for the right of every child "to acquire a nationality:"

Article 24

1. Every child shall have, without any discrimination as to race, colour, sex, language, religion, national
or social origin, property or birth, the right, to such measures of protection as are required by his status
as a minor, on the part of his family, society and the State.

2. Every child shall be registered immediately after birth and shall have a name.

3. Every child has the right to acquire a nationality.

The common thread of the UDHR, UNCRC and ICCPR is to obligate the Philippines to grant
nationality from birth and ensure that no child is stateless. This grant of nationality must be at the time
of birth, and it cannot be accomplished by the application of our present naturalization laws,
Commonwealth Act No. 473, as amended, and R.A. No. 9139, both of which require the applicant to
be at least eighteen (18) years old.

The principles found in two conventions, while yet unratified by the Philippines, are generally accepted
principles of international law. The first is Article 14 of the 1930 Hague Convention on Certain
Questions Relating to the Conflict of Nationality Laws under which a foundling is presumed to have the
"nationality of the country of birth," to wit:

Article 14

A child whose parents are both unknown shall have the nationality of the country of birth. If the child's
parentage is established, its nationality shall be determined by the rules applicable in cases where the
parentage is known.

A foundling is, until the contrary is proved, presumed to have been born on the territory of the State in
which it was found. (Underlining supplied)

The second is the principle that a foundling is presumed born of citizens of the country where he is
found, contained in Article 2 of the 1961 United Nations Convention on the Reduction of
Statelessness:

Article 2

A foundling found in the territory of a Contracting State shall, in the absence of proof to the contrary,
be considered to have been born within the territory of parents possessing the nationality of that State.

That the Philippines is not a party to the 1930 Hague Convention nor to the 1961 Convention on the
Reduction of Statelessness does not mean that their principles are not binding. While the Philippines
is not a party to the 1930 Hague Convention, it is a signatory to the Universal Declaration on Human
Rights, Article 15(1) ofwhich131effectively affirms Article 14 of the 1930 Hague Convention. Article 2 of
the 1961 "United Nations Convention on the Reduction of Statelessness" merely "gives effect" to
Article 15(1) of the UDHR.132 In Razon v. Tagitis, 133 this Court noted that the Philippines had not
signed or ratified the "International Convention for the Protection of All Persons from Enforced
Disappearance." Yet, we ruled that the proscription against enforced disappearances in the said
convention was nonetheless binding as a "generally accepted principle of international law." Razon v.
Tagitis is likewise notable for declaring the ban as a generally accepted principle of international law
although the convention had been ratified by only sixteen states and had not even come into force and
which needed the ratification of a minimum of twenty states. Additionally, as petitioner points out, the
Court was content with the practice of international and regional state organs, regional state practice in
Latin America, and State Practice in the United States.

Another case where the number of ratifying countries was not determinative is Mijares v.
Ranada, 134 where only four countries had "either ratified or acceded to" 135 the 1966 "Convention on
the Recognition and Enforcement of Foreign Judgments in Civil and Commercial Matters" when the
case was decided in 2005. The Court also pointed out that that nine member countries of the
European Common Market had acceded to the Judgments Convention. The Court also cited U.S. laws
and jurisprudence on recognition of foreign judgments. In all, only the practices of fourteen countries
were considered and yet, there was pronouncement that recognition of foreign judgments was
widespread practice.

Our approach in Razon and Mijares effectively takes into account the fact that "generally accepted
principles of international law" are based not only on international custom, but also on "general
principles of law recognized by civilized nations," as the phrase is understood in Article 38.1 paragraph
(c) of the ICJ Statute. Justice, fairness, equity and the policy against discrimination, which are
fundamental principles underlying the Bill of Rights and which are "basic to legal systems
generally,"136 support the notion that the right against enforced disappearances and the recognition of
foreign judgments, were correctly considered as "generally accepted principles of international law"
under the incorporation clause.

Petitioner's evidence137 shows that at least sixty countries in Asia, North and South America, and
Europe have passed legislation recognizing foundlings as its citizen. Forty-two (42) of those countries
follow the jus sanguinis regime. Of the sixty, only thirty-three (33) are parties to the 1961 Convention
on Statelessness; twenty-six (26) are not signatories to the Convention. Also, the Chief Justice, at the
2 February 2016 Oral Arguments pointed out that in 166 out of 189 countries surveyed (or 87.83%),
foundlings are recognized as citizens. These circumstances, including the practice of jus
sanguinis countries, show that it is a generally accepted principle of international law to presume
foundlings as having been born of nationals of the country in which the foundling is found.

Current legislation reveals the adherence of the Philippines to this generally accepted principle of
international law. In particular, R.A. No. 8552, R.A. No. 8042 and this Court's Rules on Adoption,
expressly refer to "Filipino children." In all of them, foundlings are among the Filipino children who
could be adopted. Likewise, it has been pointed that the DFA issues passports to foundlings.
Passports are by law, issued only to citizens. This shows that even the executive department, acting
through the DFA, considers foundlings as Philippine citizens.

Adopting these legal principles from the 1930 Hague Convention and the 1961 Convention on
Statelessness is rational and reasonable and consistent with the jus sanguinis regime in our
Constitution. The presumption of natural-born citizenship of foundlings stems from the presumption
that their parents are nationals of the Philippines. As the empirical data provided by the PSA show, that
presumption is at more than 99% and is a virtual certainty.

In sum, all of the international law conventions and instruments on the matter of nationality of
foundlings were designed to address the plight of a defenseless class which suffers from a misfortune
not of their own making. We cannot be restrictive as to their application if we are a country which calls
itself civilized and a member of the community of nations. The Solicitor General's warning in his
opening statement is relevant:

.... the total effect of those documents is to signify to this Honorable Court that those treaties and
conventions were drafted because the world community is concerned that the situation of foundlings
renders them legally invisible. It would be tragically ironic if this Honorable Court ended up using the
international instruments which seek to protect and uplift foundlings a tool to deny them political status
or to accord them second-class citizenship.138
The COMELEC also ruled139 that petitioner's repatriation in July 2006 under the provisions of R.A. No.
9225 did not result in the reacquisition of natural-born citizenship. The COMELEC reasoned that since
the applicant must perform an act, what is reacquired is not "natural-born" citizenship but only plain
"Philippine citizenship."

The COMELEC's rule arrogantly disregards consistent jurisprudence on the matter of repatriation
statutes in general and of R.A. No. 9225 in particular.

In the seminal case of Bengson Ill v. HRET, 140 repatriation was explained as follows:

Moreover, repatriation results in the recovery of the original nationality. This means that a naturalized
Filipino who lost his citizenship will be restored to his prior status as a naturalized Filipino citizen. On
the other hand, if he was originally a natural-born citizen before he lost his Philippine citizenship, he
will be restored to his former status as a natural-born Filipino.

R.A. No. 9225 is a repatriation statute and has been described as such in several cases. They
include Sobejana-Condon v. COMELEC141 where we described it as an "abbreviated repatriation
process that restores one's Filipino citizenship x x x." Also included is Parreno v. Commission on
Audit,142 which cited Tabasa v. Court of Appeals,143where we said that "[t]he repatriation of the former
Filipino will allow him to recover his natural-born citizenship. Parreno v. Commission on Audit144 is
categorical that "if petitioner reacquires his Filipino citizenship (under R.A. No. 9225), he
will ... recover his natural-born citizenship."

The COMELEC construed the phrase "from birth" in the definition of natural citizens as implying "that
natural-born citizenship must begin at birth and remain uninterrupted and continuous from birth." R.A.
No. 9225 was obviously passed in line with Congress' sole prerogative to determine how citizenship
may be lost or reacquired. Congress saw it fit to decree that natural-born citizenship may be
reacquired even if it had been once lost. It is not for the COMELEC to disagree with the Congress'
determination.

More importantly, COMELEC's position that natural-born status must be continuous was already
rejected in Bengson III v. HRET145 where the phrase "from birth" was clarified to mean at the time of
birth: "A person who at the time of his birth, is a citizen of a particular country, is a natural-born citizen
thereof." Neither is "repatriation" an act to "acquire or perfect" one's citizenship. In Bengson III v.
HRET, this Court pointed out that there are only two types of citizens under the 1987 Constitution:
natural-born citizen and naturalized, and that there is no third category for repatriated citizens:

It is apparent from the enumeration of who are citizens under the present Constitution that there are
only two classes of citizens: (1) those who are natural-born and (2) those who are naturalized in
accordance with law. A citizen who is not a naturalized Filipino, ie., did not have to undergo the
process of naturalization to obtain Philippine citizenship, necessarily is a natural-born Filipino.
Noteworthy is the absence in said enumeration of a separate category for persons who, after losing
Philippine citizenship, subsequently reacquire it. The reason therefor is clear: as to such persons, they
would either be natural-born or naturalized depending on the reasons for the loss of their citizenship
and the mode prescribed by the applicable law for the reacquisition thereof. As respondent Cruz was
not required by law to go through naturalization proceedings in order to reacquire his citizenship, he is
perforce a natural-born Filipino. As such, he possessed all the necessary qualifications to be elected
as member of the House of Representatives. 146

The COMELEC cannot reverse a judicial precedent. That is reserved to this Court. And while we may
always revisit a doctrine, a new rule reversing standing doctrine cannot be retroactively applied.
In Morales v. Court of Appeals and Jejomar Erwin S. Binay, Jr.,147 where we decreed reversed the
condonation doctrine, we cautioned that it "should be prospective in application for the reason that
judicial decisions applying or interpreting the laws of the Constitution, until reversed, shall form part of
the legal system of the Philippines." This Court also said that "while the future may ultimately uncover
a doctrine's error, it should be, as a general rule, recognized as good law prior to its abandonment.
Consequently, the people's reliance thereupon should be respected." 148
Lastly, it was repeatedly pointed out during the oral arguments that petitioner committed a falsehood
when she put in the spaces for "born to" in her application for repatriation under R.A. No. 9225 the
names of her adoptive parents, and this misled the BI to presume that she was a natural-born Filipino.
It has been contended that the data required were the names of her biological parents which are
precisely unknown.

This position disregards one important fact - petitioner was legally adopted. One of the effects of
adoption is "to sever all legal ties between the biological parents and the adoptee, except when the
biological parent is the spouse of the adoptee." 149 Under R.A. No. 8552, petitioner was also entitled to
an amended birth certificate "attesting to the fact that the adoptee is the child of the adopter(s)" and
which certificate "shall not bear any notation that it is an amended issue." 150 That law also requires that
"[a]ll records, books, and papers relating to the adoption cases in the files of the court, the Department
[of Social Welfare and Development], or any other agency or institution participating in the adoption
proceedings shall be kept strictly confidential." 151 The law therefore allows petitioner to state that her
adoptive parents were her birth parents as that was what would be stated in her birth certificate
anyway. And given the policy of strict confidentiality of adoption records, petitioner was not obligated to
disclose that she was an adoptee.

Clearly, to avoid a direct ruling on the qualifications of petitioner, which it cannot make in the same
case for cancellation of COC, it resorted to opinionatedness which is, moreover, erroneous. The whole
process undertaken by COMELEC is wrapped in grave abuse of discretion.

On Residence

The tainted process was repeated in disposing of the issue of whether or not petitioner committed
false material representation when she stated in her COC that she has before and until 9 May 2016
been a resident of the Philippines for ten (10) years and eleven (11) months.

Petitioner's claim that she will have been a resident for ten (10) years and eleven (11) months on the
day before the 2016 elections, is true.

The Constitution requires presidential candidates to have ten (10) years' residence in the Philippines
before the day of the elections. Since the forthcoming elections will be held on 9 May 2016, petitioner
must have been a resident of the Philippines prior to 9 May 2016 for ten (10) years. In answer to the
requested information of "Period of Residence in the Philippines up to the day before May 09, 2016,"
she put in "10 years 11 months" which according to her pleadings in these cases corresponds to a
beginning date of 25 May 2005 when she returned for good from the U.S.

When petitioner immigrated to the U.S. in 1991, she lost her original domicile, which is the Philippines.
There are three requisites to acquire a new domicile: 1. Residence or bodily presence in a new
locality; 2. an intention to remain there; and 3. an intention to abandon the old domicile. 152 To
successfully effect a change of domicile, one must demonstrate an actual removal or an actual change
of domicile; a bona fide intention of abandoning the former place of residence and establishing a new
one and definite acts which correspond with the purpose. In other words, there must basically
be animus manendi coupled with animus non revertendi. The purpose to remain in or at the domicile
of choice must be for an indefinite period of time; the change of residence must be voluntary; and the
residence at the place chosen for the new domicile must be actual. 153

Petitioner presented voluminous evidence showing that she and her family abandoned their U.S.
domicile and relocated to the Philippines for good. These evidence include petitioner's former U.S.
passport showing her arrival on 24 May 2005 and her return to the Philippines every time she travelled
abroad; e-mail correspondences starting in March 2005 to September 2006 with a freight company to
arrange for the shipment of their household items weighing about 28,000 pounds to the Philippines; e-
mail with the Philippine Bureau of Animal Industry inquiring how to ship their dog to the Philippines;
school records of her children showing enrollment in Philippine schools starting June 2005 and for
succeeding years; tax identification card for petitioner issued on July 2005; titles for condominium and
parking slot issued in February 2006 and their corresponding tax declarations issued in April 2006;
receipts dated 23 February 2005 from the Salvation Army in the U.S. acknowledging donation of items
from petitioner's family; March 2006 e-mail to the U.S. Postal Service confirming request for change of
address; final statement from the First American Title Insurance Company showing sale of their U.S.
home on 27 April 2006; 12 July 2011 filled-up questionnaire submitted to the U.S. Embassy where
petitioner indicated that she had been a Philippine resident since May 2005; affidavit from Jesusa
Sonora Poe (attesting to the return of petitioner on 24 May 2005 and that she and her family stayed
with affiant until the condominium was purchased); and Affidavit from petitioner's husband (confirming
that the spouses jointly decided to relocate to the Philippines in 2005 and that he stayed behind in the
U.S. only to finish some work and to sell the family home).

The foregoing evidence were undisputed and the facts were even listed by the COMELEC, particularly
in its Resolution in the Tatad, Contreras and Valdez cases.

However, the COMELEC refused to consider that petitioner's domicile had been timely changed as of
24 May 2005. At the oral arguments, COMELEC Commissioner Arthur Lim conceded the presence of
the first two requisites, namely, physical presence and animus manendi, but maintained there was
no animus non-revertendi.154 The COMELEC disregarded the import of all the evidence presented by
petitioner on the basis of the position that the earliest date that petitioner could have started residence
in the Philippines was in July 2006 when her application under R.A. No. 9225 was approved by the BI.
In this regard, COMELEC relied on Coquilla v. COMELEC,155 Japzon v. COMELEC156 and Caballero v.
COMELEC. 157 During the oral arguments, the private respondents also added Reyes v.
COMELEC.158 Respondents contend that these cases decree that the stay of an alien former Filipino
cannot be counted until he/she obtains a permanent resident visa or reacquires Philippine citizenship,
a visa-free entry under a balikbayan stamp being insufficient. Since petitioner was still an American
(without any resident visa) until her reacquisition of citizenship under R.A. No. 9225, her stay from 24
May 2005 to 7 July 2006 cannot be counted.

But as the petitioner pointed out, the facts in these four cases are very different from her situation.
In Coquilla v. COMELEC,159 the only evidence presented was a community tax certificate secured by
the candidate and his declaration that he would be running in the elections. Japzon v.
COMELEC160 did not involve a candidate who wanted to count residence prior to his reacquisition of
Philippine citizenship. With the Court decreeing that residence is distinct from citizenship, the issue
there was whether the candidate's acts after reacquisition sufficed to establish residence. In Caballero
v. COMELEC, 161 the candidate admitted that his place of work was abroad and that he only visited
during his frequent vacations. In Reyes v. COMELEC,162 the candidate was found to be an American
citizen who had not even reacquired Philippine citizenship under R.A. No. 9225 or had renounced her
U.S. citizenship. She was disqualified on the citizenship issue. On residence, the only proof she
offered was a seven-month stint as provincial officer. The COMELEC, quoted with approval by this
Court, said that "such fact alone is not sufficient to prove her one-year residency."

It is obvious that because of the sparse evidence on residence in the four cases cited by the
respondents, the Court had no choice but to hold that residence could be counted only from
acquisition of a permanent resident visa or from reacquisition of Philippine citizenship. In contrast, the
evidence of petitioner is overwhelming and taken together leads to no other conclusion that she
decided to permanently abandon her U.S. residence (selling the house, taking the children from U.S.
schools, getting quotes from the freight company, notifying the U.S. Post Office of the abandonment of
their address in the U.S., donating excess items to the Salvation Army, her husband resigning from
U.S. employment right after selling the U.S. house) and permanently relocate to the Philippines and
actually re-established her residence here on 24 May 2005 (securing T.I.N, enrolling her children in
Philippine schools, buying property here, constructing a residence here, returning to the Philippines
after all trips abroad, her husband getting employed here). Indeed, coupled with her eventual
application to reacquire Philippine citizenship and her family's actual continuous stay in the Philippines
over the years, it is clear that when petitioner returned on 24 May 2005 it was for good.

In this connection, the COMELEC also took it against petitioner that she had entered the Philippines
visa-free as a balikbayan. A closer look at R.A. No. 6768 as amended, otherwise known as the "An Act
Instituting a Balikbayan Program," shows that there is no overriding intent to treat balikbayans as
temporary visitors who must leave after one year. Included in the law is a former Filipino who has been
naturalized abroad and "comes or returns to the Philippines." 163 The law institutes
a balikbayan program "providing the opportunity to avail of the necessary training to enable
the balikbayan to become economically self-reliant members of society upon their return to the
country"164 in line with the government's "reintegration program." 165 Obviously, balikbayans are not
ordinary transients.

Given the law's express policy to facilitate the return of a balikbayan and help him reintegrate into
society, it would be an unduly harsh conclusion to say in absolute terms that the balikbayan must
leave after one year. That visa-free period is obviously granted him to allow him to re-establish his life
and reintegrate himself into the community before he attends to the necessary formal and legal
requirements of repatriation. And that is exactly what petitioner did - she reestablished life here by
enrolling her children and buying property while awaiting the return of her husband and then applying
for repatriation shortly thereafter.

No case similar to petitioner's, where the former Filipino's evidence of change in domicile is extensive
and overwhelming, has as yet been decided by the Court. Petitioner's evidence of residence is
unprecedented. There is no judicial precedent that comes close to the facts of residence of petitioner.
There is no indication in Coquilla v. COMELEC,166 and the other cases cited by the respondents that
the Court intended to have its rulings there apply to a situation where the facts are different. Surely,
the issue of residence has been decided particularly on the facts-of-the case basis.

To avoid the logical conclusion pointed out by the evidence of residence of petitioner, the COMELEC
ruled that petitioner's claim of residence of ten (10) years and eleven (11) months by 9 May 2016 in
her 2015 COC was false because she put six ( 6) years and six ( 6) months as "period of residence
before May 13, 2013" in her 2012 COC for Senator. Thus, according to the COMELEC, she started
being a Philippine resident only in November 2006. In doing so, the COMELEC automatically assumed
as true the statement in the 2012 COC and the 2015 COC as false.

As explained by petitioner in her verified pleadings, she misunderstood the date required in the 2013
COC as the period of residence as of the day she submitted that COC in 2012. She said that she
reckoned residency from April-May 2006 which was the period when the U.S. house was sold and her
husband returned to the Philippines. In that regard, she was advised by her lawyers in 2015 that
residence could be counted from 25 May 2005.

Petitioner's explanation that she misunderstood the query in 2012 (period of residence before 13 May
2013) as inquiring about residence as of the time she submitted the COC, is bolstered by the change
which the COMELEC itself introduced in the 2015 COC which is now "period of residence in the
Philippines up to the day before May 09, 2016." The COMELEC would not have revised the query if it
did not acknowledge that the first version was vague.

That petitioner could have reckoned residence from a date earlier than the sale of her U.S. house and
the return of her husband is plausible given the evidence that she had returned a year before. Such
evidence, to repeat, would include her passport and the school records of her children.

It was grave abuse of discretion for the COMELEC to treat the 2012 COC as a binding and conclusive
admission against petitioner. It could be given in evidence against her, yes, but it was by no means
conclusive. There is precedent after all where a candidate's mistake as to period of residence made in
a COC was overcome by evidence. In Romualdez-Marcos v. COMELEC,167 the candidate mistakenly
put seven (7) months as her period of residence where the required period was a minimum of one
year. We said that "[i]t is the fact of residence, not a statement in a certificate of candidacy which
ought to be decisive in determining whether or not an individual has satisfied the constitutions
residency qualification requirement." The COMELEC ought to have looked at the evidence presented
and see if petitioner was telling the truth that she was in the Philippines from 24 May 2005. Had the
COMELEC done its duty, it would have seen that the 2012 COC and the 2015 COC both correctly
stated the pertinent period of residency.

The COMELEC, by its own admission, disregarded the evidence that petitioner actually and physically
returned here on 24 May 2005 not because it was false, but only because COMELEC took the position
that domicile could be established only from petitioner's repatriation under R.A. No. 9225 in July 2006.
However, it does not take away the fact that in reality, petitioner had returned from the U.S. and was
here to stay permanently, on 24 May 2005. When she claimed to have been a resident for ten (10)
years and eleven (11) months, she could do so in good faith.
For another, it could not be said that petitioner was attempting to hide anything. As already stated, a
petition for quo warranto had been filed against her with the SET as early as August 2015. The event
from which the COMELEC pegged the commencement of residence, petitioner's repatriation in July
2006 under R.A. No. 9225, was an established fact to repeat, for purposes of her senatorial candidacy.

Notably, on the statement of residence of six (6) years and six (6) months in the 2012 COC, petitioner
recounted that this was first brought up in the media on 2 June 2015 by Rep. Tobias Tiangco of the
United Nationalist Alliance. Petitioner appears to have answered the issue immediately, also in the
press. Respondents have not disputed petitioner's evidence on this point. From that time therefore
when Rep. Tiangco discussed it in the media, the stated period of residence in the 2012 COC and the
circumstances that surrounded the statement were already matters of public record and were not
hidden.

Petitioner likewise proved that the 2012 COC was also brought up in the SET petition for quo
warranto. Her Verified Answer, which was filed on 1 September 2015, admitted that she made a
mistake in the 2012 COC when she put in six ( 6) years and six ( 6) months as she misunderstood the
question and could have truthfully indicated a longer period. Her answer in the SET case was a matter
of public record. Therefore, when petitioner accomplished her COC for President on 15 October 2015,
she could not be said to have been attempting to hide her erroneous statement in her 2012 COC for
Senator which was expressly mentioned in her Verified Answer.

The facts now, if not stretched to distortion, do not show or even hint at an intention to hide the 2012
statement and have it covered by the 2015 representation. Petitioner, moreover, has on her side this
Court's pronouncement that:

Concededly, a candidate's disqualification to run for public office does not necessarily constitute
material misrepresentation which is the sole ground for denying due course to, and for the cancellation
of, a COC. Further, as already discussed, the candidate's misrepresentation in his COC must not only
refer to a material fact (eligibility and qualifications for elective office), but should evince a deliberate
intent to mislead, misinform or hide a fact which would otherwise render a candidate ineligible. It must
be made with an intention to deceive the electorate as to one's qualifications to run for public office. 168

In sum, the COMELEC, with the same posture of infallibilism, virtually ignored a good number of
evidenced dates all of which can evince animus manendi to the Philippines and animus non
revertedi to the United States of America. The veracity of the events of coming and staying home was
as much as dismissed as inconsequential, the focus having been fixed at the petitioner's "sworn
declaration in her COC for Senator" which the COMELEC said "amounts to a declaration and
therefore an admission that her residence in the Philippines only commence sometime in November
2006"; such that "based on this declaration, [petitioner] fails to meet the residency requirement for
President." This conclusion, as already shown, ignores the standing jurisprudence that it is the fact of
residence, not the statement of the person that determines residence for purposes of compliance with
the constitutional requirement of residency for election as President. It ignores the easily researched
matter that cases on questions of residency have been decided favorably for the candidate on the
basis of facts of residence far less in number, weight and substance than that presented by
petitioner.169 It ignores, above all else, what we consider as a primary reason why petitioner cannot be
bound by her declaration in her COC for Senator which declaration was not even considered by the
SET as an issue against her eligibility for Senator. When petitioner made the declaration in her COC
for Senator that she has been a resident for a period of six (6) years and six (6) months counted up to
the 13 May 2013 Elections, she naturally had as reference the residency requirements for election as
Senator which was satisfied by her declared years of residence. It was uncontested during the oral
arguments before us that at the time the declaration for Senator was made, petitioner did not have as
yet any intention to vie for the Presidency in 2016 and that the general public was never made aware
by petitioner, by word or action, that she would run for President in 2016. Presidential candidacy has a
length-of-residence different from that of a senatorial candidacy. There are facts of residence other
than that which was mentioned in the COC for Senator. Such other facts of residence have never been
proven to be false, and these, to repeat include:

[Petitioner] returned to the Philippines on 24 May 2005. (petitioner's] husband however stayed in the
USA to finish pending projects and arrange the sale of their family home.
Meanwhile [petitioner] and her children lived with her mother in San Juan City. [Petitioner] enrolled
Brian in Beacon School in Taguig City in 2005 and Hanna in Assumption College in Makati City in
2005. Anika was enrolled in Learning Connection in San Juan in 2007, when she was already old
enough to go to school.

In the second half of 2005, [petitioner] and her husband acquired Unit 7F of One Wilson Place
Condominium in San Juan. [Petitioner] and her family lived in Unit 7F until the construction of their
family home in Corinthian Hills was completed.

Sometime in the second half of 2005, [petitioner's] mother discovered that her former lawyer who
handled [petitioner's] adoption in 1974 failed to secure from the Office of the Civil Registrar of Iloilo a
new Certificate of Live Birth indicating [petitioner's] new name and stating that her parents are "Ronald
Allan K. Poe" and "Jesusa L. Sonora."

In February 2006, [petitioner] travelled briefly to the US in order to supervise the disposal of some of
the family's remaining household belongings.1a\^/phi1 [Petitioner] returned to the Philippines on 11
March 2006.

In late March 2006, [petitioner's] husband informed the United States Postal Service of the family's
abandonment of their address in the US.

The family home in the US was sole on 27 April 2006.

In April 2006, [petitioner's] husband resigned from his work in the US. He returned to the Philippines
on 4 May 2006 and began working for a Philippine company in July 2006.

In early 2006, [petitioner] and her husband acquired a vacant lot in Corinthian Hills, where they
eventually built their family home.170

In light of all these, it was arbitrary for the COMELEC to satisfy its intention to let the case fall under
the exclusive ground of false representation, to consider no other date than that mentioned by
petitioner in her COC for Senator.

All put together, in the matter of the citizenship and residence of petitioner for her candidacy as
President of the Republic, the questioned Resolutions of the COMELEC in Division and En Banc are,
one and all, deadly diseased with grave abuse of discretion from root to fruits.

WHEREFORE, the petition is GRANTED. The Resolutions, to wit:

1. dated 1 December 2015 rendered through the COMELEC Second Division, in SPA No. 15-001
(DC), entitled Estrella C. Elamparo, petitioner, vs. Mary Grace Natividad Sonora Poe-Llamanzares,
respondent, stating that:

[T]he Certificate of Candidacy for President of the Republic of the Philippines in the May 9, 2016
National and Local Elections filed by respondent Mary Grace Natividad Sonora Poe-Llamanzares is
hereby GRANTED.

2. dated 11 December 2015, rendered through the COMELEC First Division, in the consolidated cases
SPA No. 15-002 (DC) entitled Francisco S. Tatad, petitioner, vs. Mary Grace Natividad Sonora Poe-
Llamanzares, respondent; SPA No. 15-007 (DC) entitled Antonio P. Contreras, petitioner, vs. Mary
Grace Natividad Sonora Poe-Llamanzares, respondent; and SPA No. 15-139 (DC) entitled Amado D.
Valdez, petitioner, v. Mary Grace Natividad Sonora Poe-Llamanzares, respondent; stating that:

WHEREFORE, premises considered, the Commission RESOLVED, as it hereby RESOLVES, to


GRANT the petitions and cancel the Certificate of Candidacy of MARY GRACE NATIVIDAD SONORA
POE-LLAMANZARES for the elective position of President of the Republic of the Philippines in
connection with the 9 May 2016 Synchronized Local and National Elections.
3. dated 23 December 2015 of the COMELEC En Banc, upholding the 1 December 2015 Resolution of
the Second Division stating that:

WHEREFORE, premises considered, the Commission RESOLVED, as it hereby RESOLVES, to


DENY the Verified Motion for Reconsideration of SENATOR MARY GRACE NATIVIDAD SONORA
POE-LLAMANZARES. The Resolution dated 11 December 2015 of the Commission First Division is
AFFIRMED.

4. dated 23 December 2015 of the COMELEC En Banc, upholding the 11 December 2015 Resolution
of the First Division.

are hereby ANNULED and SET ASIDE. Petitioner MARY GRACE NATIVIDAD SONORA POE-
LLAMANZARES is DECLARED QUALIFIED to be a candidate for President in the National and Local
Elections of 9 May 2016.

SO ORDERED.

////////////////////////////////////////////////////////////////////////////////////////////////////////////////////////////////////

GRACE POE vs. COMELEC


The Case
Grace Poe was found abandoned as a newborn infant in the Parish Church of Jaro, Iloilo by Edgardo
Militar in 1968. Parental care and custody over her was passed on by Edgardo to his relatives,
Emiliano Militar and his wife. Emiliano reported and registered Grace Poe as a foundling with the
Office of the Civil Registrar of Iloilo City. Fenando Poe, Jr. and Susan Roces adopted Grace Poe.
1991 Poe went to the US to be a permanent resident therein
2001 She became a naturalized US citizen
First quarter of 2005 she came back to the Philippines to permanently reside herein
February 14, 2006- she went back to the US to dispose family belongings
July 18, 2006 she re-acquired Filipino citizenship
According to Poe in her 2013 COC for Senator, before the May 13, 2013 election, she has been a
resident of the Philippines for 6 years and 6 months (reckoned from year 2006 when she re-acquired
her Filipino citizenship under RA 9225).
Poe filed her COC for Presidency for the May 9, 2016 elections (hence, computing from May, 2013,
she has been a resident in the Philippines for 9 years and 6 months only)
However, in her COC, Poe declared that she is a natural born and her residence in the Philippine up
to the day before election would be 10 years and 11 months counted from May 24, 2005 (when she
returned from the US to the Philippines for good).

RULING OF THE SUPREME COURT


Poe is qualified to be a candidate for President in the National and Local Election on May 9, 2016.

1) Is Poe, a foundling, a natural-born citizen? Yes, based on:


a) Circumstantial evidence
b) Legislation
c) Generally accepted principles of international law

Circumstantial evidence
There is more than sufficient evidence that Poe has Filipino parents and is therefore a natural-born
Filipino. xxx. [T]here is a high probability that her parents are Filipinos. The Solicitor General offered
official Statistics from the Philippine Statistics office that from 1965 to 1975, the total number of
foreigners born in the Philippines was 15,985. While the Filipinos born in the country were more than
10 Million. On this basis, there is a 99% chance that the child born in the Philippines would be a
Filipino which in turn, would indicate more than ample probability that Poes parents are Filipinos.

Other circumstantial evidence of the nationality of Poes parents are the fact that:
1. She was abandoned in a Roman Catholic Church in Iloilo
2. She has typical Filipino features.
There are disputable presumptions that things have happened according to the ordinary course of
nature. On this basis, it is safer to assume that Poes parents are Filipinos. To assume otherwise is to
accept the absurd.

Legislation
Foundlings are as a class, natural born citizens.
The amendment to the Constitution proposed by constitutionalist Rafols to include foundlings as
natural born citizens was not carried out, not because there was any objection to the notion that
persons of unknown parentage are not citizens, but only because their number was not enough to
merit specific mention. There was no intent or language that would permit discrimination against
foundlings. On the contrary, all three Constitutions guarantee the basic right to equal protection of the
laws.
Likewise, domestic laws on adoption support the principle that foundlings are Filipinos. These laws
do not provide that adoption confers citizenship upon the adoptee, rather, the adoptee must be Filipino
in the first place to be adopted.
Recent legislation all expressly refer to Filipino children and include foundlings as among Filipino
children who may be adopted.

Generally accepted principles of international law


The common thread of the Universal Declaration of Human Rights, the Convention on the Rights of
the Child and the International Convent on Civil and Political Rights obligates the Philippines to
grant nationality from birth and to ensure that no child is stateless. The principles stated in the:
1. Hague Convention on Certain Questions Relation to the Conflict of Nationality laws (that a foundling
is presumed to have the nationality of the country of birth)
2. Convention on the Reduction of Statelessness (foundling is presumed born of citizens of the country
where he is found)
bind the Philippines although we are not signatory to these conventions.

Poes evidence shows that at least 60 countries in Asia, North and South America and Europe have
passed legislation recognizing foundlings as its citizens. 166 out of 189 countries accept that
foundlings are recognized as citizens. Hence, there is a generally accepted principle of international
law to presume foundlings as having been born and a national of the country in which it is found.

2) After renouncing her American citizenship and after having taken her Oath of Allegiance to the
Republic of the Philippines, has Poe re-acquired her status as a natural-born Filipino citizen? Yes,
Poes repatriation resulted to reacquisition of natural born citizenship.

A natural born citizen before he lost his Philippine nationality will be restored to his former status as
natural born Filipino after repatriation (Benson v. HRET, Pareno v. Commission on Audit etc).

3) Has Poe satisfied the 10 year residency requirement? Yes, she will have been a resident for 10
years and 11 months on the day of the election.

[T]here is overwhelming evidence that leads to no to other conclusion that Poe decided to permanently
abandon her US residence and reside in the Philippines as early as May 24, 2005.

Poe presented voluminous evidence showing that she and her family abandoned their US domicile
and relocated to the Philippines for good. These evidence include former US passport showing her
arrival on May 24, 2005 and her return to the Philippines every time she travelled abroad, email
correspondences with freight company to arrange for the shipment of household items as well as with
the pet Bureau; school records of her children showing enrolment in the Philippine to the Philippine
schools starting on June 2005 etc. xxx These evidence, coupled with her eventual application to
reacquire Philippine citizenship is clear that when she returned in May 2005, it was for good.
Poe was able to prove that her statement in her 2013 COC was only a mistake in good faith. As
explained by Grace Poe, she misunderstood the date required in the 2013 COC as the period of
residence as of the day she submitted that COC in 2012. She said that she reckoned residency from
April-May 2006 which was the period when the U.S. house was sold and her husband returned to the
Philippines. In that regard, she was advised by her lawyers in 2015 that residence could be counted
from 25 May 2005. Such a mistake could be given in evidence against her but it was by no means
conclusive considering the overwhelming evidence submitted by Poe.

/////////////////////////////////////////////////////////////////////////////////////////////////////////////////

COMMONWEALTH ACT No. 63

AN ACT PROVIDING FOR THE WAYS IN WHICH PHILIPPINE CITIZENSHIP MAY BE LOST OR
REACQUIRED

Section. 2. How citizenship may be reacquired. Citizenship may be reacquired:

(1) By naturalization: Provided, That the applicant possess none of the disqualification's prescribed in
section two of Act Numbered Twenty-nine hundred and twenty-seven, 3

(2) By repatriation of deserters of the Army, Navy or Air Corp: Provided, That a woman who lost her
citizenship by reason of her marriage to an alien may be repatriated in accordance with the provisions
of this Act after the termination of the marital status; 4and

(3) By direct act of the National Assembly.

Sec. 3. Procedure incident to reacquisition of Philippine citizenship. The procedure prescribed for
naturalization under Act Numbered Twenty-nine hundred and twenty-seven, 5 as amended, shall apply
to the reacquisition of Philippine citizenship by naturalization provided for in the next preceding
section: Provided, That the qualifications and special qualifications prescribed in section three and four
of said Act shall not be required: And provided, further,

(1) That the applicant be at least twenty-one years of age and shall have resided in the Philippines at
least six months before he applies for naturalization;

(2) That he shall have conducted himself in a proper and irreproachable manner during the entire
period of his residence in the Philippines, in his relations with the constituted government as well as
with the community in which he is living; and

(3) That he subscribes to an oath declaring his intention to renounce absolutely and perpetually all
faith and allegiance to the foreign authority, state or sovereignty of which he was a citizen or subject.

Sec. 4. Repatriation shall be effected by merely taking the necessary oath of allegiance to the
Commonwealth6 of the Philippines and registration in the proper civil registry.

///////////////////////////////////////////////////////////////////////////////////////////////////////////////////

[REPUBLIC ACT NO. 9139]

AN ACT PROVIDING FOR THE ACQUISITION OF PHILIPPINE CITIZENSHIP FOR CERTAIN


ALIENS BY ADMINISTRATIVE NATURALIZATION AND FOR OTHER PURPOSES
SEC. 2. Declaration of Policy. The State shall control and regulate the admission and integration of
aliens into its territory and body politic including the grant of citizenship to aliens. Towards this end,
aliens born and residing in the Philippines may be granted Philippine citizenship by administrative
proceedings subject to certain requirements dictated by national security and interest.

SEC. 3. Qualifications. Subject to the provisions of the succeeding section, any person desiring to
avail of the benefits of this Act must meet the following qualifications:

(a) The applicant must be born in the Philippines and residing therein since birth;

(b) The applicant must not be less than eighteen (18) years of age, at the time of filing of his/her
petition;

(c) The applicant must be of good moral character and believes in the underlying principles of the
Constitution, and must have conducted himself/herself in a proper and irreproachable manner during
his/her entire period of residence in the Philippines in his relation with the duly constituted government
as well as with the community in which he/she is living;

(d) The applicant must have received his/her primary and secondary education in any public school or
private educational institution duly recognized by the Department of Education, Culture and Sports,
where Philippine history, government and civics are taught and prescribed as part of the school
curriculum and where enrollment is not limited to any race or nationality: Provided, That should he/she
have minor children of school age, he/she must have enrolled them in similar schools;

(e) The applicant must have a known trade, business, profession or lawful occupation, from which
he/she derives income sufficient for his/her support and if he/she is married and/or has dependents,
also that of his/her family: Provided, however, That this shall not apply to applicants who are college
degree holders but are unable to practice their profession because they are disqualified to do so by
reason of their citizenship;

(f) The applicant must be able to read, write and speak Filipino or any of the dialects of the Philippines;
and

(g) The applicant must have mingled with the Filipinos and evinced a sincere desire to learn and
embrace the customs, traditions and ideals of the Filipino people.

SEC. 4. Disqualifications. The following are not qualified to be naturalized as Filipino citizens under
this Act:

(a) Those opposed to organized government or affiliated with any association or group of persons who
uphold and teach doctrines opposing all organized governments;

(b) Those defending or teaching the necessity of or propriety of violence, personal assault or
assassination for the success or predominance of their ideas;

(c) Polygamists or believers in the practice of polygamy;

(d) Those convicted of crimes involving moral turpitude;

(e) Those suffering from mental alienation or incurable contagious diseases;

(f) Those who, during the period of their residence in the Philippines, have not mingled socially with
Filipinos, or who have not evinced a sincere desire to learn and embrace the customs, traditions and
ideals of the Filipinos;

(g) Citizens or subjects with whom the Philippines is at war, during the period of such war; and
(h) Citizens or subjects of a foreign country whose laws do not grant Filipinos the right to be
naturalized citizens or subjects thereof.

////////////////////////////////////////////////////////////////////////////////////////////////////////////////////////////////////

Republic of the Philippines


SUPREME COURT
Manila
EN BANC
G.R. No. 87193 June 23, 1989
JUAN GALLANOSA FRIVALDO, petitioner,
vs.
COMMISSION ON ELECTIONS AND THE LEAGUE OF MUNICIPALITIES, SORSOGON CHAPTER,
HEREIN REPRESENTED BY ITS PRESIDENT, SALVADOR NEE ESTUYE, respondents.
J.L. Misa & Associates for petitioner.
Lladoc, Huab & Associates for private respondent.

DECISION
CRUZ, J.:
Petitioner Juan G. Frivaldo was proclaimed governor-elect of the province of Sorsogon on January 22,
1988, and assumed office in due time. On October 27, 1988, the League of Municipalities, Sorsogon
Chapter (hereafter, League), represented by its President, Salvador Estuye, who was also suing in his
personal capacity, filed with the Commission on Elections a petition for the annulment of Frivaldos
election and proclamation on the ground that he was not a Filipino citizen, having been naturalized in
the United States on January 20, 1983. In his answer dated May 22, 1988, Frivaldo admitted that he
was naturalized in the United States as alleged but pleaded the special and affirmative defenses that
he had sought American citizenship only to protect himself against President Marcos. His
naturalization, he said, was merely forced upon himself as a means of survival against the unrelenting
persecution by the Martial Law Dictators agents abroad. He added that he had returned to the
Philippines after the EDSA revolution to help in the restoration of democracy. He also argued that the
challenge to his title should be dismissed, being in reality a quo warranto petition that should have
been filed within ten days from his proclamation, in accordance with Section 253 of the Omnibus
Election Code. The League, moreover, was not a proper party because it was not a voter and so could
not sue under the said section.
Frivaldo moved for a preliminary hearing on his affirmative defenses but the respondent Commission
on Elections decided instead by its Order of January 20, 1988, to set the case for hearing on the
merits. His motion for reconsideration was denied in another Order dated February 21, 1988. He then
came to this Court in a petition for certiorari and prohibition to ask that the said orders be set aside on
the ground that they had been rendered with grave abuse of discretion. Pending resolution of the
petition, we issued a temporary order against the hearing on the merits scheduled by the COMELEC
and at the same time required comments from the respondents.
In their Comment, the private respondents reiterated their assertion that Frivaldo was a naturalized
American citizen and had not reacquired Philippine citizenship on the day of the election on January
18, 1988. He was therefore not qualified to run for and be elected governor. They also argued that
their petition in the Commission on Elections was not really for quo warranto under Section 253 of the
Omnibus Election Code. The ultimate purpose was to prevent Frivaldo from continuing as governor,
his candidacy and election being null and void ab initio because of his alienage. Even if their petition
were to be considered as one for quo warranto, it could not have been filed within ten days from
Frivaldos proclamation because it was only in September 1988 that they received proof of his
naturalization. And assuming that the League itself was not a proper party, Estuye himself, who was
suing not only for the League but also in his personal capacity, could nevertheless institute the suit by
himself alone.
Speaking for the public respondent, the Solicitor General supported the contention that Frivaldo was
not a citizen of the Philippines and had not repatriated himself after his naturalization as an American
citizen. As an alien, he was disqualified from public office in the Philippines. His election did not cure
this defect because the electorate of Sorsogon could not amend the Constitution, the Local
Government Code, and the Omnibus Election Code. He also joined in the private respondents
argument that Section 253 of the Omnibus Election Code was not applicable because what the
League and Estuye were seeking was not only the annulment of the proclamation and election of
Frivaldo. He agreed that they were also asking for the termination of Frivaldos incumbency as
governor of Sorsogon on the ground that he was not a Filipino.
In his Reply, Frivaldo insisted that he was a citizen of the Philippines because his naturalization as an
American citizen was not impressed with voluntariness. In support he cited the Nottebohm Case,
[(1955 I.C.J. 4; 49 A.J.I.L. 396 (1955)] where a German nationals naturalization in Liechtenstein was
not recognized because it had been obtained for reasons of convenience only. He said he could not
have repatriated himself before the 1988 elections because the Special Committee on Naturalization
created for the purpose by LOI No. 27C had not yet been organized then. His oath in his certificate of
candidacy that he was a natural-born citizen should be a sufficient act of repatriation. Additionally, his
active participation in the 1987 congressional elections had divested him of American citizenship under
the laws of the United States, thus restoring his Philippine citizenship. He ended by reiterating his
prayer for the rejection of the move to disqualify him for being time-barred under Section 253 of the
Omnibus Election Code.
Considering the importance and urgency of the question herein raised, the Court has decided to
resolve it directly instead of allowing the normal circuitous route that will after all eventually end with
this Court, albeit only after a, long delay. We cannot permit this delay. Such delay will be inimical to the
public interest and the vital principles of public office to be here applied.
It is true that the Commission on Elections has the primary jurisdiction over this question as the sole
judge of all contests relating to the election, returns and qualifications of the members of the Congress
and elective provincial and city officials. However, the decision on Frivaldos citizenship has already
been made by the COMELEC through its counsel, the Solicitor General, who categorically claims that
Frivaldo is a foreigner. We assume this stance was taken by him after consultation with the public
respondent and with its approval. It therefore represents the decision of the COMELEC itself that we
may now review. Exercising our discretion to interpret the Rules of Court and the Constitution, we shall
consider the present petition as having been filed in accordance with Article IX-A Section 7, of the
Constitution, to challenge the aforementioned Orders of the COMELEC.
The basic question we must resolve is whether or not Juan G. Frivaldo was a citizen of the Philippines
at the time of his election on January 18, 1988, as provincial governor of Sorsogon. All the other
issues raised in this petition are merely secondary to this basic question.
The reason for this inquiry is the provision in Article XI, Section 9, of the Constitution that all public
officials and employees owe the State and the Constitution allegiance at all times and the specific
requirement in Section 42 of the Local Government Code that a candidate for local elective office must
be inter alia a citizen of the Philippines and a qualified voter of the constituency where he is running.
Section 117 of the Omnibus Election Code provides that a qualified voter must be, among other
qualifications, a citizen of the Philippines, this being an indispensable requirement for suffrage under
Article V, Section 1, of the Constitution.
In the certificate of candidacy he filed on November 19, 1987, Frivaldo described himself as a natural-
born citizen of the Philippines, omitting mention of any subsequent loss of such status. The evidence
shows, however, that he was naturalized as a citizen of the United States in 1983 per the following
certification from the United States District Court, Northern District of California, as duly authenticated
by Vice Consul Amado P. Cortez of the Philippine Consulate General in San Francisco, California,
U.S.A.
OFFICE OF THE CLERK
UNITED STATES DISTRICT COURT
NORTHERN DISTRICT OF CALIFORNIA
September 23, 1988
TO WHOM IT MAY CONCERN:
Our records show that JUAN GALLANOSA FRIVALDO, born on October 20, 1915, was naturalized in
this Court on January 20, 1983, and issued Certificate of Naturalization No. 11690178.
Petition No. 280225.
Alien Registration No. A23 079 270.
Very truly yours,
WILLIAM L. WHITTAKER
Clerk
by:
(Sgd.)
ARACELI V. BAREN
Deputy Clerk
This evidence is not denied by the petitioner. In fact, he expressly admitted it in his answer.
Nevertheless, as earlier noted, he claims it was forced on him as a measure of protection from the
persecution of the Marcos government through his agents in the United States.
The Court sees no reason not to believe that the petitioner was one of the enemies of the Marcos
dictatorship. Even so, it cannot agree that as a consequence thereof he was coerced into embracing
American citizenship. His feeble suggestion that his naturalization was not the result of his own free
and voluntary choice is totally unacceptable and must be rejected outright.
There were many other Filipinos in the United States similarly situated as Frivaldo, and some of them
subject to greater risk than he, who did not find it necessary nor do they claim to have been
coerced to abandon their cherished status as Filipinos. They did not take the oath of allegiance to
the United States, unlike the petitioner who solemnly declared on oath, that I absolutely and entirely
renounce and abjure all allegiance and fidelity to any foreign prince, potentate, state or sovereignty of
whom or which I have heretofore been a subject or citizen, meaning in his case the Republic of the
Philippines. The martyred Ninoy Aquino heads the impressive list of those Filipinos in exile who, unlike
the petitioner, held fast to their Philippine citizenship despite the perils of their resistance to the Marcos
regime.
The Nottebohm case cited by the petitioner invoked the international law principle of effective
nationality which is clearly not applicable to the case at bar. This principle is expressed in Article 5 of
the Hague Convention of 1930 on the Conflict of Nationality Laws as follows:
Art. 5. Within a third State a person having more than one nationality shall be treated as if he had only
one. Without prejudice to the application of its law in matters of personal status and of any convention
in force, a third State shall, of the nationalities which any such person possesses, recognize
exclusively in its territory either the nationality of the country in which he is habitually and principally
resident or the nationality of the country with which in the circumstances he appears to be in fact most
closely connected.
Nottebohm was a German by birth but a resident of Guatemala for 34 years when he applied for and
acquired naturalization in Liechtenstein one month before the outbreak of World War II. Many
members of his family and his business interests were in Germany. In 1943, Guatemala, which had
declared war on Germany, arrested Nottebohm and confiscated all his properties on the ground that
he was a German national. Liechtenstein thereupon filed suit on his behalf, as its citizen, against
Guatemala. The International Court of Justice held Nottebohm to be still a national of Germany, with
which he was more closely connected than with Liechtenstein.
That case is not relevant to the petition before us because it dealt with a conflict between the
nationality laws of two states as decided by a third state. No third state is involved in the case at bar; in
fact, even the United States is not actively claiming Frivaldo as its national. The sole question
presented to us is whether or not Frivaldo is a citizen of the Philippines under our own laws,
regardless of other nationality laws. We can decide this question alone as sovereign of our own
territory, conformably to Section 1 of the said Convention providing that it is for each State to
determine under its law who are its nationals.
It is also worth noting that Nottebohm was invoking his naturalization in Liechtenstein whereas in the
present case Frivaldo is rejecting his naturalization in the United States.
If he really wanted to disavow his American citizenship and reacquire Philippine citizenship, the
petitioner should have done so in accordance with the laws of our country. Under CA No. 63 as
amended by CA No. 473 and PD No. 725, Philippine citizenship may be reacquired by direct act of
Congress, by naturalization, or by repatriation.
While Frivaldo does not invoke either of the first two methods, he nevertheless claims he has
reacquired Philippine citizenship by virtue of a valid repatriation. He claims that by actively
participating in the elections in this country, he automatically forfeited American citizenship under the
laws of the United States. Such laws do not concern us here. The alleged forfeiture is between him
and the United States as his adopted country. It should be obvious that even if he did lose his
naturalized American citizenship, such forfeiture did not and could not have the effect of automatically
restoring his citizenship in the Philippines that he had earlier renounced. At best, what might have
happened as a result of the loss of his naturalized citizenship was that he became a stateless
individual.
Frivaldos contention that he could not have repatriated himself under LOI 270 because the Special
Committee provided for therein had not yet been constituted seems to suggest that the lack of that
body rendered his repatriation unnecessary. That is far-fetched if not specious Such a conclusion
would open the floodgates, as it were. It would allow all Filipinos who have renounced this country to
claim back their abandoned citizenship without formally rejecting their adopted state and reaffirming
their allegiance to the Philippines.
It does not appear that Frivaldo has taken these categorical acts. He contends that by simply filing his
certificate of candidacy he had, without more, already effectively recovered Philippine citizenship. But
that is hardly the formal declaration the law envisions surely, Philippine citizenship previously
disowned is not that cheaply recovered. If the Special Committee had not yet been convened, what
that meant simply was that the petitioner had to wait until this was done, or seek naturalization by
legislative or judicial proceedings.
The argument that the petition filed with the Commission on Elections should be dismissed for
tardiness is not well-taken. The herein private respondents are seeking to prevent Frivaldo from
continuing to discharge his office of governor because he is disqualified from doing so as a foreigner.
Qualifications for public office are continuing requirements and must be possessed not only at the time
of appointment or election or assumption of office but during the officers entire tenure. Once any of
the required qualifications is lost, his title may be seasonably challenged. If, say, a female legislator
were to marry a foreigner during her term and by her act or omission acquires his nationality, would
she have a right to remain in office simply because the challenge to her title may no longer be made
within ten days from her proclamation? It has been established, and not even denied, that the
evidence of Frivaldos naturalization was discovered only eight months after his proclamation and his
title was challenged shortly thereafter.
This Court will not permit the anomaly of a person sitting as provincial governor in this country while
owing exclusive allegiance to another country. The fact that he was elected by the people of Sorsogon
does not excuse this patent violation of the salutary rule limiting public office and employment only to
the citizens of this country. The qualifications prescribed for elective office cannot be erased by the
electorate alone. The will of the people as expressed through the ballot cannot cure the vice of
ineligibility, especially if they mistakenly believed, as in this case, that the candidate was qualified.
Obviously, this rule requires strict application when the deficiency is lack of citizenship. If a person
seeks to serve in the Republic of the Philippines, he must owe his total loyalty to this country only,
abjuring and renouncing all fealty and fidelity to any other state.
It is true as the petitioner points out that the status of the natural-born citizen is favored by the
Constitution and our laws, which is all the more reason why it should be treasured like a pearl of great
price. But once it is surrendered and renounced, the gift is gone and cannot be lightly restored. This
country of ours, for all its difficulties and limitations, is like a jealous and possessive mother. Once
rejected, it is not quick to welcome back with eager arms its prodigal if repentant children. The
returning renegade must show, by an express and unequivocal act, the renewal of his loyalty and love.
WHEREFORE, the petition is DISMISSED and petitioner JUAN G. FRIVALDO is hereby declared not
a citizen of the Philippines and therefore DISQUALIFIED from serving as Governor of the Province of
Sorsogon. Accordingly, he is ordered to vacate his office and surrender the same to the duly elected
Vice-Governor of the said province once this decision becomes final and executory. The temporary
restraining order dated March 9, 1989, is LIFTED.
SO ORDERED.
///////////////////////////////////////////////////////////////////////////////////////////////////////////////////////////////////

[G.R. No. 120295. June 28, 1996]

JUAN G. FRIVALDO, petitioner, vs. COMMISSION ON ELECTIONS, and RAUL R.


LEE, respondents.

[G.R. No. 123755. June 28, 1996]

RAUL R. LEE, petitioner, vs. COMMISSION ON ELECTIONS and JUAN G.


FRIVALDO, respondents.

DECISION
PANGANIBAN, J.:

The ultimate question posed before this Court in these twin cases is: Who should be declared the
rightful governor of Sorsogon

(i) Juan G. Frivaldo, who unquestionably obtained the highest number of votes in three successive
elections but who was twice declared by this Court to be disqualified to hold such office due to his
alien citizenship, and who now claims to have re-assumed his lost Philippine citizenship thru
repatriation;

(ii) Raul R. Lee, who was the second placer in the canvass, but who claims that the votes cast in favor
of Frivaldo should be considered void; that the electorate should be deemed to have intentionally
thrown away their ballots; and that legally, he secured the most number of valid votes; or

(iii) The incumbent Vice-Governor, Oscar G. Deri, who obviously was not voted directly to the position
of governor, but who according to prevailing jurisprudence should take over the said post inasmuch as,
by the ineligibility of Frivaldo, a "permanent vacancy in the contested office has occurred"?

In ruling for Frivaldo, the Court lays down new doctrines on repatriation,
clarifies/reiterates/amplifies existing jurisprudence on citizenship and elections, and upholds the
superiority of substantial justice over pure legalisms.
G.R. No. 123755.
This is a special civil action under Rules 65 and 58 of the Rules of Court for certiorari and
preliminary injunction to review and annul a Resolution of the respondent Commission on Elections
(Comelec), First Division,1 promulgated on December 19,19952 and another Resolution of the
Comelec en bane promulgated February 23, 19963 denying petitioner's motion for reconsideration.

The Facts

On March 20, 1995, private respondent Juan G. Frivaldo filed his Certificate of Candidacy for the
office of Governor of Sorsogon in the May 8, 1995 elections. On March 23, 1995, petitioner Raul R.
Lee, another candidate, filed a petition 4 with the Comelec docketed as SPA No. 95-028 praying that
Frivaldo "be disqualified from seeking or holding any public office or position by reason of not yet being
a citizen of the Philippines," and that his Certificate of Candidacy be cancelled. On May 1, 1995, the
Second Division of the Comelec promulgated a Resolution 5 granting the petition with the following
disposition:6

"WHEREFORE, this Division resolves to GRANT the petition and declares that respondent is
DISQUALIFIED to run for the Office of Governor of Sorsogon on the ground that he is NOT a citizen of
the Philippines. Accordingly, respondent's certificate of candidacy is cancelled."

The Motion for Reconsideration filed by Frivaldo remained unacted upon until after the May 8,
1995 elections. So, his candidacy continued and he was voted for during the elections held on said
date. On May 11, 1995, the Comelec en banc7 affirmed the aforementioned Resolution of the Second
Division.
The Provincial Board of Canvassers completed the canvass of the election returns and a
Certificate of Votes8.dated May 27, 1995 was issued showing the following votes obtained by the
candidates for the position of Governor of Sorsogon:

Antonio H. Escudero, Jr. 51,060

Juan G. Frivaldo 73,440

RaulR.Lee 53,304

Isagani P. Ocampo 1,925

On June 9, 1995, Lee filed in said SPA No. 95-028, a (supplemental) petition 9 praying for his
proclamation as the duly-elected Governor of Sorsogon.
In an order10 dated June 21, 1995, but promulgated according to the petition "only on June 29,
1995," the Comelec en bane directed "the Provincial Board of Canvassers of Sorsogon to reconvene
for the purpose of proclaiming candidate Raul Lee as the winning gubernatorial candidate in the
province of Sorsogon on June 29,1995 x x x." Accordingly, at 8:30 in the evening of June 30,1995, Lee
was proclaimed governor of Sorsogon.
On July 6, 1995, Frivaldo filed with the Comelec a new petition, 11 docketed as SPC No. 95-317,
praying for the annulment of the June 30, 1995proclamation of Lee and for his own proclamation. He
alleged that on June 30, 1995, at 2:00 in the afternoon, he took his oath of allegiance as a citizen of
the Philippines after "his petition for repatriation under P.D. 725 which he filed with the Special
Committee on Naturalization in September 1994 had been granted." As such, when "the said order
(dated June 21, 1995) (of the Comelec) x x x was released and received by Frivaldo on June 30, 1995
at 5:30 o'clock in the evening, there was no more legal impediment to the proclamation (of Frivaldo) as
governor x x x." In the alternative, he averred that pursuant to the two cases of Labo vs.
Comelec,12 the Vice-Governor not Lee should occupy said position of governor.
On December 19, 1995, the Comelec First Division promulgated the herein assailed
Resolution13 holding that Lee, "not having garnered the highest number of votes," was not legally
entitled to be proclaimed as duly-elected governor; and that Frivaldo, "having garnered the highest
number of votes, and xxx having reacquired his Filipino citizenship by repatriation on June 30, 1995
under the provisions of Presidential Decree No. 725 xxx (is) qualified to hold the office of governor of
Sorsogon"; thus:

"PREMISES CONSIDERED, the Commission (First Division), therefore RESOLVES to GRANT the
Petition.

Consistent with the decisions of the Supreme Court, the proclamation of Raul R. Lee as Governor of
Sorsogon is hereby ordered annulled, being contrary to law, he not having garnered the highest
number of votes to warrant his proclamation.

Upon the finality of the annulment of the proclamation of Raul R. Lee, the Provincial Board of
Canvassers is directed to immediately reconvene and, on the basis of the completed canvass,
proclaim petitioner Juan G. Frivaldo as the duly elected Governor of Sorsogon having garnered the
highest number of votes, and he having reacquired his Filipino citizenship by repatriation on June
30,1995 under the provisions of Presidential Decree No. 725 and, thus, qualified to hold the office of
Governor of Sorsogon.

Conformably with Section 260 of the Omnibus Election Code (B.P. Blg. 881), the Clerk of the
Commission is directed to notify His Excellency the President of the Philippines, and the Secretary of
the Sangguniang Panlalawigan of the Province of Sorsogon of this resolution immediately upon the
due implementation thereof."

On December 26,1995, Lee filed a motion for reconsideration which was denied by the Comelec en
banc in its Resolution14 promulgated on February 23, 1996. On February 26, 1996, the present petition
was filed. Acting on the prayer for a temporary restraining order, this Court issued on February 27,
1996 a Resolution which inter alia directed the parties "to maintain the status quo prevailing prior to
the filing of this petition."

The Issues in G.R. No. 123755

Petitioner Lee's "position on the matter at hand briefly be capsulized in the following
propositions":15

"First - The initiatory petition below was so far insufficient in form and substance to warrant the
exercise by the COMELEC of its jurisdiction with the result that, in effect, the COMELEC acted without
jurisdiction in taking cognizance of and deciding said petition;

Second- The judicially declared disqualification of respondent was a continuing condition and rendered
him ineligible to run for, to be elected to and to hold the Office of Governor;

Third - The alleged repatriation of respondent was neither valid nor is the effect thereof retroactive as
to cure his ineligibility and qualify him to hold the Office of Governor; and

Fourth - Correctly read and applied, the Labo Doctrine fully supports the validity of petitioner's
proclamation as duly elected Governor of Sorsogon."

G.R. No. 120295

This is a petition to annul three Resolutions of the respondent Comelec, the first two of which are
also at issue in G.R. No. 123755, as follows:

1. Resolution16 of the Second Division, promulgated on May 1, 1995, disqualifying Frivaldo from
running for governor of Sorsogon in the May 8, 1995 elections "on the ground that he is not a citizen of
the Philippines";

2. Resolution17 of the Comelec en bane, promulgated on May 11, 1995; and

3. Resolution18 of the Comelec en bane, promulgated also on May 11, 1995 suspending the
proclamation of, among others, Frivaldo.

The Facts and the Issue


The facts of this case are essentially the same as those in G.R. No. 123755. However, Frivaldo
assails the above-mentioned resolutions on a different ground: that under Section 78 of the Omnibus
Election Code, which is reproduced hereinunder:

"Section 78. Petition to deny due course or to cancel a certificate of candidacy. A verified petition
seeking to deny due course or to cancel a certificate of candidacy may be filed by any person
exclusively on the ground that any material representation contained therein as required under Section
74 hereof is false. The petition may be filed at any time not later than twenty-five days from the time of
the filing of the certificate of candidacy and shall be decided, after notice and hearing, not later than
fifteen days before the election." (Italics supplied.)

the Comelec had no jurisdiction to issue said Resolutions because they were not rendered "within the
period allowed by law," i.e., "not later than fifteen days before the election."
Otherwise stated, Frivaldo contends that the failure of the Comelec to act on the petition for
disqualification within the period of fifteen days prior to the election as provided by law is a
jurisdictional defect which renders the said Resolutions null and void.
By Resolution on March 12, 1996, the Court consolidated G.R. Nos. 120295 and 123755 since
they are intimately related in their factual environment and are identical in the ultimate question
raised, viz., who should occupy the position of governor of the province of Sorsogon.
On March 19, 1995, the Court heard oral argument from the parties and required them thereafter
to file simultaneously their respective memoranda.

The Consolidated Issues

From the foregoing submissions, the consolidated issues may be restated as follows:

1. Was the repatriation of Frivaldo valid and legal? If so, did it seasonably cure his lack of citizenship
as to qualify him to be proclaimed and to hold the Office of Governor? If not, may it be given
retroactive effect? If so, from when?

2. Is Frivaldo's "judicially declared" disqualification for lack of Filipino citizenship a continuing bar to his
eligibility to run for, be elected to or hold the governorship of Sorsogon?

3. Did the respondent Comelec have jurisdiction over the initiatory petition in SPC No. 95-317
considering that : said petition is not "a pre-proclamation case, an election protest or a quo
warranto case"?

4. Was the proclamation of Lee, a runner-up in the election, valid and legal in light of existing
jurisprudence?

5. Did the respondent Commission on Elections exceed its jurisdiction in promulgating the assailed
Resolutions, all of which prevented Frivaldo from assuming the governorship of Sorsogon, considering
that they were not rendered within ( the period referred to in Section 78 of the Omnibus Election
Code, viz., "not later than fifteen days before the elections"?

The First Issue: Frivaldo's Repatriation

The validity and effectivity of Frivaldo's repatriation is the lis mota, the threshold legal issue in this
case. All the other matters raised are secondary to this.
The Local Government Code of 1991 19 expressly requires Philippine citizenship as a qualification
for elective local officials, including that of provincial governor, thus:
"Sec. 39. Qualifications. (a) An elective local official must be a citizen of the Philippines; a registered
voter in the barangay, municipality, city, or province or, in the case of a member of the sangguniang
panlalawigan, sangguniang panlungsod, or sangguniang bayan, the district where he intends to be
elected; a resident therein for at least one (1) year immediately preceding the day of the election; and
able to read and write Filipino or any other local language or dialect.

(b) Candidates for the position of governor, vice governor or member of the sangguniang
panlalawigan, or mayor, vice mayor or member of the sangguniang panlungsod of highly urbanized
cities must be at least twenty-three (23) years of age on election day.

xxx xxx xxx


Inasmuch as Frivaldo had been declared by this Court 20 as a non-citizen, it is therefore incumbent
upon him to show that he has reacquired citizenship; in fine, that he possesses the qualifications
prescribed under the said statute (R. A. 7160).
Under Philippine law,21 citizenship may be reacquired by direct act of Congress, by naturalization
or by repatriation. Frivaldo told this Court in G.R. No. 104654 22 and during the oral argument in this
case that he tried to resume his citizenship by direct act of Congress, but that the bill allowing him to
do so "failed to materialize, notwithstanding the endorsement of several members of the House of
Representatives" due, according to him, to the "maneuvers of his political rivals." In the same case, his
attempt at naturalization was rejected by this Court because of jurisdictional, substantial and
procedural defects.
Despite his lack of Philippine citizenship, Frivaldo was overwhelmingly elected governor by the
electorate of Sorsogon, with a margin of 27,000 votes in the 1988 elections, 57,000 in 1992, and
20,000 in 1995 over the same opponent Raul Lee. Twice, he was judicially declared a non-Filipino and
thus twice disqualified from holding and discharging his popular mandate. Now, he comes to us a third
time, with a fresh vote from the people of Sorsogon and a favorable decision from the Commission on
Elections to boot. Moreover, he now boasts of having successfully passed through the third and last
mode of reacquiring citizenship: by repatriation under P.D. No. 725, with no less than the Solicitor
General himself, who was the prime opposing counsel in the previous cases he lost, this time, as
counsel for co-respondent Comelec, arguing the validity of his cause (in addition to his able private
counsel Sixto S. Brillantes, Jr.). That he took his oath of allegiance under the provisions of said Decree
at 2:00 p.m. on June 30, 1995 is not disputed. Hence, he insists that henot Leeshould have been
proclaimed as the duly-elected governor of Sorsogon when the Provincial Board of Canvassers met at
8:30 p.m. on the said date since, clearly and unquestionably, he garnered the highest number of votes
in the elections and since at that time, he already reacquired his citizenship.
En contrario, Lee argues that Frivaldo's repatriation is tainted ; with serious defects, which we
shall now discuss in seriatim.
First, Lee tells us that P.D. No. 725 had "been effectively repealed," asserting that "then President
Corazon Aquino exercising legislative powers under the Transitory Provisions of the 1987 Constitution,
forbade the grant of citizenship by Presidential Decree or Executive Issuances as the same poses a
serious and contentious issue of policy which the present government, in the exercise of prudence and
sound discretion, should best leave to the judgment of the first Congress under the 1987 Constitution,"
adding that in her memorandum dated March 27,1987 to the members of the Special Committee on
Naturalization constituted for purposes of Presidential Decree No. 725, President Aquino directed
them "to cease and desist from undertaking any and all proceedings within your functional area of
responsibility as defined under Letter of Instructions (LOI) No. 270 dated April 11, 1975, as
amended."23
This memorandum dated March 27, 198724 cannot by any stretch of legal hermeneutics be
construed as a law sanctioning or authorizing a repeal of P.D. No. 725. Laws are repealed only by
subsequent ones25 and a repeal may be express or implied. It is obvious that no express repeal
was made because then President Aquino in her memorandum based on the copy furnished us by
Lee did not categorically and/or impliedly state that P.D. 725 was being repealed or was being
rendered without any legal effect. In fact, she did not even mention it specifically by its number or text.
On the other hand, it is a basic rule of statutory construction that repeals by implication are not
favored. An implied repeal will not be allowed "unless it is convincingly and unambiguously
demonstrated that the two laws are clearly repugnant and patently inconsistent that they cannot co-
exist."26
The memorandum of then President Aquino cannot even be regarded as a legislative enactment,
for not every pronouncement of the Chief Executive even under the Transitory Provisions of the 1987
Constitution can nor should be regarded as an exercise of her law-making powers. At best, it could be
treated as an executive policy addressed to the Special Committee to halt the acceptance and
processing of applications for repatriation pending whatever "judgment the first Congress under the
1987 Constitution" might make. In other words, the former President did not repeal P.D. 725 but left it
to the first Congress once createdto deal with the matter. If she had intended to repeal such law, she
should have unequivocally said so instead of referring the matter to Congress. The fact is she carefully
couched her presidential issuance in terms that clearly indicated the intention of "the present
government, in the exercise of prudence and sound discretion" to leave the matter of repeal to the new
Congress. Any other interpretation of the said Presidential Memorandum, such as is now being
proffered to the Court by Lee, would visit unmitigated violence not only upon statutory construction but
on common sense as well.
Second. Lee also argues that "serious congenital irregularities flawed the repatriation
proceedings," asserting that Frivaldo's application therefor was "filed on June 29, 1995 x x x (and) was
approved in just one day or on June 30, 1995 x x x," which "prevented a judicious review and
evaluation of the merits thereof." Frivaldo counters that he filed his application for repatriation with the
Office of the President in Malacanang Palace on August 17, 1994. This is confirmed by the Solicitor
General. However, the Special Committee was reactivated only on June 8, 1995, when presumably
the said Committee started processing his application. On June 29, 1995, he filled up and re-submitted
the FORM that the Committee required. Under these circumstances, it could not be said that there
was "indecent haste" in the processing of his application.
Anent Lee's charge that the "sudden reconstitution of the Special Committee on Naturalization
was intended solely for the personal interest of respondent," 27 the Solicitor General explained during
the oral argument on March 19, 1996 that such allegation is simply baseless as there were many
others who applied and were considered for repatriation, a list of whom was submitted by him to this
Court, through a Manifestation28 filed on April 3, 1996.
On the basis of the parties' submissions, we are convinced that the presumption of regularity in
the performance of official duty and the presumption of legality in the repatriation of Frivaldo have not
been successfully rebutted by Lee. The mere fact that the proceedings were speeded up is by itself
not a ground to conclude that such proceedings were necessarily tainted. After all, the requirements of
repatriation under P.D. No. 725 are not difficult to comply with, nor are they tedious and cumbersome.
In fact, P.D. 72529 itself requires very little of an applicant, and even the rules and regulations to
implement the said decree were left to the Special Committee to promulgate. This is not unusual
since, unlike in naturalization where an alien covets a first-time entry into Philippine political life, in
repatriation the applicant is a former natural-born Filipino who is merely seeking to reacquire his
previous citizenship. In the case of Frivaldo, he was undoubtedly a natural-born citizen who openly
and faithfully served his country and his province prior to his naturalization in the United States a
naturalization he insists was made necessary only to escape the iron clutches of a dictatorship he
abhorred and could not in conscience embrace and who, after the fall of the dictator and the re-
establishment of democratic space, wasted no time in returning to his country of birth to offer once
more his talent and services to his people.
So too, the fact that ten other persons, as certified to by the Solicitor General, were granted
repatriation argues convincingly and conclusively against the existence of favoritism vehemently
posited by Raul Lee. At any rate, any contest on the legality of Frivaldo's repatriation should have
been pursued before the Committee itself, and, failing there, in the Office of the President, pursuant to
the doctrine of exhaustion of administrative remedies.
Third. Lee further contends that assuming the assailed repatriation to be valid, nevertheless it
could only be effective as at 2:00 p.m. of June 30, 1995 whereas the citizenship qualification
prescribed by the Local Government Code "must exist on the date of his election, if not when the
certificate of candidacy is filed," citing our decision in G.R. 104654 30 which held that "both the Local
Government Code and the Constitution require that only Philippine citizens can run and be elected to
Public office" Obviously, however, this was a mere obiter as the only issue in said case was whether
Frivaldo's naturalization was valid or not and NOT the effective date thereof. Since the Court held his
naturalization to be invalid, then the issue of when an aspirant for public office should be a citizen was
NOT resolved at all by the Court. Which question we shall now directly rule on.
Under Sec. 39 of the Local Government Code, "(a)n elective local official must be:
* a citizen of the Philippines;
* a registered voter in the barangay, municipality, city, or province x x x where he intends to
be elected;
* a resident therein for at least one (1) year immediately preceding the day of the election;
* able to read and write Filipino or any other local language or dialect."
* In addition, "candidates for the position of governor x x x must be at least twenty-three (23)
years of age on election day."
From the above, it will be noted that the law does not specify any particular date or time when the
candidate must possess citizenship, unlike that for residence (which must consist of at least one
year's residency immediately preceding the day of election) and age (at least twenty three years of
age on election day).
Philippine citizenship is an indispensable requirement for holding an elective public office, 31 and
the purpose of the citizenship qualification is none other than to ensure that no alien, i.e., no person
owing allegiance to another nation, shall govern our people and our country or a unit of territory
thereof. Now, an official begins to govern or to discharge his functions only upon his proclamation and
on the day the law mandates his term of office to begin. Since Frivaldo re-assumed his citizenship on
June 30, 1995the very day32 the term of office of governor (and other elective officials) beganhe was
therefore already qualified to be proclaimed, to hold such office and to discharge the functions and
responsibilities thereof as of said date. In short, at that time, he was already qualified to govern his
native Sorsogon. This is the liberal interpretation that should give spirit, life and meaning to our law on
qualifications consistent with the purpose for which such law was enacted. So too, even from a literal
(as distinguished from liberal) construction, it should be noted that Section 39 of the Local Government
Code speaks of "Qualifications" of "ELECTIVE OFFICIALS," not of candidates. Why then should such
qualification be required at the time of election or at the time of the filing of the certificates of
candidacies, as Lee insists? Literally, such qualifications unless otherwise expressly conditioned, as in
the case of age and residence should thus be possessed when the "elective [or elected] official"
begins to govern, i.e., at the time he is proclaimed and at the start of his term in this case, on June 30,
1995. Paraphrasing this Court's ruling in Vasquez vs. Giapand Li Seng Giap & Sons,33 if the purpose
of the citizenship requirement is to ensure that our people and country do not end up being governed
by aliens, i.e., persons owing allegiance to another nation, that aim or purpose would not be thwarted
but instead achieved by construing the citizenship qualification as applying to the time of proclamation
of the elected official and at the start of his term.
But perhaps the more difficult objection was the one raised during the oral argument 34 to the
effect that the citizenship qualification should be possessed at the time the candidate (or for that
matter the elected official) registered as a voter. After all, Section 39, apart from requiring the official to
be a citizen, also specifies as another item of qualification, that he be a "registered voter." And, under
the law35 a "voter" must be a citizen of the Philippines. So therefore, Frivaldo could not have been a
voter-much less a validly registered one if he was not a citizen at the time of such registration.
The answer to this problem again lies in discerning the purpose of the requirement. If the law
intended the citizenship qualification to be possessed prior to election consistent with the requirement
of being a registered voter, then it would not have made citizenship a SEPARATE qualification. The
law abhors a redundancy. It therefore stands to reason that the law intended CITIZENSHIP to be a
qualification distinct from being a VOTER, even if being a voter presumes being a citizen first. It also
stands to reason that the voter requirement was included as another qualification (aside from
"citizenship"), not to reiterate the need for nationality but to require that the official be registered as a
voter IN THE AREA OR TERRITORY he seeks to govern, i.e., the law states: "a registered voter in the
barangay, municipality, city, or province x x x where he intends to be elected." It should be emphasized
that the Local Government Code requires an elective official to be a registered voter. It does not
require him to vote actually. Hence, registrationnot the actual votingis the core of this "qualification." In
other words, the law's purpose in this second requirement is to ensure that the prospective official is
actually registered in the area he seeks to govern and not anywhere else.
Before this Court, Frivaldo has repeatedly emphasizedand Lee has not disputed that he "was and
is a registered voter of Sorsogon, and his registration as a voter has been sustained as valid by
judicial declaration x x x In fact, he cast his vote in his precinct on May 8, 1995."36
So too, during the oral argument, his counsel stead-fastly maintained that "Mr. Frivaldo has
always been a registered voter of Sorsogon. He has voted in 1987,1988,1992, then he voted again in
1995. In fact, his eligibility as a voter was questioned, but the court dismissed (sic) his eligibility as a
voter and he was allowed to vote as in fact, he voted in all the previous elections including on May
8,1995.37
It is thus clear that Frivaldo is a registered voter in the province where he intended to be elected.
There is yet another reason why the prime issue of citizenship should be reckoned from the date
of proclamation, not necessarily the date of election or date of filing of the certificate of candidacy.
Section 253 of the Omnibus Election Code 38 gives any voter, presumably including the defeated
candidate, the opportunity to question the ELIGIBILITY (or the disloyalty) of a candidate. This is the
only provision of the Code that authorizes a remedy on how to contest before the Comelec an
incumbent's ineligibility arising from failure to meet the qualifications enumerated under Sec. 39 of the
Local Government Code. Such remedy of Quo Warranto can be availed of "within ten days after
proclamation" of the winning candidate. Hence, it is only at such time that the issue of ineligibility may
be taken cognizance of by the Commission. And since, at the very moment of Lee's proclamation
(8:30 p.m., June 30, 1995), Juan G. Frivaldo was already and indubitably a citizen, having taken his
oath of allegiance earlier in the afternoon of the same day, then he should have been the candidate
proclaimed as he unquestionably garnered the highest number of votes in the immediately preceding
elections and such oath had already cured his previous "judicially-declared" alienage. Hence, at such
time, he was no longer ineligible.
But to remove all doubts on this important issue, we also hold that the repatriation of Frivaldo
RETRO ACTED to the date of the filing of his application on August 17,1994.
It is true that under the Civil Code of the Philippines,39 "(l)aws shall have no retroactive effect,
unless the contrary is provided." But there are settled exceptions 40 to this general rule, such as when
the statute is CURATIVE or REMEDIAL in nature or when it CREATES NEW RIGHTS.
According to Tolentino,41 curative statutes are those which undertake to cure errors and
irregularities, thereby validating judicial or administrative proceedings, acts of public officers, or private
deeds and contracts which otherwise would not produce their intended consequences by reason of
some statutory disability or failure to comply with some technical requirement. They operate on
conditions already existing, and are necessarily retroactive in operation. Agpalo, 42 on the other hand,
says that curative statutes are "healing acts x x x curing defects and adding to the means of enforcing
existing obligations x x x (and) are intended to supply defects, abridge superfluities in existing laws,
and curb certain evils x x x By their very nature, curative statutes are retroactive xxx (and) reach back
to past events to correct errors or irregularities and to render valid and effective attempted acts which
would be otherwise ineffective for the purpose the parties intended."
On the other hand, remedial or procedural laws, i.e., those statutes relating to remedies or modes
of procedure, which do not create new or take away vested rights, but only operate in furtherance of
the remedy or confirmation of such rights, ordinarily do not come within the legal meaning of a
retrospective law, nor within the general rule against the retrospective operation of statutes. 43
A reading of P.D. 725 immediately shows that it creates a new right, and also provides for a new
remedy, thereby filling certain voids in our laws. Thus, in its preamble, P.D. 725 expressly recognizes
the plight of "many Filipino women (who) had lost their Philippine citizenship by marriage to aliens"
and who could not, under the existing law (C. A. No. 63, as amended) avail of repatriation until "after
the death of their husbands or the termination of their marital status" and who could neither be
benefitted by the 1973 Constitution's new provision allowing "a Filipino woman who marries an alien to
retain her Philippine citizenship xxx" because "such provision of the new Constitution does not apply to
Filipino women who had married aliens before said constitution took effect." Thus, P.D. 725 granted a
new right to these womenthe right to re-acquire Filipino citizenship even during their marital coverture,
which right did not exist prior to P.D. 725. On the other hand, said statute also provided a new remedy
and a new right in favor of other "natural born Filipinos who (had) lost their Philippine citizenship but
now desire to re-acquire Philippine citizenship," because prior to the promulgation of P.D. 725 such
former Filipinos would have had to undergo the tedious and cumbersome process of naturalization,
but with the advent of P.D. 725 they could now re-acquire their Philippine citizenship under the
simplified procedure of repatriation.
The Solicitor General44 argues:

"By their very nature, curative statutes are retroactive, (DBP vs. CA, 96 SCRA 342), since they are
intended to supply defects, abridge superfluities in existing laws (Del Castillo vs. Securities and
Exchange Commission, 96 Phil. 119) and curb certain evils (Santos vs. Duata, 14 SCRA 1041).

In this case, P.D. No. 725 was enacted to cure the defect in the existing naturalization law, specifically
C. A. No. 63 wherein married Filipino women are allowed to repatriate only upon the death of their
husbands, and natural-born Filipinos who lost their citizenship by naturalization and other causes
faced the difficulty of undergoing the rigid procedures of C.A. 63 for reacquisition of Filipino citizenship
by naturalization.

Presidential Decree No. 725 provided a remedy for the aforementioned legal aberrations and thus its
provisions are considered essentially remedial and curative."

In light of the foregoing, and prescinding from the wording of the preamble, it is unarguable that
the legislative intent was precisely to give the statute retroactive operation. "(A) retrospective operation
is given to a statute or amendment where the intent that it should so operate clearly appears from a
consideration of the act as a whole, or from the terms thereof." 45 It is obvious to the Court that the
statute was meant to "reach back" to those persons, events and transactions not otherwise covered by
prevailing law and jurisprudence. And inasmuch as it has been held that citizenship is a political and
civil right equally as important as the freedom of speech, liberty of abode, the right against
unreasonable searches and seizures and other guarantees enshrined in the Bill of Rights, therefore
the legislative intent to give retrospective operation to P.D. 725 must be given the fullest effect
possible. "(I)t has been said that a remedial statute must be so construed as to make it effect the
evident purpose for -which it was enacted, so that if the reason of the statute extends to past
transactions, as well as to those in the future, then it will be so applied although the statute does not in
terms so direct, unless to do so would impair some vested right or violate some constitutional
guaranty."46 This is all the more true of P.D. 725, which did not specify any restrictions on or delimit or
qualify the right of repatriation granted therein.
At this point, a valid question may be raised: How can the retroactivity of P.D. 725 benefit Frivaldo
considering that said law was enacted on June 5,1975, while Frivaldo lost his Filipino citizenship much
later, on January 20, 1983, and applied for repatriation even later, on August 17, 1994?
While it is true that the law was already in effect at the time that Frivaldo became an American
citizen, nevertheless, it is not only the law itself (P.D. 725) which is tobe given retroactive effect, but
even the repatriation granted under said law to Frivaldo on June 30, 1995 is to be deemed to have
retroacted to the date of his application therefor, August 17, 1994. The reason for this is simply that if,
as in this case, it was the intent of the legislative authority that the law should apply to past events i.e.,
situations and transactions existing even before the law came into being in order to benefit the
greatest number of former Filipinos possible thereby enabling them to enjoy and exercise the
constitutionally guaranteed right of citizenship, and such legislative intention is to be given the fullest
effect and expression, then there is all the more reason to have the law apply in a retroactive or
retrospective manner to situations, events and transactions subsequent to the passage of such
law. That is, the repatriation granted to Frivaldo on June 30, 1995 can and should be made to take
effect as of date of his application. As earlier mentioned, there is nothing in the law that would bar this
or would show a contrary intention on the part of the legislative authority; and there is no showing that
damage or prejudice to anyone, or anything unjust or injurious would result from giving retroactivity to
his repatriation. Neither has Lee shown that there will result the impairment of any contractual
obligation, disturbance of any vested right or breach of some constitutional guaranty.
Being a former Filipino who has served the people repeatedly, Frivaldo deserves a liberal
interpretation of Philippine laws and whatever defects there were in his nationality should now be
deemed mooted by his repatriation.
Another argument for retroactivity to the date of filing is that it would prevent prejudice to
applicants. If P.D. 725 were not to be given retroactive effect, and the Special Committee decides not
to act, i.e., to delay the processing of applications for any substantial length of time, then the former
Filipinos who may be stateless, as Frivaldohaving already renounced his American citizenship was,
may be prejudiced for causes outside their control. This should not be. In case of doubt in the
interpretation or application of laws, it is to be presumed that the law-making body intended right and
justice to prevail.47
And as experience will show, the Special Committee was able to process, act upon and grant
applications for repatriation within relatively short spans of time after the same were filed. 48 The fact
that such interregna were relatively insignificant minimizes the likelihood of prejudice to the
government as a result of giving retroactivity to repatriation. Besides, to the mind of the Court, direct
prejudice to the government is possible only where a person's repatriation has the effect of wiping out
a liability of his to the government arising in connection with or as a result of his being an alien, and
accruing only during the interregnum between application and approval, a situation that is not present
in the instant case.
And it is but right and just that the mandate of the people, already twice frustrated, should now
prevail. Under the circumstances, there is nothing unjust or iniquitous in treating Frivaldo's repatriation
as having become effective as of the date of his application, i.e., on August 17, 1994. This being so, all
questions about his possession of the nationality qualification whether at the date of proclamation
(June 30, 1995) or the date of election (May 8, 1995) or date of filing his certificate of candidacy
(March 20, 1995) would become moot.

Based on the foregoing, any question regarding Frivaldo's status as a registered voter would also be
deemed settled. Inasmuch as he is considered as having been repatriatedi.e., his Filipino citizenship
restored as of August 17, 1994, his previous registration as a voter is likewise deemed validated as of
said date.

It is not disputed that on January 20, 1983 Frivaldo became an American. Would the retroactivity of his
repatriation not effectively give him dual citizenship, which under Sec. 40 of the Local Government
Code would disqualify him "from running for any elective local position?" 49 We answer this question in
the negative, as there is cogent reason to hold that Frivaldo was really STATELESS at the time he
took said oath of allegiance and even before that, when he ran for governor in 1988. In his Comment,
Frivaldo wrote that he "had long renounced and had long abandoned his American citizenshiplong
before May 8, 1995. At best, Frivaldo was stateless in the interim when he abandoned and renounced
his US citizenship but before he was repatriated to his Filipino citizenship." 50

On this point, we quote from the assailed Resolution dated December 19, 1995:51

"By the laws of the United States, petitioner Frivaldo lost his American citizenship when he took his
oath of allegiance to the Philippine Government when he ran for Governor in 1988, in 1992, and in
1995. Every certificate of candidacy contains an oath of allegiance to the Philippine Government."

These factual findings that Frivaldo has lost his foreign nationality long before the elections of
1995 have not been effectively rebutted by Lee. Furthermore, it is basic that such findings of the
Commission are conclusive upon this Court, absent any showing of capriciousness or arbitrariness or
abuse.52

The Second Issue: Is Lack of Citizenship a Continuing Disqualification?

Lee contends that the May 1,1995 Resolution 53 of the Comelec Second Division in SPA No. 95-
028 as affirmed in toto by Comelec En Banc in its Resolution of May 11, 1995 "became final and
executory after five (5) days or on May 17,1995, no restraining order having been issued by this
Honorable Court."54 Hence, before Lee "was proclaimed as the elected governor on June 30, 1995,
there was already a final and executory judgment disqualifying" Frivaldo. Lee adds that this Court's
two rulings (which Frivaldo now concedes were legally "correct") declaring Frivaldo an alien have also
become final and executory way before the 1995 elections, and these "judicial pronouncements of his
political status as an American citizen absolutely and for all time disqualified (him) from running for,
and holding any public office in the Philippines."
We do not agree.
It should be noted that our first ruling in G.R. No. 87193 disqualifying Frivaldo was rendered in
connection with the 1988 elections while that in G.R. No. 104654 was in connection with the 1992
elections. That he was disqualified for such elections is final and can no longer be changed. In the
words of the respondent Commission (Second Division) in its assailed Resolution: 55

"The records show that the Honorable Supreme Court had decided that Frivaldo was not a Filipino
citizen and thus disqualified for the purpose of the 1988 and 1992 elections. However, there is no
record of any 'final judgment' of the disqualification of Frivaldo as a candidate for the May 8,
1995 elections. What the Commission said in its Order of June 21, 1995 (implemented on June 30,
1995), directing the proclamation of Raul R. Lee, was that Frivaldo was not a Filipino citizen 'having
been declared by the Supreme Court in its Order dated March 25, 1995, not a citizen of the
Philippines.' This declaration of the Supreme Court, however, was in connection with the 1992
elections."

Indeed, decisions declaring the acquisition or denial of citizenship cannot govern a person's
future status with finality. This is because a person may subsequently reacquire, or for that matter
lose, his citizenship under any of the modes recognized by law for the purpose. Hence, in Lee vs.
Commissioner of Immigration,56 we held:

"Everytime the citizenship of a person is material or indispensable in a judicial or administrative case,


whatever the corresponding court or administrative authority decides therein as to such citizenship is
generally not considered res judicata, hence it has to be threshed out again and again, as the
occasion demands."

The Third Issue: Comelec's Jurisdiction


Over The Petition in SPC No. 95-317
Lee also avers that respondent Comelec had no jurisdiction to entertain the petition in SPC No.
95-317 because the only "possible types of proceedings that may be entertained by the Comelec are a
pre-proclamation case, an election protest or a quo warranto case." Again, Lee reminds us that he was
proclaimed on June 30, 1995 but that Frivaldo filed SPC No. 95-317 questioning his (Lee's)
proclamation only on July 6, 1995 "beyond the 5-day reglementary period." Hence, according to him,
Frivaldo's "recourse was to file either an election protest or a quo warranto action."
This argument is not meritorious. The Constitution 57 has given the Comelec ample power to
"exercise exclusive original jurisdiction over all contests relating to the elections, returns and
qualifications of all elective x x x provincial x x x officials." Instead of dwelling at length on the various
petitions that Comelec, in the exercise of its constitutional prerogatives, may entertain, suffice
it to say that this Court has invariably recognized the Commission's authority to hear and decide
petitions for annulment of proclamations of which SPC No. 95-317 obviously is one. 58 Thus, in
Mentang vs. COMELEC,59 we ruled:

"The petitioner argues that after proclamation and assumption of office, a pre-proclamation
controversy is no longer viable. Indeed, we are aware of cases holding that pre-proclamation
controversies may no longer be entertained by the COMELEC after the winning candidate has been
proclaimed, (citing Gallardo vs. Rimando, 187 SCRA 463; Salvacion vs. COMELEC, 170 SCRA 513;
Casimiro vs. COMELEC, 171 SCRA 468.) This rule, however, is premised on an assumption that the
proclamation is no proclamation at all and the proclaimed candidate's assumption of office cannot
deprive the COMELEC of the power to make such declaration of nullity. (citing Aguam vs. COMELEC,
23 SCRA 883; Agbayani vs. COMELEC, 186 SCRA 484.)"

The Court however cautioned that such power to annul a proclamation must "be done within ten
(10) days following the proclamation." Inasmuch as Frivaldo's petition was filed only six (6) days after
Lee's proclamation, there is no question that the Comelec correctly acquired jurisdiction over the
same.
The Fourth Issue: Was Lee's Proclamation Valid

Frivaldo assails the validity of the Lee proclamation. We uphold him for the following reasons:

First. To paraphrase this Court in Labo vs. COMELEC,60 "the fact remains that he (Lee) was not the
choice of the sovereign will," and in Aquino vs. COMELEC,61Lee is "a second placer, xxx just that, a
second placer."

In spite of this, Lee anchors his claim to the governorship on the pronouncement of this Court in
the aforesaid Labo62 case, as follows:

"The rule would have been different if the electorate fully aware in fact and in law of a candidate's
disqualification so as to bring such awareness within the realm of notoriety, would nonetheless cast
their votes in favor of the ineligible candidate. In such case, the electorate may be said to have waived
the validity and efficacy of their votes by notoriously misapplying their franchise or throwing away their
votes, in which case, the eligible candidate obtaining the next higher number of votes may be deemed
elected."

But such holding is qualified by the next paragraph, thus:

"But this is not the situation obtaining in the instant dispute. It has not been shown, and none was
alleged, that petitioner Labo was notoriously known as an ineligible candidate, much less the
electorate as having known of such fact. On the contrary, petitioner Labo was even allowed by no less
than the Comelec itself in its resolution dated May 10, 1992 to be voted for the office of the city mayor
as its resolution dated May 9,1992 denying due course to petitioner Labo's certificate of candidacy had
not yet become final and subject to the final outcome of this case."

The last-quoted paragraph in Labo, unfortunately for Lee, is the ruling appropriate in this case
because Frivaldo was in 1995 in an identical situation as Labo was in 1992 when the Comelec's
cancellation of his certificate of candidacy was not yet final on election day as there was in both cases
a pending motion for reconsideration, for which reason Comelec issued an (omnibus) resolution
declaring that Frivaldo (like Labo in 1992) and several others can still be voted for in the May 8, 1995
election, as in fact, he was.
Furthermore, there has been no sufficient evidence presented to show that the electorate of
Sorsogon was "fully aware in fact and in law" of Frivaldo's alleged disqualification as to "bring such
awareness within the realm of notoriety", in other words, that the voters intentionally wasted their
ballots knowing that, in spite of their voting for him, he was ineligible. If Labo has any relevance at all,
it is that the vice-governor and not Leeshould be proclaimed, since in losing the election, Lee was, to
paraphrase Labo again, "obviously not the choice of the people" of Sorsogon. This is the emphatic
teaching of Labo:

"The rule, therefore, is: the ineligibility of a candidate receiving majority votes does not entitle the
eligible candidate receiving the next highest number of votes to be declared elected. A minority or
defeated candidate cannot be deemed elected to the office."

Second. As we have earlier declared Frivaldo to have seasonably re-acquired his citizenship and
inasmuch as he obtained the highest number of votes in the 1995 elections, henot Lee should be
proclaimed. Hence, Lee's proclamation was patently erroneous and should now be corrected.

The Fifth Issue: Is Section 78 of the Election Code Mandatory?

In G.R. No. 120295, Frivaldo claims that the assailed Resolution of the Comelec (Second
Division) dated May 1, 1995 and the confirmatory en banc Resolution of May 11, 1995 disqualifying
him for want of citizenship should be annulled because they were rendered beyond the fifteen (15) day
period prescribed by Section 78 of the Omnibus Election Code which reads as follows:
"Section 78. Petition to deny due course or to cancel a certificate of candidacy. A verified petition
seeking to deny due course or to cancel a certificate of candidacy may be filed by any person
exclusively on the ground that any material representation contained therein as required under Section
74 hereof is false. The petition may be filed at any time not later than twenty-five days from the time of
the filing of the certificate of candidacy and shall be decided after notice and hearing, not later than
fifteen days before the election" (italics supplied.)

This claim is now moot and academic inasmuch as these resolutions are deemed superseded by
the subsequent ones issued by the Commission (First Division) on December 19, 1995, affirmed en
banc63 on February 23, 1996, which both upheld his election. At any rate, it is obvious that Section 78
is merely directory as Section 6 of R.A. No. 6646 authorizes the Commission to try and decide
petitions for disqualifications even after the elections, thus:

"SEC. 6. Effect of Disqualification Case. Any candidate who has been declared by final judgment to be
disqualified shall not be voted for, and the votes cast for him shall not be counted. If for any reason a
candidate is not declared by final judgment before an election to be disqualified and he is voted for
and receives the -winning number of votes in such election, the Court or Commission shall continue
with the trial and hearing of the action, inquiry or protest and, upon motion of the complainant or any
intervenor, may during the pendency thereof order the suspension of the proclamation of such
candidate whenever the evidence of his guilt is strong." (Italics supplied)

Refutation of Mr. Justice Davide's Dissent

In his dissenting opinion, the esteemed Mr. Justice Hilario G. Davide, Jr. argues that President
Aquino's memorandum dated March 27, 1987 should be viewed as a suspension (not a repeal, as
urged by Lee) of P.D. 725. But whether it decrees a suspension or a repeal is a purely academic
distinction because the said issuance is not a statute that can amend or abrogate an existing law. The
existence and subsistence of P.D. 725 were recognized in the first Frivaldo case; 64 viz, "(u)nder CA
No. 63 as amended by CA No. 473 and P.D. No. 725, Philippine citizenship maybe reacquired by xxx
repatriation" He also contends that by allowing Frivaldo to register and to remain as a registered voter,
the Comelec and in effect this Court abetted a "mockery" of our two previous judgments declaring him
a non-citizen. We do not see such abetting or mockery. The retroactivity of his repatriation, as
discussed earlier, legally cured whatever defects there may have been in his registration as a voter for
the purpose of the 1995 elections. Such retroactivity did not change his disqualifications in 1988 and
1992, which were the subjects of such previous rulings.
Mr. Justice Davide also believes that Quo Warranto is not the sole remedy to question the
ineligibility of a candidate, citing the Comelec's authority under Section 78 of the Omnibus Election
Code allowing the denial of a certificate of candidacy on the ground of a false material representation
therein as required by Section 74. Citing Loong, he then states his disagreement with our holding that
Section 78 is merely directory. We really have no quarrel. Our point is that Frivaldo was in error in his
claim in G.R. No. 120295 that the Comelec Resolutions promulgated on May 1, 1995 and May 11,
1995 were invalid because they were issued "not later than fifteen days before the election" as
prescribed by Section 78. In dismissing the petition in G.R. No. 120295, we hold that the Comelec did
not commit grave abuse of discretion because "Section 6 of R. A. 6646 authorizes the Comelec to try
and decide disqualifications even after the elections." In spite of his disagreement with us on this
point, i.e., that Section 78 "is merely directory," we note that just like us, Mr. Justice Davide
nonetheless votes to "DISMISS G.R. No. 120295." One other point. Loong, as quoted in the dissent,
teaches that a petition to deny due course under Section 78 must be filed within the 25-day period
prescribed therein. The present case however deals with the period during which the Comelec
may decide such petition. And we hold that it may be decided even after the fifteen day period
mentioned in Section 78. Here, we rule that a decision promulgated by the Comelec even after the
elections is valid but Loong held that a petition filed beyond the 25-day period is out of time. There is
no inconsistency nor conflict.
Mr. Justice Davide also disagrees with the Court's holding that, given the unique factual
circumstances of Frivaldo, repatriation may be given retroactive effect. He argues that such
retroactivity "dilutes" our holding in the first Frivaldo case. But the first (and even the second Frivaldo)
decision did not directly involve repatriation as a mode of acquiring citizenship. If we may repeat, there
is no question that Frivaldo was not a Filipino for purposes of determining his qualifications in the 1988
and 1992 elections. That is settled. But his supervening repatriation has changed his political status
not in 1988 or 1992, but only in the 1995 elections.
Our learned colleague also disputes our holding that Frivaldo was stateless prior to his
repatriation, saying that "informal renunciation or abandonment is not a ground to lose American
citizenship." Since our courts are charged only with the duty of the determining who are Philippine
nationals, we cannot rule on the legal question of who are or who are not Americans. It is basic in
international law that a State determines ONLY those who are its own citizens not who are the citizens
of other countries.65 The issue here is: the Comelec made a finding of fact that Frivaldo was stateless
and such finding has not been shown by Lee to be arbitrary or whimsical. Thus, following settled case
law, such finding is binding and final.
The dissenting opinion also submits that Lee who lost by chasmic margins to Frivaldo in all three
previous elections, should be declared winner because "Frivaldo's ineligibility for being an American
was publicly known." First, there is absolutely no empirical evidence for such "public" knowledge.
Second, even if there is, such knowledge can be true post facto only of the last two previous elections.
Third, even the Comelec and now this Court were/are still deliberating on his nationality before, during
and after the 1995 elections. How then can there be such "public" knowledge?
Mr. Justice Davide submits that Section 39 of the Local Government Code refers to the
qualifications of elective local officials, i.e., candidates, and not elected officials, and that the
citizenship qualification [under par. (a) of that section] must be possessed by candidates, not merely at
the commencement of the term, but by election day at the latest. We see it differently. Section 39, par.
(a) thereof speaks of "elective local official" while par. (b) to (f) refer to "candidates." If the
qualifications under par. (a) were intended to apply to "candidates" and not elected officials, the
legislature would have said so, instead of differentiating par. (a) from the rest of the paragraphs.
Secondly, if Congress had meant that the citizenship qualification should be possessed at election day
or prior thereto, it would have specifically stated such detail, the same way it did in pars. (b) to (f) for
other qualifications of candidates for governor, mayor, etc.
Mr. Justice Davide also questions the giving of retroactive effect to Frivaldo's repatriation on the
ground, among others, that the law specifically provides that it is only after taking the oath of
allegiance that applicants shall be deemed to have reacquired Philippine citizenship. We do not
question what the provision states. We hold however that the provision should be understood
thus: that after taking the oath of allegiance the applicant is deemed to have reacquired Philippine
citizenship, which reacquisition (or repatriation) is deemed for all purposes and intents to have
retroacted to the date of his application therefor.
In any event, our "so too" argument regarding the literal meaning of the word "elective" in
reference to Section 39 of the Local Government Code, as well as regarding Mr. Justice Davide's
thesis that the very wordings of P.D. 725 suggest non-retroactivity, were already taken up rather
extensively earlier in this Decision.
Mr. Justice Davide caps his paper with a clarion call: "This Court must be the first to uphold the
Rule of Law." We agree we must all follow the rule of law. But that is NOT the issue here. The issue
is how should the law be interpreted and applied in this case so it can be followed, so it can rule!
At balance, the question really boils down to a choice of philosophy and perception of how to
interpret and apply laws relating to elections: literal or liberal; the letter or the spirit; the naked
provision or its ultimate purpose; legal syllogism or substantial justice; in isolation or in the context of
social conditions; harshly against or gently in favor of the voters' obvious choice. In applying election
laws, it would be far better to err in favor of popular sovereignty than to be right in complex but little
understood legalisms. Indeed, to inflict a thrice rejected candidate upon the electorate of Sorsogon
would constitute unmitigated judicial tyranny and an unacceptable assault upon this Court's
conscience.

EPILOGUE
In sum, we rule that the citizenship requirement in the Local Government Code is to be
possessed by an elective official at the latest as of the time he is proclaimed and at the start of the
term of office to which he has been elected. We further hold P.D. No. 725 to be in full force and effect
up to the present, not having been suspended or repealed expressly nor impliedly at any time, and
Frivaldo's repatriation by virtue thereof to have been properly granted and thus valid and effective.
Moreover, by reason of the remedial or curative nature of the law granting him a new right to resume
his political status and the legislative intent behind it, as well as his unique situation of having been
forced to give up his citizenship and political aspiration as his means of escaping a regime he
abhorred, his repatriation is to be given retroactive effect as of the date of his application therefor,
during the pendency of which he was stateless, he having given ' up his U. S. nationality. Thus, in
contemplation of law, he possessed the vital requirement of Filipino citizenship as of the start of the
term of office of governor, and should have been proclaimed instead of Lee. Furthermore, since his
reacquisition of citizenship retroacted to August 17, 1994, his registration as a voter of Sorsogon is
deemed to have been validated as of said date as well. The foregoing, of course, are precisely
consistent with our holding that lack of the citizenship requirement is not a continuing disability or
disqualification to run for and hold public office. And once again, we emphasize herein our previous
rulings recognizing the Comelec's authority and jurisdiction to hear and decide petitions for annulment
of proclamations.
This Court has time and again liberally and equitably construed the electoral laws of our country
to give fullest effect to the manifest will of our people, 66 for in case of doubt, political laws must be
interpreted to give life and spirit to the popular mandate freely expressed through the ballot.
Otherwise stated, legal niceties and technicalities cannot stand in the way of the sovereign will.
Consistently, we have held:

"x x x (L)aws governing election contests must be liberally construed to the end that the will of the
people in the choice of public officials may not be defeated by mere technical objections (citations
omitted)."67

The law and the courts must accord Frivaldo every possible protection, defense and refuge, in
deference to the popular will. Indeed, this Court has repeatedly stressed the importance of giving
effect to the sovereign will in order to ensure the survival of our democracy. In any action involving the
possibility of a reversal of the popular electoral choice, this Court must exert utmost effort to resolve
the issues in a manner that would give effect to the will of the majority, for it is merely sound public
policy to cause elective offices to be filled by those who are the choice of the majority. To successfully
challenge a winning candidate's qualifications, the petitioner must clearly demonstrate that the
ineligibility is so patently antagonistic 68 to constitutional and legal principles that overriding such
ineligibility and thereby giving effect to the apparent will of the people, would ultimately create greater
prejudice to the very democratic institutions and juristic traditions that our Constitution and laws so
zealously protect and promote. In this undertaking, Lee has miserably failed.
In Frivaldo's case, it would have been technically easy to find fault with his cause. The Court
could have refused to grant retroactivity to the effects of his repatriation and hold him still ineligible due
to his failure to show his citizenship at the time he registered as a voter before the 1995 elections. Or,
it could have disputed the factual findings of the Comelec that he was stateless at the time of
repatriation and thus hold his consequent dual citizenship as a disqualification "from running for any
elective local position." But the real essence of justice does not emanate from quibblings over
patchwork legal technicality. It proceeds from the spirit's gut consciousness of the dynamic role of law
as a brick in the ultimate development of the social edifice. Thus, the Court struggled against and
eschewed the easy, legalistic, technical and sometimes harsh anachronisms of the law in order to
evoke substantial justice in the larger social context consistent with Frivaldo's unique situation
approximating venerability in Philippine political life. Concededly, he sought American citizenship only
to escape the clutches of the dictatorship. At this stage, we cannot seriously entertain any doubt about
his loyalty and dedication to this country. At the first opportunity, he returned to this land, and sought to
serve his people once more. The people of Sorsogon overwhelmingly voted for him three times. He
took an oath of allegiance to this Republic every time he filed his certificate of candidacy and during
his failed naturalization bid. And let it not be overlooked, his demonstrated tenacity and sheer
determination to re-assume his nationality of birth despite several legal set-backs speak more loudly,
in spirit, in fact and in truth than any legal technicality, of his consuming intention and burning desire to
re-embrace his native Philippines even now at the ripe old age of 81 years. Such loyalty to and love of
country as well as nobility of purpose cannot be lost on this Court of justice and equity. Mortals of
lesser mettle would have given up. After all, Frivaldo was assured of a life of ease and plenty as a
citizen of the most powerful country in the world. But he opted, nay, single-mindedly insisted on
returning to and serving once more his struggling but beloved land of birth. He therefore deserves
every liberal interpretation of the law which can be applied in his favor. And in the final analysis, over
and above Frivaldo himself, the indomitable people of Sorsogon most certainly deserve to be
governed by a leader of their overwhelming choice.
WHEREFORE, in consideration of the foregoing:
(1) The petition in G.R. No. 123755 is hereby DISMISSED. The assailed Resolutions of the
respondent Commission are AFFIRMED.
(2) The petition in G.R. No. 120295 is also DISMISSED for being moot and academic. In any
event, it has no merit.
No costs.
SO ORDERED.
///////////////////////////////////////////////////////////////////////////////////////////////////////////////////

G.R. No. 83820 May 25, 1990

JOSE B. AZNAR (as Provincial Chairman of PDP Laban in Cebu), petitioner,


vs.
COMMISSION ON ELECTIONS and EMILIO MARIO RENNER OSMEA, respondents.

Rufino B. Requina for petitioner.

Angara, Abello, Concepcion, Regala & Cruz for private respondent.

PARAS, J.:

Before Us is a petition for certiorari assailing the Resolution of the Commission on Elections
(COMELEC) dated June 11, 1988, which dismissed the petition for the disqualification of private
respondent Emilio "Lito" Osmea as candidate for Provincial Governor of Cebu Province.

The facts of the case are briefly as follows:


On November 19, 1987, private respondent Emilio "Lito" Osmea filed his certificate of candidacy with
the COMELEC for the position of Provincial Governor of Cebu Province in the January 18, 1988 local
elections.

On January 22, 1988, the Cebu PDP-Laban Provincial Council (Cebu-PDP Laban, for short), as
represented by petitioner Jose B. Aznar in his capacity as its incumbent Provincial Chairman, filed with
the COMELEC a petition for the disqualification of private respondent on the ground that he is
allegedly not a Filipino citizen, being a citizen of the United States of America.

On January 27, 1988, petitioner filed a Formal Manifestation submitting a Certificate issued by the
then Immigration and Deportation Commissioner Miriam Defensor Santiago certifying that private
respondent is an American and is a holder of Alien Certificate of Registration (ACR) No. B-21448 and
Immigrant Certificate of Residence (ICR) No. 133911, issued at Manila on March 27 and 28, 1958,
respectively. (Annex "B-1").

The petitioner also filed a Supplemental Urgent Ex-Parte Motion for the Issuance of a Temporary
Restraining Order to temporarily enjoin the Cebu Provincial Board of Canvassers from
tabulating/canvassing the votes cast in favor of private respondent and proclaiming him until the final
resolution of the main petition.

Thus, on January 28, 1988, the COMELEC en banc resolved to order the Board to continue
canvassing but to suspend the proclamation.

At the hearing before the COMELEC (First Division), the petitioner presented the following exhibits
tending to show that private respondent is an American citizen: Application for Alien Registration Form
No. 1 of the Bureau of Immigration signed by private respondent dated November 21, 1979 (Exh. "B");
Alien Certificate of Registration No. 015356 in the name of private respondent dated November 21,
1979 (Exh. "C"); Permit to Re-enter the Philippines dated November 21, 1979 (Exh. "D"); Immigration
Certificate of Clearance dated January 3, 1980 (Exh. "E"). (pp. 117-118, Rollo)

Private respondent, on the other hand, maintained that he is a Filipino citizen, alleging: that he is the
legitimate child of Dr. Emilio D. Osmea, a Filipino and son of the late President Sergio Osmea, Sr.;
that he is a holder of a valid and subsisting Philippine Passport No. 0855103 issued on March 25,
1987; that he has been continuously residing in the Philippines since birth and has not gone out of the
country for more than six months; and that he has been a registered voter in the Philippines since
1965. (pp. 107-108, Rollo)

On March 3, 1988, COMELEC (First Division) directed the Board of Canvassers to proclaim the
winning candidates. Having obtained the highest number of votes, private respondent was proclaimed
the Provincial Governor of Cebu.

Thereafter, on June 11, 1988, COMELEC (First Division) dismissed the petition for disqualification for
not having been timely filed and for lack of sufficient proof that private respondent is not a Filipino
citizen.

Hence, the present petition.

The petition is not meritorious.

There are two instances where a petition questioning the qualifications of a registered candidate to run
for the office for which his certificate of candidacy was filed can be raised under the Omnibus Election
Code (B.P. Blg. 881), to wit:

(1) Before election, pursuant to Section 78 thereof which provides that:

'Section 78. Petition to deny due course or to cancel a certificate of candidacy. A


verified petition seeking to deny due course or to cancel a certificate of candidacy may
be filed by any person exclusively on the ground that any material representation
contained therein as required under Section 74 hereof is false. The petition may
be filed at any time not later than twenty-five days from the time of the filing of the
certificate of candidacy and shall be decided, after the notice and hearing, not later
than fifteen days before the election.

and

(2) After election, pursuant to Section 253 thereof, viz:

'Sec. 253. Petition for quo warranto. Any voter contesting the election of any
Member of the Batasang Pambansa, regional, provincial, or city officer on the ground
of ineligibility or of disloyalty to the Republic of the Philippines shall file a sworn
petition for quo warranto with the Commission within ten days after the proclamation
of the results of the election.

The records show that private respondent filed his certificate of candidacy on November 19, 1987 and
that the petitioner filed its petition for disqualification of said private respondent on January 22, 1988.
Since the petition for disqualification was filed beyond the twenty five-day period required in Section 78
of the Omnibus Election Code, it is clear that said petition was filed out of time.

The petition for the disqualification of private respondent cannot also be treated as a petition for quo
warranto under Section 253 of the same Code as it is unquestionably premature, considering that
private respondent was proclaimed Provincial Governor of Cebu only on March 3, 1988.

However, We deem it is a matter of public interest to ascertain the respondent's citizenship and
qualification to hold the public office to which he has been proclaimed elected. There is enough basis
for us to rule directly on the merits of the case, as the COMELEC did below.

Petitioner's contention that private respondent is not a Filipino citizen and, therefore, disqualified from
running for and being elected to the office of Provincial Governor of Cebu, is not supported by
substantial and convincing evidence.

In the proceedings before the COMELEC, the petitioner failed to present direct proof that private
respondent had lost his Filipino citizenship by any of the modes provided for under C.A. No. 63.
Among others, these are: (1) by naturalization in a foreign country; (2) by express renunciation of
citizenship; and (3) by subscribing to an oath of allegiance to support the Constitution or laws of a
foreign country. From the evidence, it is clear that private respondent Osmea did not lose his
Philippine citizenship by any of the three mentioned hereinabove or by any other mode of losing
Philippine citizenship.

In concluding that private respondent had been naturalized as a citizen of the United States of
America, the petitioner merely relied on the fact that private respondent was issued alien certificate of
registration and was given clearance and permit to re-enter the Philippines by the Commission on
Immigration and Deportation. Petitioner assumed that because of the foregoing, the respondent is an
American and "being an American", private respondent "must have taken and sworn to the Oath of
Allegiance required by the U.S. Naturalization Laws." (p. 81, Rollo)

Philippine courts are only allowed to determine who are Filipino citizens and who are not. Whether or
not a person is considered an American under the laws of the United States does not concern Us
here.

By virtue of his being the son of a Filipino father, the presumption that private respondent is a Filipino
remains. It was incumbent upon the petitioner to prove that private respondent had lost his Philippine
citizenship. As earlier stated, however, the petitioner failed to positively establish this fact.

The cases of Juan Gallanosa Frivaldo v. COMELEC et al, (G.R. No. 87193, June 21, 1989)
and Ramon L. Labo v. COMELEC et al (G.R. No. 86564, August 1, 1989) are not applicable to the
case at bar.
In the Frivaldo case, evidence shows that he was naturalized as a citizen of the United States in 1983
per certification from the United States District Court, Northern District of California, as duly
authenticated by Vice Consul Amado P. Cortez of the Philippine Consulate General in San Francisco,
California, U.S.A.

Frivaldo expressly admitted in his answer that he was naturalized in the United States but claimed that
he was forced to embrace American citizenship to protect himself from the persecution of the Marcos
government. The Court, however, found this suggestion of involuntariness unacceptable, pointing out
that there were many other Filipinos in the United States similarly situated as Frivaldo who did not find
it necessary to abandon their status as Filipinos.

Likewise, in the case of Labo, records show that Labo was married to an Australian citizen and that he
was naturalized as an Australian citizen in 1976, per certification from the Australian Government
through its Consul in the Philippines. This was later affirmed by the Department of Foreign Affairs.

The authenticity of the above evidence was not disputed by Labo. In fact, in a number of sworn
statements, Labo categorically declared that he was a citizen of Australia.

In declaring both Frivaldo and Labo not citizens of the Philippines, therefore, disqualified from serving
as Governor of the Province of Sorsogon and Mayor of Baguio City, respectively, the Court considered
the fact that by their own admissions, they are indubitably aliens, no longer owing any allegiance to the
Republic of the Philippines since they have sworn their total allegiance to a foreign state.

In the instant case, private respondent vehemently denies having taken the oath of allegiance of the
United States (p. 81, Rollo). He is a holder of a valid and subsisting Philippine passport and has
continuously participated in the electoral process in this country since 1963 up to the present, both as
a voter and as a candidate (pp. 107-108, Rollo). Thus, private respondent remains a Filipino and the
loss of his Philippine citizenship cannot be presumed.

In the learned dissent of Mr. Justice Teodoro Padilla, he stresses the fact that because Osmea
obtained Certificates of Alien Registration as an American citizen, the first in 1958 when he was 24
years old and the second in 1979, he, Osmea should be regarded as having expressly renounced
Philippine citizenship. To Our mind, this is a case of non sequitur (It does not follow). Considering the
fact that admittedly Osmea was both a Filipino and an American, the mere fact that he has a
Certificate stating he is an American does not mean that he is not still a Filipino. Thus, by way of
analogy, if a person who has two brothers named Jose and Mario states or certifies that he has a
brother named Jose, this does not mean that he does not have a brother named Mario; or if a person
is enrolled as student simultaneously in two universities, namely University X and University Y,
presents a Certification that he is a student of University X, this does not necessarily mean that he is
not still a student of University Y. In the case of Osmea, the Certification that he is an American does
not mean that he is not still a Filipino, possessed as he is, of both nationalities or citizenships. Indeed,
there is no express renunciation here of Philippine citizenship; truth to tell, there is even no implied
renunciation of said citizenship. When We consider that the renunciation needed to lose Philippine
citizenship must be "express", it stands to reason that there can be no such loss of Philippine
'citizenship when there is no renunciation either "'express" or "implied".

Parenthetically, the statement in the 1987 Constitution that "dual allegiance of citizens is inimical to the
national interest and shall be dealt with by law"(Art. IV, Sec. 5) has no retroactive effect. And while it is
true that even before the 1987 Constitution, Our country had already frowned upon the concept of dual
citizenship or allegiance, the fact is it actually existed. Be it noted further that under the aforecited
proviso, the effect of such dual citizenship or allegiance shall be dealt with by a future law. Said law
has not yet been enacted.

WHEREFORE, the petition for certiorari is hereby DISMISSED and the Resolution of the COMELEC is
hereby AFFIRMED.

SO ORDERED.
////////////////////////////////////////////////////////////////////////////////////////////////

Republic Act No. 9225 August 29, 2003

AN ACT MAKING THE CITIZENSHIP OF PHILIPPINE CITIZENS WHO ACQUIRE FOREIGN


CITIZENSHIP PERMANENT.
AMENDING FOR THE PURPOSE COMMONWEALTH ACT. NO. 63, AS AMENDED AND FOR
OTHER PURPOSES

Section 2. Declaration of Policy - It is hereby declared the policy of the State that all Philippine
citizens of another country shall be deemed not to have lost their Philippine citizenship under the
conditions of this Act.

Section 3. Retention of Philippine Citizenship - Any provision of law to the contrary


notwithstanding, natural-born citizenship by reason of their naturalization as citizens of a foreign
country are hereby deemed to have re-acquired Philippine citizenship upon taking the following oath of
allegiance to the Republic:

"I _____________________, solemny swear (or affrim) that I will support and defend the
Constitution of the Republic of the Philippines and obey the laws and legal orders promulgated
by the duly constituted authorities of the Philippines; and I hereby declare that I recognize and
accept the supreme authority of the Philippines and will maintain true faith and allegiance
thereto; and that I imposed this obligation upon myself voluntarily without mental reservation
or purpose of evasion."

Natural born citizens of the Philippines who, after the effectivity of this Act, become citizens of a
foreign country shall retain their Philippine citizenship upon taking the aforesaid oath.

Section 4. Derivative Citizenship - The unmarried child, whether legitimate, illegitimate or adopted,
below eighteen (18) years of age, of those who re-acquire Philippine citizenship upon effectivity of this
Act shall be deemed citizenship of the Philippines.

Section 5. Civil and Political Rights and Liabilities - Those who retain or re-acquire Philippine
citizenship under this Act shall enjoy full civil and political rights and be subject to all attendant
liabilities and responsibilities under existing laws of the Philippines and the following conditions:

(1) Those intending to exercise their right of surffrage must Meet the requirements under
Section 1, Article V of the Constitution, Republic Act No. 9189, otherwise known as "The
Overseas Absentee Voting Act of 2003" and other existing laws;

(2) Those seeking elective public in the Philippines shall meet the qualification for holding
such public office as required by the Constitution and existing laws and, at the time of the filing
of the certificate of candidacy, make a personal and sworn renunciation of any and all foreign
citizenship before any public officer authorized to administer an oath;

(3) Those appointed to any public office shall subscribe and swear to an oath of allegiance to
the Republic of the Philippines and its duly constituted authorities prior to their assumption of
office: Provided, That they renounce their oath of allegiance to the country where they took
that oath;

(4) Those intending to practice their profession in the Philippines shall apply with the proper
authority for a license or permit to engage in such practice; and

(5) That right to vote or be elected or appointed to any public office in the Philippines cannot
be exercised by, or extended to, those who:
(a) are candidates for or are occupying any public office in the country of which they
are naturalized citizens; and/or

(b) are in active service as commissioned or non-commissioned officers in the armed


forces of the country which they are naturalized citizens.

////////////////////////////////////////////////////////////////////////////////////////////

G.R. No. L-27429 August 27, 1969

IN THE MATTER OF THE PETITION FOR ADMISSION AS CITIZEN OF THE PHILIPPINES.


OH HEK HOW, petitioner appellee,
vs.
REPUBLIC OF THE PHILIPPINES, oppositor-appellant.

Eliezer M. Echavez for petitioner-appellee.


Office of the Solicitor General Antonio P. Barredo, Assistant Solicitor General Felicisimo R. Rosete and
Solicitor Santiago M. Kapunan for oppositor-appellant.

CONCEPCION, C.J.:

A decision granting his petition for naturalization as citizen of the Philippines having been rendered on
January 16, 1964, petitioner Oh Hek How filed, on January 17, 1966, a motion alleging that he had
complied with the requirements of Republic Act No. 530 and praying that he be allowed to take his
oath of allegiance as such citizen and issued the corresponding certificate of naturalization. Upon
petitioner's testimony, taken on February 9, 1966, the date set for the hearing of said motion, the Court
of First Instance of Zamboanga del Norte issued forthwith an order authorizing the taking of said oath.
On that same date, petitioner took it and the certificate of naturalization was issued to him.

The Government seasonably gave notice of its intention to appeal from said order of February 9, 1966
and filed its record on appeal. Before the same was approved, it also moved to cancel petitioner's
certificate of naturalization, upon the ground, among others, that it was issued and the oath taken
before said order of February 9, 1966, had become final and executory. Acting upon this motion and
petitioner's opposition thereto, the court issued, on October 3, 1966, an order granting the motion, but,
at the same time, authorizing the taking of a new oath by the petitioner and the issuance in his favor of
another certificate of naturalization, after thirty (30) days from notice to the Solicitor General.
Thereafter, or on November 26, 1966, the court approved the record on appeal and, once more,
authorized the petitioner to "take a new or proper oath to validate the first one made on February 9,
1966." The case is now before us on said record on appeal filed by the Government.

At the outset, it is obvious that the oath of allegiance taken by petitioner on November 28, 1966, and
the certificate of naturalization issued to him in pursuance thereof, as well as the authority given
therefor by the lower court, are null and void. Indeed, the order of February 9, had not and up to the
present has not become final and executory in view of the appeal duly taken by the Government.
What is more, petitioner's second oath was taken, not only after the filing of the notice of appeal 1 and
the submission of the record on appeal, but also after the approval thereof. In other words, the lower
court had already lost its jurisdiction over the case. 2

Again, petitioner's net income in 1960 and 1961 was P3,945.65 and P5,105.79, respectively, or from
about P330 to P425 a month. His income tax return for 1962, filed subsequently to the institution of
this case, showed a net income of P6,485.50 for that year, or about P540 a month. Considering that
petitioner has a wife and three (3) children, one of them of school age, at the time of the filing of his
application for naturalization, his aforementioned income is not a lucrative one. Indeed, it has been
held that the following incomes are not lucrative, from the viewpoint of our naturalization laws, namely:
(1) P4,200 3 or P5,000 a year 4 for one married, with five (5) children; 5 (2) P6,000 a year for one
married, with two (2) minor children; 5 and (3) P6,000 6 or P6,300 a year 7 for one married, with only
one (1) child.

Lastly, it is conceded that petitioner has not required from the Minister of the Interior of Nationalist
China the permission required by the laws thereof for a valid renunciation of his Chinese citizenship.
In Go A. Leng v. Republic, 8 a decision granting the application for naturalization of a Chinese national
was reversed by this Court, upon the ground, among others, of "his failure to secure" the
aforementioned permission.

It is argued that the same is not required by our laws and that the naturalization of an alien, as a
citizen of the Philippines, is governed exclusively by such laws and cannot be controlled by any foreign
law. Section 12 of Commonwealth Act No. 473 provides, however, that before the naturalization
certificate is issued, the petitioner shall "solemnly swear," inter alia, that he renounces "absolutely and
forever all allegiance and fidelity to any foreign prince, potentate" and particularly to the state "of
which" he is "a subject or citizen." The obvious purpose of this requirement is to divest him of his
former nationality, before acquiring Philippine citizenship, because, otherwise, he would have two
nationalities and owe allegiance to two (2) distinct sovereignties, which our laws do not permit, except
that, pursuant to Republic Act No. 2639, "the acquisition of citizenship by a natural-born Filipino citizen
from one of the Iberian and any friendly democratic Ibero-American countries shall not produce loss or
forfeiture of his Philippine citizenship, if the law of that country grants the same privilege to its citizens
and such had been agreed upon by treaty between the Philippines and the foreign country from which
citizenship is acquired." The question of how a Chinese citizen may strip himself of that status is
necessarily governed pursuant to Articles 15 and 16 of our Civil Code by the laws of China, not
by those of the Philippines. 9 As a consequence, a Chinese national cannot be naturalized as a citizen
of the Philippines, unless he has complied with the laws of Nationalist China requiring previous
permission of its Minister of the Interior for the renunciation of nationality.

The view to the contrary, adhered to in Parado v. Republic, 10 Chausintek v. Republic, 11 and Lim So v.
Republic12 has been superseded by our ruling in the subsequent case of Go A. Leng v.
Republic 13 which we hereby reiterate.

WHEREFORE, the order appealed from is reversed, and the oath of allegiance taken, on November
28, 1966, by petitioner Oh Hek How, as well as the certificate of naturalization issued in pursuance
thereto, are hereby declared null and void, with costs against said petitioner, who is, moreover,
directed to surrender the aforementioned certificate of naturalization to the Clerk of the Court of First
Instance of Zamboanga del Norte, within ten (10) days after this decision shall have become final. It is
so ordered.

You might also like